You are on page 1of 110

MCQs

Faculty of General Medicine


VI year
1. Mitral valve according to it’s B. Indentation of the middle third
structure also is called: of the esophagus by an enlarged
A. Bicuspid left atrium
B. Tricuspid C. Notching of the ribs
C. Aortic D. Bounding, full pulse
D. Pulmonary E. Angina pectoris
E. No correct answer
5. Which arrhythmia develops most
2. How many cusps does the right oftenly in patient with mitral
atrioventricular valve have? stenosis?
A. 2 A. Atrial fibrillation
B. 3 B. Atrioventricular block
C. 4 C. Supraventricular tachycardia
D. 5 D. Ventricular fibrillation
E. 6 E. Wolff-Parkinson-White
syndrome
3. Mitral stenosis in 99% of all cases is
caused by: 6. Choose ECG signs typical for atrial
A. Infective endocarditis fibrillation: 1. Negative T wave; 2.
B. Congenital mitral stenosis Absence of P wave; 3. S-T
C. Myxoma elevation; 4. f waves; 5. Different
D. Rheumatic heart disease R-R distance.
E. Mitral annual calcification A. 1, 2, 3
B. 2, 3 4
4. After suffering a streptococcal throat C. 2, 4, 5
infection, a 12-year-old boy D. 1, 3, 5
develops cardiac symptoms that E. All are correct
are attributed to rheumatic fever.
Years later, at the age of 34 he is 7. Choose chest x-ray findings typical
admitted to the hospital with for mitral stenosis: 1. Convexity
pulmonary edema. Further from enlarged left atrial
examination reveals a diastolic appendage 2. Straightening of left
murmur at the apex and mitral heart border 3. Left ventricular
stenosis is diagnosed. Before hypertrophy 4.Large aorta 5.
surgical evaluation, which of the Small aorta
following findings can be A. 1, 2, 3
attributed to mitral stenosis? B. 2, 3, 4
A. Large left ventricle C. All are correct
D. 1, 2, 4
E. 1, 2, 5
2
B. Percutaneous aortic balloon
8. Clinical signs and symptoms of valvuloplasty suffers from a
mitral stenosis develops with the relatively high complication
reduction of the area of mitral and recurrence rate
valve orifice less then: C. Percutaneous aortic balloon
A. 2 cm2 valvuloplasty has a relatively
B. 3 cm2 low complication and
C. 4 cm2 recurrence rate
D. 5 cm2 D. Open mitral commissurotomy
E. 7 cm2 can be successfully performed
if there is limited calcification,
9. Which instrumental examination leaflet stiffness, chordal fusion
is a method of choice to make E. Commissurotomy carries up to
diagnosis and specify valvular a 20% chance of reoperation
heart disease? within 5 years and a 60%
A. ECG chance at 10 years
B. Holter monitoring
C. Doppler ultrasound test 12.Which signs are typical for patients
D. Coronarography with aortic stenosis? Choose
E. Cardiac catheterization correct combination: 1. Dyspnea;
2. Orthopnea; 3. Angina pectoris;
10.Indications for surgical treatment of 4. Syncope; 5. Congestive heart
patients with mitral stenosis are: 1. failure.
Critical mitral stenosis (mitral A. 1, 2, 3
valve orifice area less then 1 cm2); B. 2, 3, 4
2. Symptomatic patients; 3. C. 1, 2, 4
Systemic embolism; 4. Mitral D. 3, 4, 5
valve gradient 12-15 mmHg; 5. E. All are correct
End-diastolic gradient 8-10
mmHg. 13.On auscultation “Austin Flint
A. 1, 2, 3 murmur” is associated with:
B. 2, 3, 4 A. Mitral stenosis
C. 1, 2, 5 B. Aortic regurgitation
D. 2, 3, 5 C. Aortic stenosis
E. All are correct D. Mitral regurgitation
E. Tricuspid regurgitation
11.Which statement is wrong
concerning surgical treatment of 14. A 63-year-old woman fell while
patients with mitral stenosis? crossing the street after her
A. Percutaneous balloon Thursday afternoon bridge game.
valvuloplasty is a new Attempts at resuscitation for cardiac
technique used for minimally arrest by the emergency medical
invasive treatment of patients service (EMS) team were
with mitral stenosis; unsuccessful. The woman had
previously been diagnosed as
3
having aortic stenosis and left infancy. His atrial rate on ECG is
ventricular hypertrophy. In addition 450 bpm, and his ventricular rate is
to these factors, which of the 160 bpm. His pulse rate is 88 bpm.
following predisposes to sudden The left atrium is enlarged. Similar
cardiac death? findings were noted 1 year ago, but
A. Split first heart sound he declined to take any medication.
B. Hypokalemia Treatment should entail which of
C. Soft murmur at left of sternum the following?
that varies with inspiration A. Continue with colonoscopy
D. Failure of the central venous B. Continue with colonoscopy
pressure (CVP) to rise more after administration of
than 1 cm H2O with 30-second parenteral antibiotics
pressure on the liver C. Immediate administration of
(hepatojugular reflux) antibiotics and follow-up
E. CVP of -1 cm H2O colonoscopy at a later date
D. Immediate administration of
15. While lying on the examining table anticoagulation and digoxin
before colonoscopy, a 68-year-old and follow-up colonoscopy at a
electrician notes palpitations. The later date
colonoscopy was scheduled as a E. Immediate electrocardioversion
routine procedure following with a current of 300–400 J
removal of a benign polyp 1 year
earlier. He had rheumatic fever in 17. After experiencing progressive
infancy. His atrial rate on ECG is chest pain for 2 months, a surgical-
450 bpm, and his ventricular rate is supply store owner undergoes a CT
160 bpm. His pulse rate is 88 bpm. scan that reveals a space-occupying
The left atrium is enlarged. Similar lesion of the wall of the left atrium,
findings were noted 1 year ago, but which was confirmed to be
he declined to take any medication. myxoma. There is no evidence of
Make the diagnosis? disease elsewhere. What would the
A. Mitral stenosis, atrial fibrilation next line of treatment be?
B. Mitral stenosis, AV block A. Excision of a myxoma
C. Mitral regurgitation, performed with a bypass
paroxysmal tachycardia procedure
D. Tricuspid regurgitation, atrial B. Excision of a myxoma
fibrilation performed without a bypass
E. Aortic stenosis procedure
C. Insertion of a pacemaker
16. While lying on the examining table D. Chemotherapy
before colonoscopy, a 68-year-old E. Radiotherapy
electrician notes palpitations. The
colonoscopy was scheduled as a 18. Following a car accident, a 52-year-
routine procedure following old lawyer complains of pain in the
removal of a benign polyp 1 year left abdomen and back. After arrival
earlier. He had rheumatic fever in of the EMS team, her pulse rate is
4
84 bpm, but of small volume. She patients with valvular heart
states that she has some cardiac diseases: 1. Valvular calcification,
condition but is uncertain of its 2. Cardiac chamber enlargement, 3.
nature. Which is the most likely Low ejection fraction, 4. Diastolic
cause of the small pulse volume? filling disorder, 5. Pulmonary
A. Aortic regurgitation congestion.
B. Mitral regurgitation A. 1, 2, 5
C. Aortic stenosis B. 1, 3, 4
D. Tricuspid regurgitation C. 1, 4, 5
E. Hyperthyroidism D. 2, 3, 5
E. 2, 3, 4
19. On auscultation “Opening snap
sound” is associated with: 23. Echocardiography allow real-time
A. Mitral stenosis assessment of all, EXCEPT:
B. Aortic regurgitation A. Chamber size
C. Aortic stenosis B. Wall thickness
D. Mitral regurgitation C. Valve appearance
E. Tricuspid regurgitation D. Valve motion
E. Extrasystole presence
20. Choose ECG signs typical for
mitral stenosis: 1. Atrial fibrillation; 24. Cardiac catheterization is used to
2. Left atrium enlargement; 3. Right measure (choose correct answer):
atrium enlargement; 4. Right A. Valve structure
ventricle hypertrophy. B. Valve motion
A. 1, 2, 3 C. Valve gradients
B. 2, 3, 4 D. Valve vegetations
C. 1, 2, 4 E. Chamber size
D. 1, 2
E. All are correct 25. Afterload reduction with
vasodilator agents is applied to
21. The mechanisms that compensate enhance pressure gradients in
hemodynamics derangements patients with valvular heart disease.
associated with valvular heart Choose the right combination of
disease include all, EXCEPT: disorders, in which afterload
A. Atrial chamber enlargement reduction can improve circulation.
B. Ventricular chamber 1. Mitral regurgitation, 2. Aortic
enlargement insufficiency, 3. Aortic stenosis, 4.
C. Myocardial hypertrophy Mitral stenosis.
D. Increased adrenergic A. 1,2
stimulation B. 1,3
E. Hypovolemia C. 1,4
D. 2,3
22. The routine posteroanterior and E. 2,4
lateral chest roentgenogram may
provide nonspecific information in
5
26. Operation for aortic stenosis is
indicated for all symptoms, EXCEPT: 30. In deciding the tactics of treatment
A. LV hypertrophy of patients with severe limb
B. Congestive heart failure ischemia of unknown origin, the
C. Calculated valve area less optimal combination of the
than 0.8 cm2 following instrumental methods of
D. Angina pectoris investigation are: 1.
E. Syncope Sphygmography 2. Aorto-
arteriography 3. Thermography; 4.
27. Mitral stenosis can cause all Ultrasound dopplerography 5.
following conditions, EXCEPT: Occlusive plethysmography. Select
A. Atrial fibrillation the correct combination:
B. Pulmonary hypertension А. 1,2,3;
C. Right ventricular dilation В. 2,3;
D. LV hypertrophy С. 4,5;
E. Hemoptysis D. 2,4;
Е. 1,3,5.
28. For acute occlusion of main arteries
of the extremities are typical: 1. 31. In case of femoral artery embolism,
Paleness; 2. Pain; 3. Edema; 4. ischemia III-B degree (last stage –
Paresthesia; 5. No pulse. Choose the limb contractures) the method of
correct configuration of answers: choice of treatment is:
А. 1,2,4,5; А. Emergency embolectomy;
B. 4,5; В. Thrombolysis;
C. 2,3; С. Anticoagulant therapy;
D. 1,2,3; D. Symptomatic therapy;
E. All correct. Е. Primary amputation of limb.

29. In the absence, in patients with 32. Cause of brachial artery embolism
embolism of femoral artery, of a may be any disease, except:
heart disease as the cause of А. Mitral stenosis;
embolism should be suspected: 1. В. MI;
Aortic aneurysm 2. Iliac artery С. Heart aneurism;
aneurysm, 3. Aorta atheromatosis D. Abdominal aorta aneurism;
with wall surface blood clot 4. Е. Compression of subclavian
Thrombosis of superior mesenteric artery with additional cervical rib.
vein 5. Additional cervical rib,
which compresses the subclavian 33. What changes of coagulation are
artery. Select the correct typical for patients with acute
combination of answers: ischemia of the lower extremity due
А. 1,2,5; to arterial embolism: 1.
В. 3,4,5; Hypercoagulation 2.
С. 1,2,3; Hypocoagulation 3. Inhibition of
D. 4,5; fibrinolysis, 4. Platelets aggregation
E. All correct.
6
5. Activation of fibrinolysys. Select varicosities. On examination, a
the correct combination of answers: bruit is heard over the right groin.
А. 1,4; What is the most likely diagnosis?
В. 2,3,5; A. Femoral artery thrombosis
С 1,3,4; B. Superficial venous
D. 2,3; insufficiency
Е. 3,4. C. Arteriovenous (AV) fistula
D. Pseudoaneurysm
34. The best instrument for E. Deep vein insufficiency
embolectomy from big arteries is:
А. Vessel ring; 38. A 60-year-old man with a history of
В. Vacuum aspirator; atrial fibrillation is found to have a
С. Fogarty catheter; cyanotic, cold right lower extremity.
D. Dormia catheter; The embolus is most probably
Е. Kelly forceps. originating from which of the
following?
35. In the topical diagnosis of acute A. An atherosclerotic plaque
arterial occlusion the most B. An abdominal aortic aneurysm
informative method of C. Heart
examination is: D. Lungs
А. Sphygmography; E. Paradoxical embolus
В. Thermography;
С. Aorto-arteriography; 39. A 60-year-old man with a history of
D. Ultrasound dopplerography; atrial fibrillation is found to have a
Е. Plethysmography. cyanotic, cold right lower
extremity. Which is the most
36. A 63-year-old man has had a common site at which an arterial
cyanotic painful left foot fourth toe embolus lodges?
for 2 days. The dorsalis pedis and A. Aortic bifurcation
posterior tibial arteries are palpable B. Dorsalis pedis artery
on both sides. There is no history of C. Tibial arteries
cardiac or vascular disease. What is D. Popliteal artery
the most likely diagnosis? E. Iliac artery
A. Cardiac embolus
B Atheroembolism 40. A 60-year-old man with a history of
C Lupus vasculitis atrial fibrillation is found to have a
D Digital atherosclerosis cyanotic, cold right lower
E Raynaud’s syndrome extremity. What is the most
appropriate management?
37. A 45-year-old woman undergoes A. Embolectomy
cardiac catheterization through a B. Lumbar sympathectomy
right femoral approach. Two C. Bypass surgery
months later, she complains of right D. Amputation
lower extremity swelling and notes E. Arteriography
the appearance of multiple
7
41. A young patient sustains blunt D. Skin paresthesias
trauma to his right knee that results E. Muscle contracture
in acute thrombosis of his popliteal
artery. Which tissue is most 45. Which clinical sign suggests
sensitive to ischemia? necrosis of the muscles in patients
A. Muscle with acute extremity ischemia?
B. Nerve A. Pain
C. Skin B. Pallor
D. Fat C. Edema
E. Bone D. Paralysis
E. Muscle contracture
42. A young college student injures his
left knee while playing football and 46. Which location IS NOT usual
is unable to bear weight. The source for emboli in case of acute
provisional x-ray report indicates extremity ischemia?
that there are no fractures seen. He A. Mitral valve
is discharged home but presents the B. Left atrium
next morning to the emergency C. Aorta
department with a severely swollen, D. Left ventricle
painful left knee and severe pain in E. Right ventricle
the foot. On examination, the foot is
pale, cold, and pulseless. What is 47. Thrombosis may occur in the
the most likely diagnosis? following settings: 1)
A. Traumatic deep vein thrombosis atherosclerosis, 2)
B. Gastrocnemius muscle tear hypercoagulable states, 3)
C. Traumatic arteriovenous fistula hyperhydration states, 4) vascular
D. Posterior knee dislocation with grafts, 5) systemic anticoagulants
thrombosed popliteal artery use.
E. Traumatic sciatic neuropathy A. 1,3,5
B. 1,2,4
43. Name the earliest sign in case of C. 3,4,5
acute extremity ischemia caused D. 1,2,5
by thromboembolism? E. 2,3,4
A. Pain
B. Pallor 48. Which metabolic derangements are
C. Edema related to reperfusion in patient
D. Paralysis with acute lower-extremity
E. Muscle contracture ischemia after revascularization?
1) acidosis, 2) hyperkalemia, 3)
44. Symptoms of acute arterial alkalosis, 4) hypernatriemia, 5)
occlusion are all except: myoglobinuria.
A. Muscle pain A. 1,2,5
B. Skin pallor B. 2,3,5,
C. Increased pulse on peripheral C. 1,4,5
arteries D. 2,3,5
8
E. 1,3,4 increase collateral perfusion, 5)
decrease severity of the pain.
49. The embolic occlusion should be A. 1,2
suspected in patients with all B. 2,5
following features, EXCEPT: C. 1,4
A. acute onset of the disease D. 2,3
B. prior history of embolism E. 1,4
C. atrial fibrillation
D. history of intermittent 53. Which statement concerning acute
claudication limb ischemia is wrong?
E. MI A. Thrombosis of peripheral
arteries is most oftenly
50. In case of acute limb ischemia so- associated with atherosclerotic
called “5 Ps” have been used as a plaque
mnemonic to remember the B. The extent of collateral flow
clinical presentation of the across the site of occlusion
disease. What are they? determines the severity of
A. palpable cord, painlessness, symptoms.
pallor, pulselessness, paralysis C. Patients with long-standing
B. paresthesia, painlessness, atherosclerotic lesions very
pallor, pulselessness, palpabile quickly develop dramatic
cord symptoms
C. paresthesia, pain, palpable D. Emboli originate from the
cord, pulse paradoxus, paralysis heart in more than 90% of cases
D. palpable cord, pain, pallor, and normally lodge at the site of
pulse paradoxus, painlessness an arterial bifurcation
E. paresthesia, pain, pallor, E. In case of thrombosis
pulselessness, paralysis symptoms may be less dramatic
than embolic occlusion
51. What is the “gold standard” in
evaluation of patient with acute 54. Which of the following is NOT a
limb ischemia? classic risk factor for PE?
A. Aorto-arteriography A. Recent surgery
B. Doppler ultrasonography B. Pregnancy
C. CT C. Oral contraceptives
D. MRI D. Hypertension
E. Coagulogram E. Malignancy

52. The goal of systemic 55. Predominant source of PE is:


anticoagulation, in case of A. Deep lower extremity veins
embolization, are: 1) decrease the B. Left atrium
risk of thrombus propagation, 2) C. Portal vein
prevent recurrent embolization, 3) D. Upper extremity veins
resolve thrombotic formation, 4) E. Mitral valve
9
56. Rare sources of PE are: 1. Portal
vein; 2. Right atrium; 3. Left 60. Which statement is wrong
atrium; 4. Upper extremity veins; 5. concerning test for D-dimer?
Superficial varicose lower extremity A. D-dimer is fibrin degradation
veins. product
A. 1, 3, 4 B. Positive D-dimer test confirms
B. 1, 4, 5 with high probability the
C. 2, 4, 5 diagnosis of PE
D. 2, 3, 4 C. D-dimer can be falsely elevated
E. 3, 4, 5 postoperatively and in the setting
of sepsis, ARDS, MI
57. Pathogeneticaly for PE is typical: D. Negative D-dimer with 98%
A. Decreas of central venous probability exclude the diagnosis
pressure of PE in patients with low PE
B. Increas of arterial blood pressure probability
C. Bradycardia E. D-dimer is indicative of any
D. Splenomegaly thrombosis
E. Increas of central venous
pressure 61. Which statement is wrong
concerning the V/Q scan?
58. What are the most common signs A. Radiolabelled marker is injected
and symptoms of PE? IV
A. Back pain, hypotension, and a B. Patient inhale aerosolized
pulsatile abdominal mass radiolabelled marker
B. Dyspnea, pleuritic chest pain, C. For PE is typical – perfusion
and tachypnea defect and normal ventilation
C. Productive cough, wheezing, and scan
bilateral pedal edema D. For pneumonia is typical –
D. Chest pain, syncope, arterial perfusion and ventilation defect
hypertension E. For PE is typical – normal
E. Arterial hypertension, pain in the perfusion scan and ventilation
thigh defect

59. Choose the combination of tests, 62. Which test is a gold standard in
which are performed in case of diagnosing PE?
suspicion for PE to make A. D-dimer
preliminary diagnosis (baseline B. Angiopulmonography
tests): 1. Chest x-ray; 2. Leg C. Leg doppler
doppler; 3. ECG; 4. Arterial blood D. CT-pulmonary angiorgaphy
gases; 5. D-dimer. E. Echocardiography
A. 1, 2, 3
B. 2, 3, 4 63. 72 year-old patient, with obesity
C. 3, 4, 5 and superficial varicose veins, was
D. 1, 3, 4 admitted to the surgical department
E. 2, 3, 5 with strangulated large incisional
10
hernia. On the next day after 60 mm Hg, heart rate – 130 per
operation patient was complaining minute. Choose the first step in the
for pain in the left thigh. On 3 day treatment of this patient?
after operation during attempt to A. Antibiotics + stabilization of
stand up appeared: inspiratory hemodynamic
dyspnea, dull chest pain, systolic BP B. Thrombolysis + stabilization of
60 mm Hg, heart rate – 130 per hemodynamic
minute. Make the diagnosis? C. Heparin + stabilization of
A. Massive PE hemodynamic
B. Acute myocardial infarction D. Warfarine + stabilization of
C. Minor PE hemodynamic
D. Septic shock E. LMWH + stabilization of
E. Pneumonia hemodynamic

64. 72 year-old patient, with obesity 66. 72 year-old patient, with obesity
and superficial varicose veins, was and superficial varicose veins, was
admitted to the surgical department admitted to the surgical department
with strangulated large incisional with strangulated large incisional
hernia. On the next day after hernia. On the next day after
operation patient was complaining operation patient was complaining
for pain in the left thigh. On 3 day for pain in the left thigh. On 3 day
after operation during attempt to after operation during attempt to
stand up appeared: inspiratory stand up appeared: inspiratory
dyspnea, dull chest pain, systolic BP dyspnea, dull chest pain, systolic BP
60 mm Hg, heart rate – 130 per 60 mm Hg, heart rate – 130 per
minute. Which diagnostic test minute. Which method of treatment
should be performed to confirm the would be most effective in this
diagnosis? case?
A. Chest x-ray A. Medical treatment
B. V/Q scan B. Surgical embolectomy
C. Leg doppler C. Thrombolysis
D. Transesophageal D. Therapy of pulmonary edema
echocardiography E. Heparin
E. D-dimers
67. Which of the following
65. 72 year-old patient, with obesity management options may be used in
and superficial varicose veins, was the treatment of a PE?
admitted to the surgical department A. Anticoagulation
with strangulated large incisional B. Thrombolysis
hernia. On the next day after C. Inferior vena cava (IVC) filter
operation patient was complaining D. Surgical or catheter
for pain in the left thigh. On 3 day embolectomy
after operation during attempt to E. All of the above
stand up appeared: inspiratory
dyspnea, dull chest pain, systolic BP
11
68. Choose the drug which is used for B. Acute myocardial infarction
pathogenetical treatment of PE: C. Major (submassive) PE
A. t-PA D. Minor PE
B. Heparin E. Pneumonia
C. Pentoxyphyllin
D. Warfarin 71. 28 years old man who is in an
E. Aspirin army, presented with sudden onset
of shortness of breath and localised
69. A 30-year-old male is admitted to pleuritic chest pain. He also has
the hospital after a motorcycle non-productive cough, with an
accident that resulted in a fracture of episode of haemoptysis, low grade
the right femur. The fracture is fever without chills and rigors and
managed with traction. Three days rashes. He had previous history of
later the patient becomes confused trauma to the knee and anterior
and tachypneic. A petechial rash is cruciate ligament repair done 1
noted over the chest. Lungs are months ago. No history of
clear to auscultation. Arterial blood pneumonia, recent air travel or long
gases show PO2 of 50, PCO2 of 28, journey drive. No other medical
and pH of 7.49. The most likely illness. Acoording to examination
diagnosis is? 20% occlusion of pulmonary
A. Unilateral pulmonary edema vasculature was find. Which
B. Hematoma of the chest diagnostic test is most suitable to
C. Fat embolism confirm the diagnosis?
D. Pulmonary thromboembolism A. CT-pulmonary angiogram
E. Early Staphylococcus aureus B. ECG
pneumonia C. Chest x-ray
D. Leg doppler
70. 28 years old man who is in an E. D-dimers
army, presented with sudden onset
of shortness of breath and localised 72. 28 years old man who is in an
pleuritic chest pain. He also has army, presented with sudden onset
non-productive cough, with an of shortness of breath and localised
episode of haemoptysis, low grade pleuritic chest pain. He also has
fever without chills and rigors and non-productive cough, with an
rashes. He had previous history of episode of haemoptysis, low grade
trauma to the knee and anterior fever without chills and rigors and
cruciate ligament repair done 1 rashes. He had previous history of
months ago. No history of trauma to the knee and anterior
pneumonia, recent air travel or long cruciate ligament repair done 1
journey drive. No other medical months ago. No history of
illness. Acoording to examination pneumonia, recent air travel or long
20% occlusion of pulmonary journey drive. No other medical
vasculature was find. Make the illness. Acoording to examination
diagnosis? 20% occlusion of pulmonary
A. Massive PE vasculature was find. Which method
12
of treatment would be most A. 1,3,5
effective in this case? B. 2,4,6
A. Antibioticotherapy C. 1,4,5
B. Surgical embolectomy D. 1,4,6
C. Thrombolysis E. 2,3,6
D. Therapy of pulmonary edema
E. Heparin + Warfarine 76. Virchow’s triad, consists of three
components, which predispose a
73. Which of the following are patient to thrombosis. Name them:
contraindications for thrombolysis? 1. Hypoventilation; 2. Endothelial
1. Significant trauma (< 2 months); injury; 3. Stasis of blood flow; 4.
2. Recent major surgery (< 2 Hyperthrombolysis; 5. Blood
months); 3. Prolonged cardio- hypercoagulability
pulmonary resuscitation ( > 10 A. 1,3,5
minutes); 4. Recent puncture of a B. 1,4,5
non-compressible vessel (< 10 C. 2,3,5
days); 5. Bleeding diathesis. D. 1,2,4
A. All of them E. 2,3,4
B. 1, 2, 3
C. 2, 3, 4 77. Which statement, concerning
D. 3, 4, 5 Pulmonary Embolism, is NOT
E. 1, 2, 5 TRUE?
A. Embolism is acute if it occludes
74. The "classic" clinical presentation a vessel
of pulmonary embolism includes B. An embolism is chronic if it is
all, EXCEPT: eccentric and contiguous with
A. abrupt onset the vessel wall
B. acute pleuritic chest pain C. A pulmonary embolism is called
C. shortness of breath massive when it involves both
D. hypoxia pulmonary arteries
E. dehydration D. Routine laboratory findings are
nonspecific
75. Pulmonary embolism can cause E. US dopplerography remains the
such pathological processes: 1. gold standard examination for
Reduces the cross-sectional area of the diagnosis of pulmonary
the pulmonary vasculature; 2. embolism
Increases the cross-sectional area of
the pulmonary vasculature; 3. 78. For massive pulmonary embolism
Reduces pulmonary vascular is typical systolic arterial pressure
resistance; 4. Increases pulmonary less then
vascular resistance; 5. Increases the A. less than 90 mm Hg
right ventricular afterload; 6. B. less than 100 mm Hg
Reduces the right ventricular C. less than 110 mm Hg
afterload. Choose the correct D. more then 120 mm Hg
combination. E. more then 200 mm Hg
13
84. Massive pulmonary embolism is
79. What is NOT a risk factor for caused by occlusion of pulmonary
pulmonary embolism: vasculature:
A. Immobilization A. 10-30%
B. Surgery and trauma B. < 10%
C. Pregnancy C. < 30%
D. Bone fracture D. 30-50%
E. Alcohol abusing E. > 50%

80. The onset of pulmonary embolism 85. All problems that compromise
may include all symptoms, blood flow listed below can cause acute
EXCEPT: visceral ischemia, except?
A. Syncope A. Acute embolic occlusion
B. Arterial hypertention B. Acute thrombotic occlusion
C. Productive cough C. Nonocclusive mesenteric
D. Wheezing ischemia
E. Decreasing level of consciousness D. Splanchnic artery aneurysm
E. Mesenteric veins thrombosis
81. Patients with massive pulmonary
embolism may develop all symptoms, 86. Which clinical sings are typical for
EXCEPT early stages of acute visceral
A. Arterial hypotension ischemia?
B. Tachycardia A. Severe abdominal pain,
C. Polyuria vomiting, diarrhea, leukocytosis
D. Pleuritic chest pain B. Mild abdominal pain,
E. Tachypnea constipation
C. Pulsating abdominal mass
82. Minor pulmonary embolism is D. No typical clinical signs
caused by occlusion of pulmonary E. Bloody stool, signs of
vasculature: peritonitis
A. 50-70%
B. 70-90% 87. Name later manifestations of acute
C. < 30% visceral ischemia?
D. 30-50% A. Severe abdominal pain,
E. >90% vomiting, diarrhea, leukocytosis
B. Mild abdominal pain,
83. Major pulmonary embolism is constipation
caused by occlusion of pulmonary C. Pulsating abdominal mass
vasculature: D. No typical clinical signs
A. 50-70% E. Bloody stool, signs of
B. 70-90% peritonitis
C. < 30%
D. 30-50% 88. Which examination is considered to
E. > 50% be a gold standard for diagnosis of
acute visceral ischemia?
14
A. Ultrasound B. Arteriosclerotic plaque
B. CT C. Mesenteric artery aneurysm
C. Selective mesenteric D. Embolus
angiography E. Vasospasm
D. Duplex ultrasonography
E. MRI 93. A 66-year-old woman is admitted
for hyperalimentation due to
89. Which examination is considered to malnutrition consequent to massive
be the best for screening of acute small-bowel resection. What is the
visceral ischemia caused by thrombotic most likely condition that leads to
ischemia or venous thrombosis? the need to perform a massive
A. Ultrasound resection?
B. CT A. Autoimmune disease
C. Selective mesenteric B. Mesenteric ischemia
angiography C. Mesenteric adenitis
D. Duplex ultrasonography D. Cancer
E. MRI E. Pseudomyxoma peritonei

90. A 60-year-old man with a history of 94. Name the form of acute mesenteric
atrial fibrillation is found to severe ischemia which has the highest mortality
abdominal pain, vomiting, diarrhea, rate?
WBC=23*109/l. The embolus is most A. Acute embolic occlusion
probably originating from which of the B. Acute thrombotic occlusion
following? C. Nonocclusive mesenteric
A. An atherosclerotic plaque ischemia
B. An abdominal aortic aneurysm D. Splanchnic artery aneurysm
C. Heart E. Mesenteric vein thrombosis
D. Lungs
E. Paradoxical embolus 95. The intestine is viable in over 90%
of patients if the duration of mesenteric
91. A 60-year-old man with a history of ischemia symptoms lasts:
atrial fibrillation is found to severe A. 12 hours or less
abdominal pain, vomiting, diarrhea, B. 24 hours or less
WBC=23*109/l. What is the most C. 36 hours or less
appropriate surgical treatment for this D. 48 hours or less
patient? E. 72 hours or less
A. Embolectomy
B. Lumbar sympathectomy 96. Superior mesenteric artery
C. Bypass surgery embolism can be caused by: 1.
D. Intestine resection Aneurysm of the left ventricle after
E. Heparinization myocardial infarction; 2. Atrial
fibrillation; 3. Bacterial
92. Name the most often cause for endocarditis; 4. Right ventricle
mesenteric thrombosis? hypertrophy; 5. Pulmonary artery
A. Blunt abdominal trauma
15
stenosis. Choose correct What causes acute mesenteric
combination: ischemia in this case?
А. 1, 3, 4; A. Acute embolic occlusion of
B. 1, 2, 3; superior mesenteric artery
C. 2, 3, 4; B. Acute thrombotic occlusion
D. 2, 3, 5; of superior mesenteric artery
E. All are correct. C. Nonocclusive mesenteric
ischemia
97. What can cause infarction of the D. Portal vein thrombosis
intestine: 1. Superior mesenteric artery E. Mesenteric vein thrombosis
embolism; 2. Superior and inferior
mesenteric artery embolism; 3. 100. Which parts of the GI tract will be
Superior mesenteric artery thrombosis; ischemic in case of thrombosis of the
4. Superior mesenteric vein orifice of superior mesenteric artery?
thrombosis; 5. Prolonged spasm of K. Stomach and duodenum
small intestine arteries. Choose the L. Stomach, duodenum and
BEST combination. ileum
A. 1, 2, 3 M. Small intestine, cecum,
B. 2, 3, 4 colon ascendance
C. 3, 4, 5 N. Colon and rectum
D. All diseases can cause O. All parts of small and large
intestine infarction intestine
E. None of these diseases can
cause intestine infarction 101. Which operations we can perform
in case of embolic occlusion of
98. In a patient with superior superior mesenteric artery (choose the
mesenteric artery embolism in stage of best combination): 1. Embolectomy; 2.
bowel infarction (part of small intestine Embolectomy and resection of part of
necrotised) should be performed the small intestine; 3. Embolectomy and
following operation: left hemicolectomy; 4. Embolectomy
F. Thrombectomy and right hemicolectomy; 5. Total
G. Isolated embolectomy excision of ileum, jejunum and right
H. Embolectomy and hemicolectomy.
resection of necrotised P. 1, 2, 3
intestine Q. 2, 3, 4
I. Total colectomy R. 1, 4, 5
J. Periarterial sympathectomy S. 1, 2, 4
T. All operations can be
99. A 42-years-old patient, who suffers performed
from mitral stenosis and atrial
fibrillation, 6 hours ago appeared 102. Most often cause of acute
severe abdominal pain, vomiting, mesenteric ischemia is:
diarrhea. On examination: tenderness A. Embolisation to the
in mesogastrium, negative Blumberg superior mesenteric artery
sign. CBC: Leukocytes – 21*109/l.
16
B. Thrombosis of superior
mesenteric artery 105. A 68-year-old man is admitted to
C. Nonocclusive mesenteric the coronary care unit with an
ischemia acute myocardial infarction. His
D. Portal vein thrombosis postinfarction course is marked by
E. Mesenteric vein thrombosis congestive heart failure and
intermittent hypotension. On the
103. What can cause nonocclusive fourth hospital day, he develops
mesenteric ischemia (NOMI)? 1. severe midabdominal pain. On
Severe mesenteric vasoconstriction; 2. physical examination, blood
Myocardial infarction; 3. Septic shock; pressure is 90/60 mm Hg and pulse
4. Hemorrhagic shock; 5. Cardiac is 110 beats/min and regular; the
decompensation. abdomen is soft with mild
U. 1, 2, 3 generalized tenderness and
V. 2, 3, 5 distention. Bowel sounds are
W. 2, 3, 4 hypoactive; stool hematest is
X. None of these diseases positive. The next step in this
cause NOMI patient’s management should be
Y. All these diseases cause which of the following?
NOMI A. Barium enema
B. Upper gastrointestinal
104. In patients with acute mesenteric endoscopy
ischemia due to mesenteric C. Angiography
embolism, which of the following D. Ultrasonography
statements is correct? E. Celiotomy
A. Most oftenly embolization
to inferior mesenteric 106. Most oftenly chronic lower
artery is observed extremities ischemia is caused by:
B. Embolus most oftenly A. Buerger’s disease
origins from right heart B. Atherosclerosis
C. Thrombolytic therapy may C. Popliteal artery entrapment
be attempted in patients D. Mucinous cystic degeneration
without signs of bowel E. Fibrodysplasia
infarction or
gastrointestinal bleeding 107. All statements concerning
D. Arteriography usually atherosclerosis are true EXCEPT:
reveals the embolus lodged A. Atherosclerosis results in lipid
at the orifice of the accumulation within the intimal
superior mesenteric artery layer of blood vessels
E. At the time of exploration B. Is the basis of most peripheral
in case of superior vascular disease
mesenteric artery C. Atherosclerosis affects most
embolism, ischemia is oftenly young patients
most severe in the left D. Is the major degenerative disease
colon of arteries
17
E. Oftenly occurs as a result of the exercise-induced increase in
aging process blood flow
E. Is lower extremity muscular pain
108. Major risk factors for induced by exercise
atherosclerosis are all EXCEPT:
A. Cigarette smoking 112. Asymptomatic clinical stage of
B. Hypertension chronic lower extremity ischemia
C. Diabetes mellitus can be diagnosed by signs: 1.
D. Hyperlipidemia Ulcers; 2. Muscles atrophy; 3.
E. Alcohol consumption Hyperkeratosis; 4. Diminished hair
and nail growth on affected limb
109. According to the traditional and digits; 5. Claudication. Choose
Fontaine classification system for correct combination.
lower extremity arterial occlusive A. 1, 2, 3
disease II stage means: B. 2, 3, 4
A. Asymptomatic C. 3, 4, 5
B. Claudication D. 1, 3, 5
C. Ischemic rest pain E. 2, 4, 5
D. Ischemic ulceration
E. Ischemic necrosis 113. Critical limb ischemia is
characterized by: 1. Rest pain; 2.
110. According to the traditional Fontaine stage II; 3. Pedal necrosis;
Fontaine classification system for 4. Intermittent claudication; 5.
lower extremity arterial occlusive Fontaine stages III and IV:
disease III stage means: A. 1, 2, 3
A. Asymptomatic B. 1, 2, 4
B. Claudication C. 2, 3, 4
C. Ischemic rest pain D. 1, 3, 5
D. Ischemic ulceration E. 2, 3, 5
E. Ischemic necrosis
114. Which examination or test is used
111. Choose not correct statement as the most common tool to
concerning intermittent diagnose peripheral vascular disease
claudication: and stratify objectively the extent of
A. Is clinically diagnosed as rest occlusive disease?
pain A. Phlebography
B. Is relieved with short periods of B. Duplex ultrasound
rest C. Angiography
C. Pain is located in the calves (less D. Echocardiography
frequently in the buttocks or E. Ankle-brachial pressure index
thighs)
D. Is caused by arterial obstruction 115. Initial nonoperative treatment of
proximal to affected muscle patients with peripheral artery
beds, which limits the normal disease in the I-II stage according to
Fontaine classification consists of
18
such components: 1. Adequate pain in both lower and upper
antiplatelet therapy; 2. extremities, migratory superficial
Anticoagulant therapy; 3. phlebitis of the feet occurring a few
Antibiotics; 4. Hypertension years ago. Physical examination
control; 5. Treatment of findings are remarkable for absent
hyperlipidemia. Choose best bilateral posterior tibial and dorsalis
combination: pedis pulses with palpable popliteal
A. 1, 2, 3 pulses.What is the single most
B. 1, 3, 4 important step in management?
C. 1, 4, 5 A. Multiple toe amputations
D. 2, 3, 4 B. Long-term anticoagulant therapy
E. 3, 4, 5 C. Immediate operative intervention
D. Angiography followed by bypass
116. Which antiplatelet agent is used surgery
most oftenly? E. Cessation of smoking
A. Aspirin
B. Clopidogrel 119. A middle-aged man undergoes a
C. Ticlopidine left below-knee amputation for left-
D. Heparin foot gangrene secondary to arterial
E. Pentoxyphyllin occlusive disease. Which of the
following statements is true after the
117. A 27-year-old chronic smoker below-knee amputation?
presents with ulceration of the tip A. There is less efficient function
of the right second, third, and than after above-knee
fourth toes. He gives a history of amputation
recurrent pain in both lower and B. Stump prognosis can be judged
upper extremities, migratory by transcutaneous oxygen
superficial phlebitis of the feet monitoring
occurring a few years ago. C. Poor prognosis is inevitable if
Physical examination findings are Doppler fails to record a pulse at
remarkable for absent bilateral that level
posterior tibial and dorsalis pedis D. Is performed in patients with
pulses with palpable popliteal occlusion of femoral artery
pulses. Make the diagnosis? E. The level of transection is 5 cm
A. Thromboangiitis obliterans above the medial malleolus
(Buerger’s disease)
B. Lupus vasculitis 120. A 70-year-old man with a long-
C. Atheroembolism standing history of diabetes
D. Raynaud’s syndrome develops gangrene of the right
E. Atherosclerosis obliterans second toe. What is true of his
diabetic foot?
118. A 27-year-old chronic smoker A. Dorsalis pedis and posterior
presents with ulceration of the tip of tibial arteries pulses are always
the right second, third, and fourth absent
toes. He gives a history of recurrent
19
B. Gangrene of the toe always symptoms or signs of arterial
requires urgent below-knee insufficiency?
amputation A. Ischemic ulceration
C. Arterial reconstruction is B. Ischemic neuropathy (not
invariably required diabetic neuropathy)
D. His right common femoral artery C. Claudication
is most probably occluded or D. Nocturnal foot pain
stenosed E. Toe gangrene
E In pathogenesis of diabetic foot
development big role play 124. Symptoms or signs of
stenosis of lower extremity atherosclerotic occlusive disease of
arteries and progression of the bifurcation of the abdominal
peripheral neuropathy aorta (Leriche syndrome) include?
A. Claudication of the buttock and
121. An elderly patient with ischemic thigh, impotence
rest pain is found to have combined B. Causalgia of the lower leg
aortoiliac and femoropopliteal C. Retrograde ejaculation
occlusive disease. What is the D. Gangrene of the feet
treatment of choice in this case? E. Dependent rubor of the feet
A. Lumbar sympathectomy
B. Femoropopliteal bypass 125. On which artery we measure pulse
C. Aortofemoral and below the middle of inguinal
femoropopliteal bypass ligament?
D. Aortofemoral bypass A. A. iliaca interna
E. Vasodilator therapy B. A. femoralis communis
C. A. poplitea
122. A 72-year-old man complains of D. A. profunda femoris
bilateral thigh and buttock E. A. iliaca externa
claudication of several months
duration. He was told by his 126. Choose all pathological processes
physician that the angiogram which are responsible for the
revealed findings indicating that he development of varicosae veins: 1.
has Leriche syndrome. What does Venous valvular incompetence; 2.
this patient have? Venous hypertension; 3. High
A. Abdominal aortic aneurysm intraabdominal preassure; 4.
B. Aortoiliac occlusive disease Endothelial function imparement; 5.
C. Iliac artery aneurysm Smooth muscle function
D. Femoropopliteal occlusive imparement.
disease A. 1, 2, 3;
E. Tibial occlusive disease B. 2, 3, 5;
C. 1, 3, 5;
123. Conservative management rather D. 2, 4, 5;
than reconstructive arterial surgery E. All are correct.
is generally recommended for
patients with which of the following 127. The greater saphenous vein arises:
20
A. Anterior to the medial malleolus; 132. Choose all etiological factors
B. Posterior to the medial which can cause varicose veins: 1.
malleolus; Pregnancy; 2. Chronic straining; 3.
C. Anterior to the lateral malleolus; Prolonged standing; 4. Pelvic veins
D. Posterior to the lateral malleolus; obstruction; 5. Obesity.
E. Below the knee joint. A. 1, 2, 3;
B. 2, 3, 5;
128. Most oftenly the greater C. 1, 3, 5;
saphenous vein join: D. 2, 4, 5;
A. External iliac vein; E. All are correct.
B. Common femoral vein;
C. Popliteal vein; 133. Choose symptoms which are
D. Posterior tibial vein; typical for early stages of varicose
E. Deep femoral vein. veins: 1. Ankle swelling; 2. Leg
heaviness; 3. Cutaneous
129. The lesser saphenous vein arises: pigmentation; 4. Ulceration; 5.
A. Anterior to the medial malleolus; Cosmetically displeasing dilated
B. Posterior to the medial superficial veins.
malleolus; A. 1, 2, 3;
C. Anterior to the lateral malleolus; B. 2, 3, 4;
D. Posterior to the lateral malleolus; C. 1, 2, 5;
E. Below the knee joint. D. 2, 3, 4;
E. All are correct.
130. Most oftenly the lesser saphenous
vein join: 134. All statements about pain in
A. External iliac vein; patients with varicose veins are true
B. Common femoral vein; except:
C. Popliteal vein; A. The most frequent type is limb
D. Posterior tibial vein; heaviness;
E. Anterior tibial vein. B. Ache that occurs after prolonged
standing;
131. All statements about veins of C. The pain is usually felt over the
lower extremities are true except: calf area;
A. 90% of blood is returning to the D. Walking may increase the calf
heart by deep vein system; ache associated with varicose
B. The leg muscles act like a heart veins;
for veins; E. Lying down, particularly with
C. Backward floow of the blood in elevation of the limb, relieves
veins is prevented by valves; limb heaviness within a short
D. Blood flows from superficial to period of time.
deep venous system by
perforator veins; 135. Varicose veins are defined as
E. Superficial vein system consists dilated palpable subcutaneous
most oftenly from 3 veins. veins larger than:
A. 4 mm;
21
B. 7 mm; B. 2,3;
C. 10 mm; C. 3,4;
D. 2 mm; D. 1, 2;
E. 1 mm. E. All are correct.

136. Which statement is wrong 139. Name main principles of


concerning venous imaging studies? nonsurgical treatment of not
A. Duplex scanning is screening complicated varicose veins: 1.
method for patients with Rest; 2. Exercises; 3. Leg
varicose veins; elevation; 4. Elastic compression;
B. For all patients with varicose 5. Lower leg. Choose best
veins phlebography is indicated; combination:
C. Phlebography and duplex A. 2, 3, 5;
scanning provide detailed B. 1, 2, 5;
anatomic information; C. 1, 3, 4;
D. Phlebography is indicated in D. 2, 4, 5;
complex cases (vein valve E. 1, 2, 4.
transplantation or multiple re-do
procedures); 140. A 21-year-old woman is referred
E. Duplex scanning allows vein to your office because of multiple
mapping, perforator mapping. lower extremity varicose veins. She
has large varicosities in the
137. Venous disease of the legs can be distribution of the long saphenous
classified according to the vein, she was never examined for
severity, cause, site and specific varicose veins. What is the next step
abnormality using the CEAP in management?
classification. What C1 means A. A ligation and stripping
according to this classification? operation;
A. No visible or palpable signs of B. Ligation of both the long and
venous disease; short saphenous system;
B. Telangiectases, reticular veins, C. Sclerotherapy;
malleolar flare; D. Duplex evaluation along with
C. Varicose veins; clinical correlation as an essential
D. Edema without skin changes; initial step;
E. Skin changes ascribed to venous E. Compression stockings and
disease anticoagulation therapy.

138. Specify most efficient and 141. A middle-age woman has right
noninavasive examinations, which foot nonpitting edema, leg
helps to identify the perforator heavyness. The diagnosis of
incompetence: 1. Torniquet test. 2. chronic venous insufficiency is
Duplex scanning. 3. Phlebography. made, stage C1. What is the
4. Arteriography. 5. March test. treatment of choice?
Choose best combination: A. Vein stripping;
А. 2,5; B. Pressure-gradient stockings;
22
C. Skin grafting; B. Sclerotherapy is not a
D. Perforator vein ligation; reasonable treatment option
E. Valvuloplasty. C. There is no association with deep
venous insufficiency
142. A patient complains of an D. These veins will never bleed
ulceration on the inner surface of E. Walking will likely improve her
the lower third of the right shin. symptoms
On xamination: the ulcer is round-
shaped, up to 5 cm in diameter, 144. Development of chronic venous
with sloping edges. On the inner insufficiency of lower extremities
surface of this shin there are depends from the functional
varicose veins. What complication condition of so-called "muscle
appeared in this patient? pump". In relation to which muscle
A. Elephantiasis group isused this term?
B. Erysipelatous inflammation A. Buttocks
C. Deep venous thrombosis B. Abdominal wall
D. Varicose veins with trophic C. Foot
ulceration D. Hip
E. Popliteal artery thrombosis E. Shin

143. A 61-year-old woman comes to 145. Operation for varicose veins of


the office complaining of ”spidery lower extremities allows to: 1. To
veins”. Occasionally, she has a dull, overcome abnormal dump of blood
achy feeling in her legs that usually from deep veins to superficial; 2.
occurs at the end of the day. Remove varicose veins; 3. Restore
Recently her shoes have begun to patency of deep veins; 4. Adjust
feel tight at the end of the day. She femoral vein valve failure; 5.
used to work as a bank teller, Remove trophicaly changed tissues.
spending many hours on her feet Select best combination of answers:
each day. She denies any shortness A. 3,4,5
of breath or difficulty walking up B. 1,2,4
stairs. There is no history of deep C. All are correct
vein thrombosis. On physical D. 1,2,3
examination her legs are symmetric E. 2,3,4
in size, without evidence of trauma
or skin breakdown. On the inner 146. Reticular veins are defined as
aspect of her upper and lower thigh dilated nonpalpable subcutaneous
are dilated superficial veins. There veins:
are good dorsal pedalis pulses A. 1-4 mm;
bilaterally, and motor and sensory B. 5-10 mm;
examination is normal. The most C. 10-15 mm;
accurate statement regarding her D. > 15 mm;
condition is: E. < 1 mm.
A. Compression stockings may
provide relief
23
147. Teleangiectases are defined as Lung cancer; 3. Genitourinary
dilated nonpalpable subcutaneous cancer; 4. CNS cancer; 5.
veins: Gastrointestinal cancer.
A. 1-4 mm; A. 2, 3, 4;
B. 5-10 mm; B. 2, 3, 5;
C. 10-15 mm; C. 1, 2, 3;
D. > 15 mm; D. 3, 4, 5;
E. < 1 mm. E. All are correct

148. Specify factors, which plays 152. In women, especially in young


important role in the pathogenesis age, deep vein thrombosis can be
of deep vein thrombosis: 1. Stasis; caused by: 1. Pregnancy; 2.
2. Hypocoagulability; 3. Abortion; 3. Oral contraceptives; 4.
Hypercoagulability; 4. Vein wall Hormonal replacement therapt.
(endothelial) injury; 5. Allergic A. All are correct;
reaction. B. 1, 2, 3;
A. 1, 2, 3; C. 2, 3, 4;
B. 1, 3, 4; D. 1, 2, 4;
C. 1, 2, 4; E. 1, 3, 4.
D. 2, 3, 5;
E. All are correct. 153. Name the signs of acute deep vein
thrombosis: 1. Thigh pain; 2.
149. Where deep vein thrombosis most Massive leg swelling; 3. Pale colour
oftenly begins: of the thigh skin; 4. Early leg
A. Foot; paralysis; 5. Leg cyanosis.
B. Anlke; A. 1, 2, 3;
C. Calf; B. 2, 3, 4;
D. Knee; C. 3, 4, 5;
E. Thigh D. 1, 2, 5;
E. 2, 3, 5
150. Choose the risk factors for the
development of deep vein 154. Homans’ sign – is:
thrombosis: 1. Advanced age; 2. A. Thigh swelling;
Immobilization; 3. Active cancer; 4. B. Leg cyanosis;
Orthopedic procedures; 5. Primary C. Pain with passive dorsiflexion of
hypercoagulable states. the foot;
A. 1, 2, 3; D. Tenderness of the calf;
B. 2, 3, 4; E. Prominent superficial veins in
C. 1, 3, 5; case of deep vein thrombosis
D. 2, 3, 5;
E. All are correct. 155. Which statement concerning D-
dimer are not true?
151. What cancers are most suspicios A. D-dimers are products of the
for the development of deep vein degradation of cross-linked
thrombosis: 1. Thyroid cancer; 2. fibrin by plasmin;
24
B. D-dimer blood levels reflect the E. Assay of fibrin/fibrinogen
presence of intravascular fibrin; products
C. D-dimer is sensitive for the
diagnosis of venous 158. Name the most dangerous
thromboembolism complication of deep vein
D. D-dimer measurement is most thrombosis:
valuable as an adjunct to other A. Venous ulcers;
diagnostic modalities (duplex B. Pulmonary embolism;
scanning etc) C. Phlebitis;
E. Increased level of D-dimer is D. Valvular incompetency;
highly specific for deep vein E. Obliteration of lower extremity
thrombosis deep veins.

156. Four days after undergoing 159. A middle-aged man known to


subtotal gastrectomy for stomach have peptic ulcer disease is admitted
cancer, a 58-year-old woman with upper gastrointestinal bleeding.
complains of right leg and thigh During his hospital stay, he
pain, swelling and redness, and has develops DVT of the left lower
tenderness on examination. The extremity. What is the most
diagnosis of deep vein thrombosis is appropriate management?
entertained. What is the initial A. Anticoagulants
screening test to establish the B. Observation
diagnosis? C. Thrombolytic therapy
A. Venography D. Inferior vena cava (IVC) filter
B. Venous duplex ultrasound E. Aspirin
C. Impedance plethysmography
D. Radio-labeled fibrinogen 160. An 18-year-old man develops a
E. Assay of fibrin/fibrinogen painful, swollen left leg while
products training for the Kiev Marathon.
There is tenderness in the left calf
157. Four days after undergoing and ecchymosis is present. No signs
subtotal gastrectomy for stomach of varicose veins on both legs,
cancer, a 58-year-old woman active and passive movements in the
complains of right leg and thigh left knee joint are not painfull. What
pain, swelling and redness, and is the most likely diagnosis?
has tenderness on examination. A. Cellulitis;
The diagnosis of deep vein B. Deep vein thrombosis;
thrombosis is entertained. What is C. Superficial thrombophlebitis;
the most efficient and sensitive D. Tear of the plantaris muscle;
(gold standard) test to establish the E. Popliteal artery embolism
diagnosis?
A. Venography 161. Choose the best combination of
B. Venous duplex ultrasound deep vein thrombosis prevention
C. Impedance plethysmography methods in the patient of high risk
D. Radio-labeled fibrinogen group: 1. Early activation of the
25
patient after surgical procedure; 2. patients; 5. Young age of the
Low-molecular weight heparins; 3. patient with deep vein thrombosis.
Graduated-compression stockings; A. 2, 3, 4;
4. Intermittent leg compression; 5. B. 1, 2, 3;
Regional anesthesia (intraspinal or C. 3, 4, 5;
epidural anesthesia). D. 1, 2, 5;
A. 1, 2, 3; E. 1, 3, 5
B. 2, 3, 4;
C. 3, 4, 5; 165. Prophylactic regimens of
D. 1, 2, 4; documented benefit in decreasing
E. All are correct the risk of POSToperative
thromboembolism include:
162. Treatment of deep vein A. Early activation and ambulation
thrombosis is directed towards all B. External pneumatic compression
factors except: devices placed on the upper
A. To prevent death from extremities
pulmonary embolism; C. Long bedrest after operation
B. To prevent recurrent deep D. Leg elevation for 24 h
vein thrombosis; postoperatively
C. To prevent development of E. Dipyridamole therapy for 48 h
varicose veins; postoperatively
D. To prevent the post-
thrombotic syndrome; 166. Patients with phlebographically
E. To prevent proximal confirmed deep vein thrombosis of
propagation of the thrombosis the calf:
A. Can expect asymptomatic
163. Which thrombolytic agent is used recovery if treated promptly with
for the treatment of deep vein anticoagulants
thrombosis? B. May be effectively treated with
A. Aspirin; low-dose heparin
B. Recombinant tissue C. May be effectively treated with
plasminogen activator; pneumatic compression
C. Heparin stockings
D. Fondaparinux (Arixtra); D. May be effectively treated with
E. Pentoxyphyllin acetylsalicylic acid
E. Are at risk for significant
164. Specify idications for inferior pulmonary embolism
vena caval interruption: 1.
Contraindications for thromlytics in 167. Choose the indication for
the low risk group patient; 2. placement of inferior vena cava
Contraindications to anticoagulants; filter:
3. Recurrent pulmonary embolism A. Axillary vein thrombosis
despite adequate anticoagulation; 4. B. Recurrent pulmonary embolus
Prophylactic placement in high-risk despite adequate anticoagulation
therapy
26
C. Pulmonary embolism in a patient pulmonary embolism. Choose
with a hypercoagulable condition WRONG statement concerning
D. Pulmonary embolus due to deep superficial vein thrombophlebitis:
vein thrombosis of the lower A. Superficial vein
extremity that occurs 2 weeks thrombophlebitis is a life-
postoperatively threatening condition with high
E. Pulmonary embolus in a patient risk of pulmonary embolism
with metastatic pancreatic B. The most common source of
carcinoma trauma associated with
superficial vein thrombophlebitis
168. Choose ALL risk factors for is an intravenous cannula
development of superficial vein C. Migratory thrombophlebitis may
thrombophlebitis: 1. Varicose veins: be associated with occult
2. Age > 60; 3. Obesity; 4. Tobacco malignancy
smoking; 5. History of deep vein D. Mondor’s disease is a
thrombosis. thrombophlebitis of
A. 1, 2, 3 thoracoepigastric vein of the
B. 1, 3, 4 brest and chest wall
C. 2, 3, 4 E. Lesser saphenous vein superficial
D. 1, 3, 5 vein thrombophlebitis may progress
E. All are risk factors for superficial into popliteal deep vein thrombosis
vein thrombosis
171. Which condition is most
169. The physical diagnosis of prothrombotic amoung all inherited
superficial thrombophlebitis is thrombophilias?
based on the presence of: 1. A. Antithrombin III deficiency
Erythema in distribution of the B. Protein C deficiency
superficial veins; 2. Pale lower C. Protein S excess
extremity; 3. Tenderness in the D. Antiphospholipid antibodies
distribution of the superficial veins; E. Hypocysteinemia
4. Palpable and painfull cord in
distribution of the superficial veins; 172. The factors, which propagate
5. Pulsless lower extremity. thrombosis during pregnancy are all
A. 1, 3, 4 except:
B. 1, 2, 3 A. Increased fibrinogen level
C. 2, 3, 4 B. Decreased fibrinolytic activity
D. 2, 3, 5 C. Reduction in protein S level
E. All are clinical signs of D. Increased levels of factors VII,
superficial thrombophlebitis VIII, IX, X
E. Increased level of protein C
170. Superficial vein thrombophlebitis
is a relatively common disorder 173.Which statement concerning
with a significant incidence of thrombotic predisposing factors, is
recurrence and has potential wrong?
morbidity from extension and
27
A. Women have a slight
predilection over men 177. The laboratory evaluation for
B. No racial predilection factor-related hypercoagulability
C. Age may be a predisposing conditions includes measurement of all
factor in both SVT, DVT the following, except:
D. Oral contracept intake A. Prothrombin time, activated
predispose for venous partial thromboplastin time
thrombosis B. Protein C and protein S level
E. Decreased total platelet count C. Antithrombin level
D. Homocysteine level
174. Which statement is not correct? E. D-dimers
A. Superficial thrombophlebitis
usually occurs over a previous 178. All statements concerning heparin
varicose vein are true, except:
B. Pain associated with SVT is A. Causes inhibition of thrombin
usually localized over the site of B. Prevents formation and/or
thrombosis. extension of thrombus
C. Pain associated with DVT is C. Allows recanalization of the
generally diffuse blood vessel over time
D. Recent surgery, immobilization D. Decrease the level of protein C
are factors that can contribute to E. Can be used IV
SVT or DVT
E. Bilateral extremity swelling is 179. Oral anticoagulants are used: (1)
suspectious for superficial to prevent recurrent thrombotic events,
thrombophlebitis (2) for long-term outpatient therapy in
patients, (3) to inhibite vitamin K
175. The classic findings of SVT are a metabolism, (4) to treat acute
firm(1), tender(2), erythematous thromboflebitis, (5) for resolution of
fibrous cord(3), evidence of heart thrombi.
failure(4), positive Homan’s sign(5): A. 1,2,3
A. 1,2,3 B. 1,2,5
B. 1,3,5 C. 2,3,5
C. 2,3,5 D. 3,4,5
D. 3,4,5 E. 1,4,5
E. 1,4,5
180. What medicines have no
176. The signs of DVT are painfull significant role for therapy of venous
calf(1), swollen leg(2), erythematous thrombosis?
fibrous cord(3), evidence of heart A. Antiplatelet agents
failure(4), positive Homan’s sign(5): B. Protein C concentrate
A. 1,2,3 C. Thrombolytic agents
B. 1,2,5 D. Heparin
C. 2,3,5 E. Oral anticoagulants (Coumadin)
D. 3,4,5
E. 1,4,5
28
181. A 24-year-old woman on oral extremity. Objectively: moderate
contraceptive pills develops an episode edema of shin, brown induration of
of deep vein thrombosis that is skin in the lower third of shin, varix
adequately treated with dilatation of superficial shin veins are
anticoagulation. She is at increased risk present. What is the most probable
of developing which of the following? diagnosis?
A. Recurrent foot infections A. Parkes-Weber syndrome
B. Claudication B. Lymphedema of lower right
C. Pulmonary embolism extremity
D. Postphlebitic syndrome C. Acute thrombophlebitis of
E. Superficial varicose veins superficial veins
D. Postthrombophlebitic syndrome,
182. A 37 y.o. patient complains of varicose form
pain in the right arm which increases E. Acute thrombosis of deep veins
during motion, raised body temperature
up to 390C. In the right cubital fossa 185. Venous insufficiency in the deep
there is a trace of injection, hyperemia or superficial system causes all
and thickening along the vein. Your except:
diagnosis? A. pain
A. Inflammation of lymph B. swelling
B. Phlegmon C. skin changes
C. Phlebit D. ulcerations
D. Erysipelas E. intermittent claudication
E. Abscess
186. Which statement is wrong?
183. A 28 y.o. woman comes to the A. Patients with deep venous
Emergency Room with a slightly system insufficiency nearly
reddened, painful "knot", 8 cm above always are symptomatic;
the medial malleolus. Examination in B. Pain caused by venous
the standing position demonstrates a insufficiency often is improved
distended vein above and below the by walking or by elevating the
mass. There are no other abnormalities legs;
on physical examination. The most C. Warmth tends to aggravate the
likely diagnosis is: symptoms of venous
A. Cellulitis insufficiency, and cold tends to
B. Early deep vein thrombosis relieve them;
C. Subcutaneous hematoma D. Compression stockings usually
D. Insect bite ameliorate or prevent the pain in
E. Superficial venous thrombosis patients with chronic arterial
insufficiency;
184. A 43 year old patient had right- E. Leg aching, heaviness, and
sided deep vein thrombosis of soreness are the most common
iliofemoral segment 3 years ago. Now subjective symptoms of venous
he is suffering from the sense of insufficiency.
heaviness, edema of the lower right
29
187. The most common signs of E. D-dimmer
venous system insufficiency are:
Pitting edema(1), 191. Which functional tests can
Hyperpigmentation(2), Paresthesia(3), physician provide to evaluate both
Ulceration(4), “Cold extremity”(5). deep and superficial venous
A. 1,2,4 system: (1) venous refilling time,
B. 1,3,4 (2) the maximum venous outflow,
C. 1,3,5 (3) calf muscle pump ejection
D. 2,3,5 fraction, (4) peak arterial velocity,
E. 2,4,5 (5) D-dimmers.
A. 1,2,3
188. Skin ulcerations localized on B. 1,4,5
the lateral aspect of the ankle are C. 1,3,5
more likely to be related to: prior D. 2, 4,5
trauma (1), perforating veins valve E. 2,3,4
failure (2), pure venous
insufficiency (3), basal cell 192. What kind of prolonged
carcinoma (4), chronic arterial activities are not allowed for
insufficiency (5). patients with symptoms of venous
A. 1,2,4 insufficiency and intact
B. 1,4,5 functioning calf muscle pump.
C. 1,3,5 A. Walking
D. 2,3,5 B. Running
E. 2,3,4 C. Bicycling
D. Swimming
189. Nonhealing ulcers on the E. Standing
medial part of the ankle are most
likely due to underlying: 193. Patients with muscle pump failure
A. venous stasis are not allowed for such prolonged
B. previous trauma activities like:
C. arterial insufficiency A. Walking
D. basal cell carcinoma B. Running
E. allergic reactions C. Bicycling
D. Standing
190. What is the most sensitive and E. Not allowed everything above
specific test for the assessment of
deep and superficial venous 194. Name the contraindication for
disease in the lower extremity and compression stockings wearing:
pelvis, areas not accessible with A. arterial insufficiency
other modalities. B. deep venous insufficiency
A. Magnetic resonance C. hypocoagulation states
venography (MRV) D. bleeding varicosity in past
B. Doppler ultrasound E. varicose veins
C. Duplex ultrasound
D. Trendelenburg test
30
195. What are the contraindications A. Endoscopy
for skin graft placement in patients B. Contrast esophagogram
with ulceration due to venous C. Manometry
insufficiency: (1) Uncorrected leg D. 24-hour pH-monitoring
venous hypertension, (2) E. Endoscopic US
Untreated arteriovenous
malformation, (3) Congenital 200. A 69-year-old man is admitted to
varicose veins, (4) Purulent ulcer the emergency department with an
bottom, (5) Abnormal protein C & acute UGI hemorrhage following a
S levels. bout of repeated vomiting.
A. 1,2,4 Fiberoptic gastoscopy reveals three
B. 1,4,5 linear mucosal tears at the GE
C. 1,3,5 junction. What is the diagnosis?
D. 2,3,5 A. Reflux esophagitis with
E. 2,3,4 ulceration
B. Barrett’s esophagus
196. The most frequent cause of C. Carcinoma of the esophagus
descending necrotizing D. Mallory-Weiss tear
mediastenitis is: E. Scleroderma
A. Odontogenic infection
B. Retropharyngeal abscesses 201. A 79-year-old retired opera singer
C. Iatrogenic pharyngeal injuries presents with dysphagia, which has
D. Cervical lymphadenitis become progressively worse during
E. Parotitis the last 5 years. He states that he is
sometimes aware of a lump on the
197. Name the organism which didn’t left side of his neck and that he
cause descending necrotizing hears gurgling sounds during
mediastenitis: swallowing. He sometimes
A. Staphylococcus regurgitates food during eating.
B. β-hemolytic Streptococcus What is the likely diagnosis?
C. Pseudomonas A. Carcinoma of the esophagus
D. Bacteroides B. Foreign body in the esophagus
E. E.coli C. Plummer-Vinson (Kelly-
Patteson) syndrome
198. What type of surgery is D. Zenker’s (pharyngoesophageal)
complicated most oftenly by diverticulum
postoperative mediastenitis? E. Scleroderma
A. Endocrine
B. Pulmonary 202. A symptomatic patient has a
C. Cardiac barium swallow that reveals a 3-cm
D. Esophageal Zenker’s diverticulum. The next
E. Vascular step in management is?
A. H2 blockers
199. The first step diagnostic test for B. Anticholinergic drugs
esophageal diseases is: C. Elemental diet
31
D. Bougienage
E. Surgery (cricopharyngeal 206. A 54-year-old clerk complains of
myotomy and diverticulectomy) having had dysphagia for 15 years.
The clinical diagnosis of achalasia
203. The gold standart test for making is confirmed by a barium study.
diagnosis achalasia is? What is TRUE in this condition?
A. Endoscopy A. The most common symptom is
B. Contrast esophagogram dysphagia
C. Manometry B. The dysphagia is not typical
D. 24-hour pH-monitoring C. The incidence of sarcoma is
E. Endoscopic US increased
D. Recurrent pulmonary infections
204. A 38-year-old man develops are rare
increasing dysphagia for solid food E. Endoscopic dilatation should be
over many months. What is the avoided
most likely cause of his clinical
presentation? 207. Name the most common
A. Carcinoma of the esophagus condition/disease of esophagus:
B. Achalasia A. Achalasia
C. Sliding hiatal hernia B. Diverticulum
D. Paraesophageal hernia C. Foreign bodies
E. Esophageal diverticulum D. Mallory-Weiss syndrome
E. Tumors
205. A 53-year-old moderately obese
woman presents with heartburn 208. The esophagus is a muscular tube
aggravated mainly by eating and lined with nonkeratinizing
lying down in the horizontal squamous epithelium that starts as a
position. Her symptoms are continuation of the pharynx and
suggestive of gastroesophangeal ends as the cardia of the stomach.
reflux disease (GERD). Which of Which of the following is NOT
the following statements is TRUE? TRUE concerning anatomy of
A. It is best diagnosed by an esophagus?
anteroposterior (AP) and lateral film A. Significant mobility is a normal
of the chest esophageal characteristics and
B. It may be alleviated by certain pathological states can easily
drugs, especially theophylline, displace the esophagus;
diazepam, and calcium channel B. The esophagus is fixed only at its
blockers upper and lower ends, the upper
C. It is not relieved by cessation of end being firmly attached to the
smoking cricoid cartilage and the lower
D. If it is associated with dysphagia, end to the diaphragm;
it suggest a stricture or motility C. The esophagus passes through
disorder three compartments: neck,
E. It should be immediately treated thorax and abdomen;
with surgery
32
D. Ingested foreign bodies tend to A. 1,2,3
lodge at the narrowest area - B. 1,2,5
gastroesophageal sphincter; C. 1,3,4
E. The lower third of esophageus D. 2,4,5
contains voluntary muscles. E. 1,2,4

209. In the adult male length of 212. Contrast esophagogram do NOT


esophagus is from 22 to 28 cm and reveal (choose one option):
averages 2 cm shorter in the female A. Diverticula
Which statement, concerning B. Narrowing or strictures
esophageal size is wrong? C. Achalasia
A. The thoracic part of the D. Diaphragmatic hernias
esophagus is approximately 20 E. Gastroesophageal reflux disease
cm long;
B. The abdominal portion of the 213. Esophageal manometry is
esophagus is approximately 2 cm essential to confirm the diagnosis of
in length; primary esophageal motility
C. The anatomical length of the disorders: 1. Achalasia; 2. Diffuse
esophagus is the distance from esophageal spasm; 3. After-
the cricoid cartilage to the gastric burnings esophageal strictures; 4.
orifice; Hypertensive lower esophageal
D. The length of the esophagus, sphincter; 5. Gastroesophageal
according to endoscopical reflux disease.
measure, is the distance from the A. 1,2,3
incisors to the gastric inlet; B. 1,3,5
E. Esophageal length doesn’t varies C. 1,2,4
with individual height. D. 2,4,5
E. 3,4,5
210. The esophagus receives blood
from: 1. Superior thyroidal artery, 2. 214. In cases of ACHALASIA, the
Both left and right pulmonary arteries, majority of patients presenting between
3. Bronchial arteries, 4. Middle the ages:
mediastinal artery, 5.Left gastric artery A. 20-40 years
A. 1,2,3 B. 10-20 years
B. 1,3,5 C. 40-60 years
C. 2,3,4 D. More than 60 years
D. 2,4,5 E. No age predisposition
E. 1,4,5
215. A 36-year-old male patient, with a
211. Nowadays assessment of 2 years history of dysphagia,
esophageal function and structure underwent barium swallow, which
includes such examinations: 1. reveals typical bird’s-beak
Contrast esophagogram; 2. deformity in the distal esophagus
Endoscopy; 3. Manometry; 4. with more proximal esophageal
Irrigography; 5. Aortography.
33
dilatation. What is the most likely epiphrenic diverticulum; 4.
diagnosis? Underlying achalasia 5. Failed
A. Achalasia esophageal peristalsis.
B. Foreign body of the esophagus A. 1,2,3
C. Hiatal esophageal hernia B. 1,3,4
D. Diverticula C. 1,2,5
E. Distal esophagitis D. 2,4,5
E. 3,4,5
216. The goal of medical treatment of
achalasia is to relax smooth 220. Esophageal manifestations of
muscles. Which of the following GERD includes: 1. Heartburn; 2.
drugs are useless in case of Chest pain; 3. Dysphagia; 4. Weight
achalasia? loss, 5. Bronchospasm.
A. Calcium channel blockers A. 1,2,3
B. Opioids B. 1,3,4
C. Nitrates C. 1,2,5
D. Anticholinergics D. 2,4,5
E. H2 blockers E. 3,4,5

217. According to location esophageal 221. Extraesophageal manifestations of


diverticula are classified as: 1. GERD include all of the following
Retropericardiac, 2. EXCEPT:
Pharyngoesophageal, 3. A. Chronic cough
Retrosternal, 4. Midesophageal, 5. B. Laryngitis
Epiphrenic. Choose the correct C. Dental damage
combination. D. Regurgitation of undigested food
A. 1,2,3 E. Chronic sinusitis
B. 1,3,5
C. 1,2,4 222. The most common cause of
D. 2,4,5 esophageal injury or perforation is:
E. 3,4,5 A. Instrumental procedures
B. Penetrating chest trauma,
218. What symptom is NOT typical for especially height fall
pharyngoesophageal diverticulum C. Chest tube placement for tension
A. Dysphagia pneumothorax
B. Regurgitation of undigested food D. Caustic poisoning
C. Enlarged cervical lymph nodes E. Foreign body ingestion
D. Frequent aspiration
E. Voice changes 223. “Pseudoachalasia”, caused by a
mediastinal tumor, may be
219. Gastroesophageal reflux (GERD) distinguished from primary
is a mechanical disorder that is achalasia using?
caused by: 1. Defective lower A. Esophageal manometry
esophageal sphincter; 2. Gastric B. Esophagogram
emptying disorder; 3. Underlying C. Upper GI endoscopy
34
D. Physical examination
E. Auscultation during meal 227. Which of the following,
concerning gastroesophageal reflux
224. Which statement, concerning (GER) is WRONG?
esophageal diverticula, is WRONG? A. GER is an extremely common
A. Midesophageal diverticula are condition among esophageal
rare and most commonly pathology
associated with mediastinal B. Lower esophageal sphincter
granulomatous disease dysfunction is the most common
(histoplasmosis or tuberculosis) cause of GER
B. A midesophageal diverticulum is C. H2-receptor antagonists are more
typically asymptomatic and effective than PPIs in case of GER
diagnosed incidentally D. Medical therapy is the first line
C. Epiphrenic diverticulum most of management for GER
commonly associated with E. Extraesophageal manifestations
esophageal motor abnormalities of GER are generally pulmonary
(achalasia, hypertensive LES)
D. Epiphrenic diverticulum 228. The initial step in therapy for
typically occurs within the distal patients with mild and intermittent
10 cm of the esophagus symptoms of gastroesophageal
E. Midesophageal diverticula are reflux (GER) is:
common conditions A. Lifestyle modifications
B. Nissen procedure
225. All following conditions may C. Exclude fats from meal
cause descending necrotizing D. α-adreno blockers
mediastinitis except: E. BoTox injection
A. Retropharyngeal abscesses
B. Acute peritonitis 229. Assessment of thyroid gland
C. Iatrogenic pharyngeal injuries functional condition is based on the
D. Infections of the mandibular level of:
molars A. Thyroglobulin
E. Cervical lymphadenitis B. Liver function tests
C. Hormones of hypophysis-adrenal
226. Clinical findings suspicious for system
descending necrotizing mediastinitis D. Parathyroid hormone
include following: 1. recent stomach E. Thyroid-stimulating hormone
operation; 2. fever, 3. localized (TSH), thyroxine (T4) and
cervical or oropharyngeal pain, 4. triiodothyronine (T3)
purulent sputum, 5. respiratory
distress. 230. Extent of surgical intervention on
A. 1,2,3 thyroid gland does NOT depend
B. 1,3,4 from:
C. 2,3,5 A. Thyroid tissue malignant
D. 2,4,5 transformation
E. 1,4,5
35
B. Hormones level of hypophysis- A. Thyreoidectomy
thyroid system B. Subtotal thyreoidectomy
C. Hormones level of hypophysis- C. Hemithyroidectomy with
adrenal system pretracheal lymphatic node
D. Character of process in thyroid biopsy
gland D. Hemithyroidectomy
E. Nodule localization in thyroid E. Lymphadenectomy
gland
236. Choose the best method of
231. “Cold” nodule on thyroid nodular toxic goiter treatment:
scinigram is typical for: A. Methimazole treatment
A. Autoimmune thyroiditis B. Radioactive iodine treatment
B. Grave’s disease C. Surgical treatment
C. Toxic nodular goiter D. Methimazole + glucocorticoids
D. Thyroid cancer E. Methimazole + β-adrenoblockers
E. Thyrotoxic adenoma
237. Extent of operation on thyroid
232. Inferior thyroid arteries are gland in patients with multinodular
branches of: goiter is:
A. a. catotis communis A. Thyreoidectomy
B. a. thyroidea communis B. Hemithyroidectomy
C. aortic arch C. Subtotal thyreoidectomy
D. tr.thyrocervicalis D. Enucleation of nodes
E. a. subclavia E. Isthmectomy

233. Superior thyroid artery is the 238. Thyrocalcitonin is produced by:


branch of: A. Thyroid A-cells
A. a. catotis communis B. Thyroid B-cells
B. aorta C. Thyroid C-cells
C. a. subclavia D. Thyroid D-cells
D. tr.thyrocervicalis E. Thyroid A, B-cells
E. a. carotis externa
239. In 6 hours after
234. Effeciency of levothyroxine hemithyroidectomy the wound
therapy after thyroidectomy is hematoma arised and is increasing.
controlled by: What is your further action?
A. T3 level A. Continue hemostatic therapy
B. T4 level B. Surgical exploration, ligation of
C. T4 excretion with urine the bleeding vessel and
D. Thyrotropin-releasing hormone removing of hematoma
concentration C. Cold compress on wound area
E. Thyroid-stimulating hormone D. Antibiotic therapy
E. Hot compress on neck area
235. Standard operation for the
thyrotoxic adenoma is:
36
240. Preparing the patient with 245. Ultrasonographic sign of thyroid
thyrotoxic adenoma for surgical cyst is:
intervention we do NOT use: A. Size of the nodule – 1 cm
A. Thyrostatic drugs B. Fluid inside the nodule
B. β-blockers C. Peripheral vascularization of the
C. Radioactive iodine nodule
D. Sedative drugs D. Unclear circuit of the nodule
E. Steroid hormones E. Multiple nodules

241. Ultrasonographic sign of 246. On second day after


malignant thyroid nodule is: hemithyroidectomy the deep wound
A. Size of the nodule less then 1 cm hematoma arised, but does not
B. Fluid in the central area of the increase in size. What is your
nodule further action?
C. Peripheral vascularization of the A. No further intervention
nodule B. Surgical exploration, removing
D. Unclear circuit of the nodule wound hematoma and ligation of
E. Multiple nodules the bleeding vessel
C. Cold compress on wound area,
242. Thyrotoxic adenoma generally antibiotic therapy, hemostatic
produces: drugs
A. Triiodothyronine D. Antibiotic therapy, aspirin
B. Thyroxin E. Hot compress on neck area
C. Thyrocalcitonin
D. Thyrotropin-releasing hormone 247. In patient with suspicion for
E. Thyroid-stimulating hormone postoperative hypothyroidism first
of all we should check:
243. The best surgical procedure for A. Iodine excretion with urine
Grave’s disease is: B. Radioiodine absorption
A. Resection of thyroid gland C. T4
B. Hemithyroidectomy B. T3
C. Subtotal thyroid resection E. TSH
D. Isthmectomy
E. Total thyroidectomy 248. Levothyroxine therapy after
thyroidectomy in patient with
244. Ultrasonographic sign of colloid multinodular goiter is called:
nodule is: A. Suppressive
A. Size of the nodule – 1 cm B. Replacing
B. Fluid in the central area of the C. Supportive
nodule D. Stimulating
C. Peripheral vascularization of the E. Adjuvant
nodule
D. Unclear circuit of the nodule 249. For last three months young
E. Multiple nodules woman lost 8 kg of weight. She
complains of heartbeats, neck
37
thickening, sensation of foreign 252. 28-year-old male, has been
object during the swallowing, operated because of thyrotoxic
fingers tremor, exophthalmos, low goiter. In 12 hours after operation
grade body fever. On palpation body temperature increased to 39°C,
diffusely enlarged thyroid gland. Ps – 160/min., BP – 180/110
The most likely diagnosis is? mm.Hg. What complication has
A. Grave’s disease been developed?
B. Hysteria A. Hypoparathyroidism
C. Brain tumour B. Severe arterial hypertension
D. Thyrotoxic adenoma C. Laryngeal recurrent nerve injury
E. Endemic goiter (diffuse) D. Thyrotoxic crisis
E. Pneumonia
250. 42-year-old woman for the FIRST
time was diagnosed severe 253. Patient M., 28-year-old male, has
thyrotoxicosis caused by Grave’s been operated because of thyrotoxic
disease. Choose the right sequence goiter. On 2 day after operation the
of treatment? patient`s condition worsen:
A. Urgent operation convulsions of hands, legs, face
B. Medical therapy, then resection muscels, Chvostek and Trousseau
of thyroid gland signs are positive. Patient complains
C. Medical therapy, then of chest pain. ECG: prolonged QT
endocrinologist supervision interval. What complication has
D. Medical therapy been developed?
E. Medical therapy, then perform A. Hyperparathyroidism
subtotal resection of thyroid B. Hypoparathyroidism
gland C. Thyrotoxic crisis
D. Laryngeal recurrent nerve injury
251. 46-year-old patient, has been E. Thyrotoxic myocardiodystrophy
operated 12 years ago for diffuse
thyrotoxic goiter. Now patient was 254. A 26-year-old woman was
diagnosed the recurency of diagnosed: diffuse thyrotoxic goiter.
thyrotoxicosis. One more operation The conservative treatment under
is offered. It is necessary to find out endocrinologist supervision was not
where the thyroid tissue is located, effective during 12 month. She
US examination was not effective. agreed for surgical intervention.
What diagnostic method we must What measures of preoperative
use? preparation should be performed for
A. Fine needle aspiration biopsy of prevention of thyrotoxic crisis in
the thyroid gland postoperative period?
B. CT A. Use of antithyroid agents to
C. Thyroid radioactive iodine achieve euthyroid condition
scintigraphy B. Minimally invasive surgery
D. X-ray examination of neck C. Bed rest
E. Contrast esophagogram D. Detoxication therapy
E. Application of a-blockers
38
D. Total thyroidectomy
255. Patient K., 29-year-old male, has E. NSAIDs
been diagnosed: diffuse thyrotoxic
goiter. The conservative treatment 258. What laboratory test confirms the
under endocrinologist supervision diagnosis of Graves disease?
for last 3 years. Recently the tight A. free T3
nodule appeared in the right lobe of B. TSH-receptor’s antibodies
thyroid gland, which is increasing in C. Thyroglobulin antibodies
size. What diagnostic method will D. TSH
help to exclude thyroid cancer E. Thyroperoxidase antibodies
preoperatively?
A. US 259. Levothyroxine therapy after
B. Intraoperative lymphatic node thyroidectomy in patient with
biopsy cancer of thyroid gland is called:
C. X-ray examination of the neck A. Suppressive
D. Thyroid radioactive iodine B. Replacing
scintigraphy C. Supportive
E. Fine needle aspiration biopsy D. Stimulating
E. Adjuvant
256. A 36-year-old patient complains
of permanent hoarseness of voice 260. * A patient of 32 y.o. complains
and symptoms of respiratory of severe weakness, tremor of
insufficiency after physical extremities. Objective examination:
exercices. These complaints body weight loss, wet and warm
appeared 2 years ago, after total skin. The thyroid gland is enlarged
thyroidectomy. Name the up to the 2-nd degree, painless,
complication which develops in this elastic. Ps – 108 bpm, BP – 160/55
case: mm Hg. There are no other
A. Larynx cancer abnormalties. The diagnosis is:
B. Laryngeal recurrent nerve injury A. Chronic autoimmune thyroiditis,
C. Chronic laryngitis hypertrophic type
D. Laryngophthysis B. Toxiferous adenoma of the
E. Scleroma of vocal cords thyroid gland
C. Diffuse toxic goiter of the 2-nd
257. A 40-year-old woman presents degree, thyrotoxicosis of the
with weight loss, palpitations, and average degree
exopthalmos. On physical D. Diffuse euthyroid goiter of the 2-
examination, the thyroid gland is nd degree
diffusely enlarged. Blood tests E. Chronic fibrous thyroiditis
reveal hyperthyroidism. Which
medication is not used for the 261. * A 63 y.o. patient was operated
treatment of thyrotoxicosis? on account of big multinodular
A. Methimazole euthyroid goiter. Despite of techical
B. Lugols iodine difficulties a forced subtotal
C. I131 resection of both parts of the thyroid
39
gland was performed. On the 4-th A. human body reaction to own
day after the operation the woman thyroid proteins
had cramps of face muscles and B. human body reaction to foreign
upper extremities, stomach ache. proteins which get to organism
Positive Chvostek's and Trousseau's from environment
signs. What is the most probable C. influence of light irradiation to
cause of such condition? human body
A. Insufficiency of parathyroid D. increased blood level of
glands thyrostimulating hormone
B. Thyrotoxic crisis E. increased blood level of
C. Injury of recurrent nerve thyrotropic hormone
D. Postoperative hypothyroidism
E. Tracheomalacia 265. *A 50 year old woman with a 2-
year history of mild, diffuse, tender
262. * Examination of a 26 year old thyroid enlargement complains of
female patient revealed a node in 10 pound weight gain and fatigu E.
the right lobe of thyroid gland. The What is the most probable
node appeared no earlier than 3 diagnosis?
months ago. The patient associates A. Riedel's thyroiditis
this node with stress. She doesn't B. Subacute thyroiditis
complain either about pain or C. Papillary thyroid carcinoma
enlargement of the node. Ultrasonic D. Suppurative thyroiditis
scanning revealed a 2x2,5 cm large E. Hashimoto's thyroiditis
node in the inferior part of the right
lobe of thyroid gland. What 266. *Examination of a 12 year old
treatment should be administered? child revealed diffuse thyroid
A. Dynamic observation enlargement of the II degree. Heart
B. No need for treatment auscultation revealed dullness of
C. Radioactive iodine heart sounds, heart rate was 64/min.
D. Conservative therapy The child has frequent
E. Surgical intervention constipations, anemia.
Concentration of thyreoglobulin
263. What arrythmia most oftenly antibodies is increased. What
develops in patients with toxic disease might have caused such
goiter? symptoms?
A. Atrial fibrillation A. Thyroid carcinoma
B. Ventricle fibrillation B. Autoimmune thyroiditis
C. AV block C. Thyroid hyperplasia
D. Extrasystole D. Endemic goiter
E. Thyroid hormones doesn’t cause E. Diffuse toxic goiter
any heart problems
267. Thyroiditis includes the following
264. What is the most probable reason inflammatory diseases of the
of autoimmune thyroid thyroid gland: 1. Acute suppurative
inflammation initiation? thyroiditis; 2. Subacute thyroiditis;
40
3. Chronic thyroiditis; 4. D. Dysphagia
Hashimoto's thyroiditis; 5. Riedel E. With everything
thyroiditis
A. 1, 2, 3 272. What is the main complication of
B. 1, 3, 4 chronic autoimmune thyroiditis?
C. 1, 2, 4 A. Hypothyroidism
D. 2, 3, 4 B. Hyperthyroidism
E. All are correct C. Hypoparathyroidism
D. Abscess formation
268. What medicine may cause E. Descending necrotizing
secondary thyroiditis? mediastinitis
A. Amiodarone
B. Cefuroxime 273. Which statement, concerning
C. Potassium iodine acute thyroiditis is NOT TRUE?
D. Digoxin A. The usual microorganism
E. All can cause responsible for acute thyroiditis
is Staphylococcus aureus
269. The history of acute thyroiditis B. Patients with acute thyroiditis
include all of the following, generally maintain normal
EXCEPT: thyroid function
A. Fever C. Radioactive iodine thyroid
B. Neck pain scanning is useless in patients
C. Hoarseness with acute thyroiditis
D. Dysphagia D. Thyroid ultrasonography is
E. Viral infection useful in patients with acute
thyroiditis
270. The history of subacute thyroiditis E. In acute thyroiditis, if diagnosis
include: 1. Symptoms of is confirmed on the stage of
hyperthyroidism; 2. Symptoms of infiltration, immediate surgery is
hypothyroidism; 3. Previous viral required
infection; 4. Previous bacterial
infection; 5. Low grade fever; 6. 274. Which statement, concerning
High grade fever subacute thyroiditis, is NOT
A. 1,3,5 TRUE?
B. 1,3,6 A. Subacute thyroiditis is generally
C. 2,4,6 thought to be due to viral
D. 2,3,6 infection
E. 1,4,5 B. C-reactive protein levels are
usually elevated in subacute
271. Chronic autoimmune thyroiditis is thyroiditis
associated with which of the C. Treatmant of subacute thyroiditis
following? is symptomatic
A. High fever D. The most common symptom, in
B. Previous bacterial infection case of subacute thyroiditis , is
C. Thyroid gland enlargement fever
41
E. Signs of hyperthyroidism may be 278. The main effects of parathyroid
present hormone are to increase the
concentration of plasma calcium by
275. Which statement, concerning (choose the WRONG answer):
chronic autoimmune thyroiditis, is A. Increasing the release of
NOT TRUE? calcium and phosphate from
A. Chronic thyroiditis is associated bone matrix
with leukocytosis with a shift to B. Increasing calcium reabsorption
the left and an increased ESR by the kidney
B. Prevalence of autoimmune C. Increasing calcitonin
thyroiditis in adults has a 90%- concentration
female predominance D. Increasing renal production of
C. Patients with autoimmune 1,25-dihydroxyvitamin D-3
thyroiditis frequently develop (calcitriol)
hypothyroidism E. Increasing intestinal absorption
D. Autoimmune thyroiditis is also of calcium
frequently part of the
polyglandular autoimmune 279. Name the most often cause of
syndromes primary hyperparathyroidism:
E. Treatment for chronic A. Single parathyroid adenoma
autoimmune thyroiditis may B. Multiple parathyroid adenoma
require replacement therapy with C. Parathyroid glands hyperplasia
Levothiroxin D. Parathyroid carcinoma
E. Medullary thyroid carcinoma
276. Name hormones and vitamin
which maintain calcium plasma 280. Primary hyperparathyroidism is
homeostasis: 1. Parathyroid the unregulated overproduction of
hormone; 2. Gastrin; 3. Calcitonin; parathyroid hormone resulting in:
4. Vitamin K; 5. Vitamin D3. A. Hypocalcemia
A. 1, 2, 3 B. Hypercalcemia
B. 1, 3, 5 C. Hypokalemia
C. 1, 3, 4 D. Hyperkalemia
D. 1, 2, 5 E. Vitamin D deficiency
E. 2, 3, 5
281. A 22-year-old patient is scheduled
277. Usually, parathyroid glands are to under go parathyroidectomy for
situated posterior to the thyroid hyperparathryoidism associated
gland. How many parathyroid with familial multiglandular
glands usualy have human? syndrome. His sister developed
A. 1 peptic ulcer disease secondary to a
B. 2 Zollinger-Ellison
C. 3 (hypergastrinemia) tumor of the
D. 4 pancreas. On examination, a
E. 5 swelling was noted over the
42
posterior aspect of the patient’s fifth 284. Which one of the following is
rib. What is the most likely finding? not part of the management of a
A. Metastasis from a parathyroid patient with hyperparathyroidism
carcinoma A. Hydration with intravenous
B. Osteitis fibrosa cystica (brown normal saline
tumor) and subperiosteal B. Steroids
resorption of the phalanges C. Exploration of the neck for
C. Dermoid cyst parathyroidectomy
D. Eosinophilic granuloma D. Parathyroid scan
E. Chondroma E. Vitamin D

282. The clinical syndrome of primary 285. Each adrenal gland is supplied
hyperparathyroidism is presented by by small arterial branches that
(choose the WRONG answer): originate from three distinct
A. Osteopenia -> osteoporosis sources. Name these sources: 1.
B. Renal stones Inferior phrenic artery; 2. Celiac
C. Peptic ulcer trunk; 3. Aorta; 4. Renal artery; 5.
D. Colon polips Common hepatic artery.
E. Acute pancreatitis A. 1, 2, 3
B. 2, 3, 4
283. A 56-year-old women complaints C. 3, 4, 5
for bone and joint pain, nausea, D. 1, 3, 4
vomiting, abdominal pain, E. 1, 4, 5
constipation, weakness and easy
fatigability, depression, inability to 286. Which statement is WRONG
concentrate, and memory problems. concerning the venous and
In past medical history she has lymphatic drainage of adrenal
suffered from renal stones in both gland?
kidneys, peptic ulcer disease. What A. The right adrenal gland usually
changes in the biochemical analysis drains by one short vein, which
of blood will diagnose primary empties directly into the vena
hyperparathyroidism? 1. Elevated cava
ionized calcium; 2. Hypercalcemia; B. Accessory adrenal veins are not
3. Elevated intact parathyroid infrequently present
hormone level; 4. C. The right adrenal vein drains into
Hypophosphatemia; 5. Elevated 24- the right renal vein
hour urine calcium. D. Lymphatic drainage from the
A. 1, 2, 3 adrenal glands drains into
B. 2, 3, 4 periaortic and paracaval nodes
C. 1, 3, 5 E. The left adrenal vein drains into
D. 1, 2, 4 the left renal vein
E. All changes diagnose primary
hyperparathyroidism 287. The adrenal gland is composed
of two distinct organs, the adrenal
cortex and the adrenal medulla.
43
Choose the WRONG statement pheochromocytoma. Which
about adrenal physiology. statement is WRONG concerning
A. The cortex is divided into three adrenal tumor biopsy?
functional zones: the outer A. Percutaneous biopsy of the
glomerulosa, the intermediate adrenal gland can be performed
fasciculata, and the inner reticularis. under either CT or ultrasound
B. Zona glomerulosa produce guidance
mineralocorticoids B. Biopsy should be performed in
C. Zona fasciculata produce all cases of adrenal masses
glucocorticoids C. The most common indication is
D. Zona reticularis produce sex suspicion of metastatic disease to
steroids the adrenal gland
E. Only mineralocorticoids are D. A percutaneous biopsy cannot
absolutely required for life reliably distinguish between an
adrenal adenoma and an adrenal
288. The adrenal medulla is carcinoma
embryologically analogous to a E. Biopsy should never be
peripheral sympathetic ganglia performed in a patient until a
and consists of: biochemical workup has been
A. Chromaffin tissue completed to rule out a
B. Connective tissue pheochromocytoma
C. Chromogranin tissue
D. Muscle tissue 292. A 26-year-old patient was
E. Epithelial tissue performed CT scan to rule out
pathology of pancreas. On CT
289. The adrenal medulla produces: scans 6 cm tumor of left adrenal
A. Estrogen gland was localized. What
B. Aldosteron questions should be answered for
C. Cortisol appropriate management of
D. Epinephrine incidentaloma: 1. Is it functional?;
E. Androsteron 2. Is it likely to be a malignant
adrenal tumor? 3. Is it metastatic?
290. Choose examinations which are A. 1, 2
used for localization of adrenal B. 1, 3
masses: 1. Ultrasound; 2. CT scan; C. 2, 3
3. MRI; 4. Radioisotope scan; 5. D. All of them should be answered
Intraoperative ultrasound. E. There is no need to answer to any
A. 1, 2, 3 question
B. 1, 2, 4
C. 1, 2, 5 293. A 26-year-old patient was
D. 1, 3, 4 performed CT scan to rule out
E. All are used pathology of pancreas. On CT
scans 6 cm tumor of left adrenal
291. Sudden death has been reported gland was localized. What size of
following biopsy of unsuspected
44
incidentaloma is an indication for urination. Objectively:
surgical treatment? hyperpigmentation of skin, gums,
A. 0,5 cm internal surface of cheeks. BP –
B. 1,0 cm 90/58 mm Hg. Blood count: RBC-
C. 2,0 cm 3,1*1012/L, Hb - 95 g/L, C.I.- 0,92;
D.3,0 cm leukocytes - 9,4*109/L, eosinophils
E. 4,0 cm - 7, segmentonuclear leukocytes -
45, stab neutrophils - 1,
294. *A 49 y.o. female patient was lymphocytes - 40, monocytes - 7,
admitted to the hospital with acute Na+- 115 mmol/L, К+- 7,3 mmol/L.
attacks of headache accompanied by What is the preliminary diagnosis?
pulsation in temples, BP rised up to A. Congenital adrenocortical
280/140 mm Hg. hyperplasia
Pheochromocytoma is suspected. B. Primary hyperaldosteronism
What mechanism of hypertensive C. Pheochromocytoma
atack does this patient have? D. Primary adrenocortical
A. Increasing of thyroxine excretion insufficiency
B. Increasing of catecholamines E. Diabetes insipidus
concentration
C. Increasing of plasma renin 297. *A 38 y.o. woman suffers from
activity paroxysmal BP rises up to 240/120
D. Increasing of aldosterone level in mm Hg accompanied by nausea,
blood vomiting, tachycardia, excessive
E. Increasing of vasopressin sweating. During the onset blood is
excretion hyperglycemic. After the onset there
is voluminous urination. Kidneys
295. *A 40 y.o. patient was diagnosed: sonography revealed accessory
1. Medular thyroid gland cancer. 2. mass bordering upon the upper pole
Pheochromocytoma. What of the right kidney, presumably it
operation should be performed at belongs to the adrenal gland. What
first? laboratory test will allow to make a
A. Subtotal resection of thyroid more precise diagnosis?
gland and fascicular resection of A. Estimation of thyroxin and
limphatic nodes thyrotropic hormon in blood
B. Krail's operation B. Estimation of insulin and C-
C. Operation on account of peptide content in blood
pheochromocytoma C. Estimation of catecholamine and
D. Vanach's operation vanillylmandelic acid excretion with
E. Operation on thyroid gland urine
D. Estimation of renin content in
296. *A 34 y.o. patient has been blood
suffering from pulmonary E. Estimation of glomerular
tuberculosis for 7 years; he filtration rate
complains of muscle weakness,
weight loss, diarrhea, frequent
45
298. A 57-year-old woman presents E. Abdominal ultrasonogram
with vague abdominal pain. After a
course of treatment with H2- 301. A 30-year-old primigravida
blockers failed and abdominal complains of headaches,
ultrasound was negative, she restlessness, sweating, and
underwent a CAT scan of the tachycardia. She is 18 wk pregnant
abdomen. The scan was negative and her blood pressure is 200/120
except for the presence of a 3-cm mm Hg. Pheochromocytoma was
mass in the left adrenal gland. Her suspected. Appropriate treatment
pain disappeared. Urine and serum might consist of
biochemical studies for a A. Therapeutic abortion
functioning adrenal tumor are B. Urgent excision of the tumor and
negative. Her past medical history is a therapeutic abortion
negative. The next step should be: C. Phenoxybenzamine and
A. Adrenalectomy propranolol followed by a
B. CT-guided percutaneous core combined cesarean section and
needle biopsy excision of the tumor
C. Arteriography D. Propranolol blockade followed
D. MRI by a combined cesarean section
E. Repeat CAT scan in 3 months and excision of the tumor
E. Phenoxybenzamine and
299. A 40 y.o. patient was diagnosed: propranolol followed by a
1. Medular thyroid gland cancer. 2. combined vaginal delivery at
Pheochromocytoma. Make the term and excision of the tumor
diagnosis?
A. MEN-1 302. Which of the following statements
B. MEN-2 concerning Cushing syndrome
C. Zolliger-Ellison syndrome secondary to adrenal adenoma is
D. Watery diarrhea, hypokalemia, true?
and achlorhydria (WDHA) A. Adrenal adenomas cause 80% of
syndrome all cases of Cushing syndrome
E. Werner’s syndrome B. CT scan is generally
unsuccessful in lateralizing the
300. A 30-year-old primigravida tumors preoperatively
complains of headaches, C. Exploration of both adrenal
restlessness, sweating, and glands is indicated
tachycardia. She is 18 wk pregnant D. For uncomplicated small tumors,
and her blood pressure is 200/120 an open transperitoneal surgical
mm Hg. Pheochromocytoma was approach is usually employed
suspected. What examination E. Postoperative corticoid therapy is
should be performed? required to prevent
A. Exploratory laparotomy hypoadrenalism
B. Biopsy of adrenal mass
C. Radioisotope scan 303. Primary aldosteronism is defined
D. Abdominal CT scan as excessive secretion of
46
aldosterone. Choose WRONG
statement concerning primary 306. Patients with endogenous
hyperaldosteronism? Cushing’s syndrome caused by a
A. Aldosteronomas occur in pituitary adenoma will have: 1.
approximately 65% of patients with Elevated cortisol level; 2. Decreased
primary aldosteronism cortisol level; 3. Elevated ACTH; 4.
B. Idiopathic hyperaldosteronism is Decreased ACTH; 5. Hypokalemia.
caused by bilateral adrenal A. 1, 4, 5
hyperplasia B. 1, 3, 5
C. Aldosteronomas are almost C. 2, 3, 4
always unilateral and are often less D. 1, 2, 3
than 2 cm in size E. All are correct
D. Unilateral adrenalectomy in the
setting of Idiopathic 307. Cushing’s syndrome can be
hyperaldosteronism is curative caused by: 1. Adrenocortical
E. Adrenal cortical aldosterone- adenoma; 2. Adrenocortical
producing carcinoma is carcinoma; 3. Pituitary
extremely rare and represents corticotroph adenoma; 4. Ectopic
another surgically treatable form secretion of ACTH by
of primary aldosteronism nonpituitary tumor; 5. Ectopic
secretion of corticotropin-
304. Main clinical signs of primary releasing hormone.
hyperaldosteronism are: 1. Arterial A. 1, 2, 3
hypertension; 2. Arythmia; 3. B. 1, 2, 4
Hypokalemia; 4. Increased C. 2, 4, 5
aldosterone level; 5. Decreased D. 1, 3, 5
plasma renin activity. E. All can cause
A. 1, 2, 3
B. 1, 3, 5 308. Choose the main clinical signs
C. 1, 4, 5 typical for Cushing’s syndrome: 1.
D. 2, 3, 5 Centripetal obesity; 2.
E. All are correct Hypertension; 3. Osteoporosis; 4.
Acne; 5. Polyuria, polydipsia.
305. Patients with endogenous A. 1, 2, 3
Cushing’s syndrome caused by a B. 1, 2, 4
unilateral adrenal tumor will have: C. 2, 4, 5
1. Elevated cortisol level; 2. D. 1, 3, 5
Decreased cortisol level; 3. Elevated E. All are typical
ACTH; 4. Decreased ACTH; 5.
Elevated 24-h urinary free cortisol. 309. For Cushing’s syndrome is typical
A. 1, 4, 5 deposition of adipose tissue in
B. 1, 3, 5 characteristic sites: 1. Face
C. 2, 3, 4 ("moon" facies); 2. Interscapular
D. 1, 2, 3 area ("buffalo hump"); 3.
E. All are correct Mesenteric bed ("truncal"
47
obesity); 4. Lower extremity; 5. E. Breast abscess
Upper extremity.
A. 1, 2, 3 313. A 28-year-old female figure skater
B. 1, 2, 4 presents several weeks after having
C. 2, 4, 5 sustained an injury to her left breast.
D. 1, 3, 5 She has a painful mass in the upper
E. All are typical outer quadrant. Skin retraction is
noticed, and a hard mass, 3–4 cm in
310. A 35-year-old professional dancer diameter, can easily be palpated.
presents with a well-defined, tense, What is the most likely diagnosis?
smooth mass in the upper outer A. Infiltrating carcinoma
quadrant of the left breast. She B. Breast abscess
states that the mass becomes larger C. Hematoma
just before onset of her periods. D. Fat necrosis
Aspiration yields a clear yellow E. Sclerosing adenosis
fluid and the mass disappears. The
most likely diagnosis is: 314. A 35-year-old patient presents to
A. Fibroadenoma in a cyst your office with chronic draining
B Breast cyst subcutaneous periareolar abscesses,
C Carcinoma in a cyst which have been incised and
D. Lipoma drained many times in the past 5
E.Galactocele years but keep recurring. What is
the best treatment of choice?
311. An 23-year-old presents with a A. Repeat incision and drainage (I
wellcircumscribed 2-cm mass in her and D) since the previous
right breast. The mass is painless procedures were inadequate
and has a rubbery consistency and B. Long-term antibiotics
discrete borders. It appears to move C. Major duct excision
freely through the breast tissue. D. Complete excision of the
What is the likeliest diagnosis? drainage tract
A. Carcinoma E. Tell the patient there is nothing
B. Cyst to do and that this will
C. Fibroadenoma eventually resolve with age
D. Cystosarcoma phyllodes
E. Intramammary lymph node 315. A patient presents 1 month after a
benign right breast biopsy with a
312. Galactorrhea, a milky discharge lateral subcutaneous cord felt just
from the nipple in nonpregnant under the skin and causing pain.
women, is most likely to be The etiology of this condition is?
associated with which of the A. Fat necrosis
following? B. Infection
A. Fibroadenoma C. Superficial thrombophlebitis
B. Tubular adenoma D. Suture granuloma
C. Pituitary adenoma E. Misdiagnosed breast cancer
D. Hyperparathyroidism
48
316. A 36-year-old woman complains B. Previous history of benign
of a 3-month history of bloody breast biopsies
discharge from the nipple. At C. Atypia seen on pathology from
examination, a small nodule is previous breast biopsy
found, deep to the areola. Careful D. First-degree relative with history
palpation of the nippleareolar of breast cancer
complex results in blood arrearing E. Increasing age
at the 3 O’clock position.
Mammogram findings are normal. 319. A 46-year-old woman presents
What is the likeliest diagnosis? with a mammogram that shows a 1-
A. Intraductal papilloma cm cluster of fine calcification in
B. Breast cyst the right breast. Following
C. Intraductal carcinoma mammographic wire localization,
D. Carcinoma in situ the lesion is excised and the
E. Fat necrosis pathology reported as ductal
carcinoma in situ (DCIS) with
317. During a routine screening comedo features and free margins.
mammography, a 62-year-old What advice should be given to the
teacher is informed that she has patient?
changes on her mammography, and A. If untreated, about 30% of such
she should consult her physician. lesions become invasive over a
She can be reassured that the 10-year period
findings that indicate a benign B. Comedo DCIS is less aggressive
condition are which of the than noncomedo DCIS
following? C. Bilateral mastectomy and
A. Discrete, stellate mass radiotherapy are the preferred
B. Fine, clustered calcifictions treatments
C. Coarse calcifications D. Axillary node dissection is
D. Solid, clearly defined mass with always indicated
irregular edges E. Total mastectomy carries a high
E. Discrete, nonpalpable mass that (50%) risk of carcinoma
has enlarged when compared recurrence.
with a mass shown on a
mammogram taken 1 year 320. Breast cancer most often occurred
previously in age:
A. 25-30
318. A 40-year-old lawyer comes into B. 30-40
your office after seeing some C. 40-50
information on the Internet relating D. 50-60
to breast cancer. Which of the E. 30-50
following factors has not shown to
increase a woman’s risk for breast 321. A 50-year-old woman underwent
cancer? wide excision of a 2.5-cm
A. Smoking infiltrating ductal carcinoma of the
breast with axillary lymph node
49
dissection followed by radiation and induration of the left mammary
chemotherapy 2 years ago. The gland, body temperature rise up to
patient now complains of RUQ 390С, headache, indisposition.
abdominal pain. CT scan reveals Objectively: fissure of nipple,
two masses in the right lobe of the enlargement of the left mammary
liver. Select the most likely gland, pain on palpation. What
diagnosis. pathology would you think about in
A. Adenoma this case?
B. Focal nodular hyperplasia A. Lactational mastitis
C. Hemangioma B. Lacteal cyst with suppuration
D. Hepatocellular carcinoma C. Breast cancer
E. Metastatic carcinoma D. Fibrous adenoma of the left
mammary gland
322. A 52-year-old undergoes a left E. Phlegmon of mammary gland
modified radical mastectomy for a
2-cm breast cancer. She should be 325. A 40 year old woman has changes
informed that the factor which has of mammary gland. What is the
the greatest impact on her prognosis most often symtomp that precede
is? the malignization?
A. The size of the primary tumor A. Painless movable induration
B. The histological type of the B. Bloody discharges from the
carcinoma nipple
C. The number of axillary nodes C. Pure discharges from the nipple
positive for metastasis D. Painful movable induration
D. Hormonal receptor status of the E. Skin induration with inverted
primary tumor nipple
E. Positive findings on tests for the
presence of the BRCA(breast 326. A parturient complains about pain
cancer)1 gene in the mammary gland. Palpation
revealed a 3х4 cm large infiltration,
323. A young man has painful soft in the centre. Body temperature
indurations in the peripapillary is 38,5oC. What is the most probable
regions of both mammary glands. diagnosis?
The most reasonable action will be: A. Retention of milk
A. To take an aspirate for bacterial B. Birth trauma
inoculation and cytology C. Acute purulent mastitis
B. To cut and drain them D. Pneumonia
C. To administer steroids locally E. Pleuritis
D. To remove them
E. To leave these indurations 327. Blunt chest trauma can be caused
untouched most oftenly by: 1. Motor vehicle
crashe; 2. Stab wound; 3. Height
324. A woman consulted a doctor on falls; 4. Airplane crash; 5. Bullet
the 14-th day after labor about wound:
sudden pain, hyperemy and A. 1, 2, 3
50
B. 2, 3, 4 paradoxically with respiration. At
C. 3, 4, 5 thoracentesis signs of
D. 2, 3, 5 tensionpneumothorax absent. Make
E. 1, 3, 4 the diagnosis?
A. Hemothorax
328. Name the most common thoracic B. Sternal fracture
injury in blunt trauma? C. Flail chest
A. Hemothorax D. Esophageal injury
B. Flail chest E. Aorta rupture
C. Rib fracture
D. Sternal fracture 332. A 70-year-old man is brought into
E. Pericardial tamponade the emergency department
following his injury as a passenger
329. A 45-year-old man skidded from in a car crash. He complains of right
the road at high speed and hit a tree. side chest pain. Physical
Examples of deceleration injuries in examination reveals a respiratory
this patient include: rate of 42 breaths per minute and
A. Aortic valve rupture multiple broken ribs of a segment of
B. Aortic arch rupture the chest wall that moves
C Posterior dislocation of shoulder paradoxically with respiration. At
D. Hemothorax thoracentesis signs of
E. Rib fracture tensionpneumothorax absent. What
should the next step be?
330. Choose life-threatening injuries, A. Tube thoracostomy
which should be identified B. Tracheostomy
immediately: 1. Esophageal rupture; C. Insertion of a nasogastric tube
2. Tension pneumothorax; 3. D. Endotracheal intubation
Massive hemothorax; 4. Cardiac E. Intercostal nerve blocks
tamponade; 5. Pulmonary
contusion. 333. An 18-year-old man is brought to
A. 2, 3, 4 the emergency department with a
B. 1, 2, 3 stab wound just to the right of the
C. 1, 3, 5 sternum in the sixth intercostal
D. 1, 4, 5 space. His blood pressure is 80 mm
E. 2, 3, 5 Hg. Faint heart sounds and pulsus
paradoxus are noted. Auscultation
331. A 70-year-old man is brought into of the right chest reveals markedly
the emergency department decreased breath sounds. The initial
following his injury as a passenger management of this patient should
in a car crash. He complains of right be which of the following?
side chest pain. Physical A. Aspiration of the right chest
examination reveals a respiratory cavity
rate of 42 breaths per minute and B. Analgesics
multiple broken ribs of a segment of C. Echocardiogram
the chest wall that moves D. Pericardial window
51
E. Insertion of central venous access 337. A 26-year-old man is stabbed in
line the right intercostal space in the
midclavicular line and presents to
334. An 18-year-old man presents to the emergency department. On
the emergency department with a examination, subcutaneous
gunshot wound to the left chest in emphysema of the right chest wall,
the anterior axillary line in the sixth absent breath sounds, and a trachea
intercostal space. His blood pressure shifted to the left are noted. What is
is 120/70 mm Hg, pulse – 78 bpm. the most likely diagnosis?
A sucking sound is audible during A. Pneumothorax
inspiration. Immediate management B. Tension pneumothorax
is which of the following? C. Massive hemothorax
A. Exploratory laparotomy D. Hemopneumothorax
B. Exploratory thoracotomy E. Chest wall laceration
C. Pleurocentesis
D. Closure of the hole with sterile 338. A 31-year-old man is shot in the
dressing back of the left chest, and the bullet
E. Insertion of central venous access exits the left anterior chest. The
line patient’s blood pressure is 130/90
mm Hg, respiration rate is 28
335. While landing at the end of flight breaths per minute, and pulse is 110
a young man develops shortness of bpm. A chest x-ray reveals
breath and rightsided pressure chest hemothorax. A chest tube is inserted
pain. He is tall and thin. He has not and yields 800 mL of blood; the
previously consulted a doctor. A first and second hour drainage is
chest film is likely to show? 200 mL/h and 240 mL/h,
A. Left pleural effusion respectively. What is the next step
B. Spontaneous pneumothorax in management?
C. Dilated stomach A. Place a second chest tube
D. Hemothorax B. Collect the blood for
E. Cardiomegaly autotransfusion
C. Transfuse and observe drainage
336. While landing at the end of flight for another hour
a young man develops shortness of D. Insert a Swan-Ganz catheter
breath and rightsided pressure chest E. Perform a left thoracotomy
pain. He is tall and thin. He has not
previously consulted a doctor. The 339. A 31-year-old man is shot in the
treatment is: back of the left chest, and the bullet
A. Insertion of a chest tube exits the left anterior chest. The
B. Immediate cardiology consult patient’s blood pressure is 130/90
C. Thoracentesis mm Hg, respiration rate is 28
D. Insertion of a nasogastric tube breaths per minute, and pulse is 110
E. Thoracoscopy bpm. A chest x-ray reveals
hemothorax. A chest tube is inserted
and yields 800 mL of blood; the
52
first and second hour drainage is 343. Which statement is wrong
200 mL/h and 240 mL/h, concerning sternal fracture?
respectively. In the patient A. Is seen in 60% of patients with
described above the most likely blunt trauma
cause of the bleeding is injury to B. Is very uncommon injury
which of the following? C. Needs large traumatic force
A. Pulmonary artery D. Is caused by direct blow to front
B. Lung parenchyma of the chest
C. Internal thoracic (mammary) E. Is associated with high rate of
and/or intercostals arteries myocardial contusion and cardiac
D. Pulmonary vein tamponade
E. Left atrium
344. A 25-year-old man is shot in the
st
340. Which statement concerning 1 left lateral chest. In the emergency
and 2nd ribs fractures is wrong? department, his blood pressure is
A. Require high force 120/90 mm Hg, pulse rate is 104
B. Frequently have injury to aorta beats per minute (bpm), and
C. Frequently have injury to bronchi respiration rate is 36 breaths per
D. May injure subclavian minute. Chest x-ray shows air and
artery/vein fluid in the left pleural cavity.
E. Causes pulsus paradoxicus Nasogastric aspiration reveals
blood-stained fluid. What is the best
341. Most oftenly fracture of 11th or step to rule out esophageal injury?
12th ribs are associated with: A. Insertion of chest tube
A. Flail chest B. Insertion of nasogastric tube
B. Damage to underlying abdominal C. Esophagogram with gastrografin
solid organs (liver, spleen, kidney) D. Esophagoscopy
C. Injury to aorta E. Peritoneal lavage
D. Injury to bronchi
E. Pneumothorax 345. Because of his involvement in a
motor vehicle accident, a 23-year-
342. In what cases patients with rib old football player has a chest wall
fractures should be treated injury. The only abnormal findings
immediately or monitored carefully: on clinical and radiologic
1. Elderly patients; 2. Patients with examination are a fracture of the left
concomitant heart diseases; 3. fifth to seventh ribs and a small
Patients with COPD; 4. Patients hemothorax. What should treatment
with multiple rib fractures; 5. include?
Patients with flail chest. A. Insertion of an intercostal drain
A. 1, 2, 3 to avoid pneumothorax
B. 2, 4, 5 B. Thoracotomy to treat a small
C. 3, 4, 5 hemothorax in the left base
D. 1, 2, 4 C. Insertion of a metal plate to fix
E. All are correct the fracture
53
D. Administration of analgesic minute. She complains of right
medication lower chest wall and severe right
E. Administration of cortisone to upper quadrant (RUQ) tenderness.
prevent callus formation Her breath sounds are questionably
diminished. The immediate priority
346. A 25-year-old woman was stabbed is to perform which of the
by her boyfriend in the left chest. following?
On examination, she has a 1-cm A. Peritoneal lavage
stab wound just inferior to her left B. Chest x-ray
breast in the mid-clavicular line. C. CT scan of chest and abdomen
There is jugular venous distension D. Thoracentesis with an 18-gauge
and breath sounds are completely needle
absent on the left side. She is E. Endotracheal intubation
becoming extremely dyspneic and
hypoxic. Make the diagnosis. 349. In the case of isolated
A. Cardiac tamponade pneumothorax the tube should be
B. Tension pneumothorax placed in the:
C. Massive hemothorax A. Second intercostal space,
D. Flail chest anterior axillary line
E. Rupture diaphram B. Second intercostal space,
midclavicular line
347. A 45-year-old man was a C. Second intercostal space, mid
passenger in a car when he was T- axillary line
boned by a truck at a high speed. He D. Fifth intercostal space,
is short in breath, complains of midclavicular line
severe pain in the chest, and is E. Fifth intercostal space, mid
hypoxic on the pulse oximeter. The axillary line
breath sounds are diminished on the
left and the percussion note is 350. In the case of hemothorax the tube
completely dull. He rapidly should be placed in the:
becomes tachycardic and A. Second intercostal space,
hypotensive. anterior axillary line
A. Cardiac tamponade B. Second intercostal space,
B. Tension pneumothorax midclavicular line
C Open pneumothorax C. Second intercostal space, mid
D. Flail chest axillary line
E. Massive hemothorax D. Fifth intercostal space,
midclavicular line
348. A 40-year-old woman is brought E. Fifth intercostal space, mid
to the emergency department axillary line
following a car crash in which she
was the driver. In the emergency 351. A 55-year-old man involved in an
department, her blood pressure is automobile accident is unresponsive
80/60 mm Hg, pulse is 128 bpm, and is intubated at the scene. On
and respiratory rate is 36 breaths per arrival in the emergency
54
department, he responds to painful A. Insert a chest tube and observe
stimulation. His systolic BP is 60 for drainage.
mm Hg, his HR is 140 bpm, his B. Perform an immediate right
neck veins are distended, and his thoracotomy.
breath sounds are absent on the left C. Perform an angiogram to rule out
side. Immediate management great vessels injury.
should involve which of the D. Perform median sternotomy with
following? extension along with right anterior
A. Insertion of a central venous line boarder of the sternocleidomastoid
on the right side muscle.
B. Insertion of an 18-gauge needle E. Perform a CAT scan with
in the left second intercostal space contrast, to evaluate extent of
C. Pericardiocentesis injury.
D. Peritoneal lavage
E. CT scan of head 354. A 31-year-old man is brought to
the emergency room following an
352. During a car crash a young man automobile accident in which his
suffers bilateral multiple fracture chest struck the steering wheel.
ribs. He is alert and presents Examination reveals stable vital
shortness of breath. His blood signs, but the patient exhibits
pressure is 100/60 mm Hg and chest palpable 7 rib fractures from the
is unstable. Treatment for this is: right side and paradoxical
A. Prolonged intubation and movement of the right side of the
ventilatory support until rib chest. Chest x-ray shows no
fractures heal along with aggressive evidence of pneumothorax or
bronchial toilette. hemothorax, but a large pulmonary
B. Once the patient is stable, open contusion is developing. Proper
rib fracture reduction and treatment would consist of which of
stabilization with plates. the following?
C. Fracture stabiliztion, with towel A. Tracheostomy, mechanical
clips on ribs and attached to weights ventilation, and positive end-
(external fixation). expiratory pressure
D. Avoid intubation, control pain, B. Stabilization of the chest wall
and perform aggressive bronchial with sandbags
toilette. C. Stabilization with towel clips
E. Temporary extracorporeal D. Immediate operative stabilization
circulation to allow fractures to E. No treatment unless signs of
heal. respiratory distress develop

353. A young man is shot at the level 355. Select the proper intervention for
of the right sternoclavicular joint. a life-threatening injury of the chest:
His blood pressure is 80/60 mm Hg, LARYNGEAL OBSTRUCTION
pulse 120 bpm, and a chest xray A. Endotracheal intubation
shows a right hydropneumothorax. B. Cricothyroidotomy
The first step should be: C. Subxiphoid window
55
D. Tube thoracostomy associated with ribs fracture; 4. A
E. Occlusive dressing massive air leak following chest
tube insertion; 5. High rate of blood
356. 29. Select the proper intervention loss via the chest tube (>200 ml/h).
for a life-threatening injury of the A. 1,2,3
chest: OPEN PNEUMOTHORAX B. 1,3,5
A. Endotracheal intubation C. 1,4,5
B. Cricothyroidotomy D. 2,4,5
C. Subxiphoid window E. 2,3,4
D. Tube thoracostomy
E. Occlusive dressing 361. Name indications for immediate
surgery in patients after chest
357. Select the proper intervention for trauma: 1. Traumatic disruption
a life-threatening injury of the chest: with loss of chest wall integrity; 2.
FLAIL CHEST Cardiac tamponade; 3.
A. Endotracheal intubation Tracheoesophageal fistula; 4. A
B. Cricothyroidotomy persistent thoracic duct
C. Subxiphoid window fistula/chylothorax; 5. Large vessel
D. Tube thoracostomy injury.
E. Occlusive dressing A. 1, 3, 4
B. 1, 2, 5
358. Select the proper intervention for C. 1, 3, 5
a life-threatening injury of the chest: D. 2, 4, 5
TENSION PNEUMOTHORAX E. 2, 3, 4
A. Endotracheal intubation
B. Cricothyroidotomy 362. What is the cornerstone in
C. Subxiphoid window management of patients with rib
D. Tube thoracostomy fractures?
E. Occlusive dressing A. Pain control
B. Immediate surgery
359. Select the proper intervention for C. Endotracheal intubation
a life-threatening injury of the chest: D. Ipsilateral chest tube placement
PERICARDIAL TAMPONADE E. Diagnostic thoracotomy
A. Endotracheal intubation
B. Cricothyroidotomy 363. Concerning thoracic trauma which
C. Subxiphoid window statement IS NOT true?
D. Tube thoracostomy A. Rib fractures do not require
E. Occlusive dressing surgery
B. Flail chest is associated with
360. Name please indications for paradoxical motion of the flail
immediate surgery in patients with segment
chest trauma: 1. Traumatic C. Cardiac tamponade is an
disruption with loss of chest wall indications for immediate surgery
integrity; 2. A chronic clotted D. First and second rib fractures are
hemothorax; 3. Athelectasis caused by excessive energy force
56
E. Isolated first and second rib D. 2,4,5
fractures require surgical therapy E. 2,3,4

364. What is the obvious sign of 367. All conditions belong to blunt
diaphragmatic disruption on chest cardiac injuries, except:
radiographs? A. Rupture of the valves,
A. Abdominal visceral herniation B. Rupture of interventricular
into the chest septum
B. Absence of complete expansion C. Cardiac chamber rupture
of the lung D. Cardiac tamponade
C. Distended shade of the E. Atrioventricular stenosis
mediastinum
D. Mediastinum dislocation to the 368. Which condition CAN NOT occur
contralateral side in patients with chest trauma?
E. Evidence of ipsilateral A. Pericardial tamponade
pneumothorax B. Esophageal rupture
C. Esophageal ahalasia
365. Which statement IS NOT true D. Heart contusion
сoncerning thoracic trauma? E. Main bronchial disruption
A. All patients with pneumothorax
due to trauma need a tube 369. Trauma is the leading cause of
thoracostomy death, morbidity, hospitalization,
B. Open pneumothorax is caused by and disability in Americans in the
lung tissue defect that is larger age:
than the cross-sectional area of A. 0 – 1y
the larynx B. 1 – 10y
C. Treatment for an open C. 10 – 45y
pneumothorax consists of D. 45 – 65y
placing a 3-way occlusive E. More than 65y
dressing over the wound
D. Tension pneumothoraces are 370. What injury DOES NOT
always life-threatening states compromise ventilation?
E. Large, clotted hemothoraces may A. Lung contusion
require an operation B. Open pneumothorax
C. Painfull ribs fracture
366. What are the primary therapies for D. Flail chest
pulmonary contusions? 1. Surgical E. Acute cardiac tamponade
debridement; 2. Pain control, iv
fluid restriction; 3. Immobilization 371. Measurement of serum creatine
with a figure-of-eight dressing; 4. kinase isoenzyme (creatine kinase-
Pulmonary toilet; 5. supplemental MB) levels is frequently
oxygen. performed in patients with
A. 1,3,4 possible:
B. 1,3,5 A. Blunt myocardial injury
C. 1,2,5 B. Multiple ribs fracture
57
C. Traumatic asphyxia sufficiently to drain its
D. Flail chest developing contents, cavitation can occur
E. Tension pneumothorax D. Lung abscesses occur in patients
with oral or dental infections
372. What is the initial study of choice who sustained a depression in
in patients with thoracic blunt their level of consciousness and
trauma, suspicious on aspirated their oral secretions
pneumothorax? E. Abscess is walled off the
A. The chest radiogram infectious process
B. 12-lead ECG
C. Echocardiography 376. Name the microorganism, which
D. CT scaning causes aspiration lung abscess most
E. Ventilation test oftenly:
A. Anaerobic bacteria
373. Ultrasound examinations of B. Gram-positive bacteria
thoracic cavities can be performed C. Gram-negative bacteria
to confirm the diagnosis: D. Fungal infections
A. Acute cardiac tamponade E. Protozoan infection
B. Open pneumothorax
C. Multiple ribs fracture 377. Choose incorrect statement
D. Main bronchi disruption concerning instrumental diagnosis of
E. Multiple athelectasis lung abscess:
A. The diagnosis is confirmed with
374. Choose signs typical for a chest radiograph
pneumonia: 1. Abnormal temperature; B. A chest radiograph demonstrate
2. White blood cells in an air–fluid level
the sputum; 3. Abnormal white blood C. Bronchoscopy is essencial in
cell count; 4. Infiltrate on chest making the diagnosis lung
radiographs; 5. Crepitation on abscess
auscultation: D. In patients with concomitant
A. 1, 2, 3 lung diseases, most efficient in
B. 1, 2, 4 diagnosing lung abscess is CT
C. 1, 2, 5 scan
D. 2, 4, 5 E. The goal of bronchoscopy is to
E. All are correct determine the bacterial identity
and sensitivities
375. Which statement is wrong
concerning lung abscess? 378. Which statement concerning the
A. Infecting organisms involved in treatment of patients with lung abscess
a pneumonic process in the lung is wrong?
will promote abscess formation A. Most lung abscesses will
B. Most lung abscesses occur in the respond and resolve with
upper lobe of lung appropriate antibiotic therapy
C. If the abscess cavity erodes into and pulmonary toilet
a bronchoalveolar space
58
B. Surgical therapy is reserved for C. Gram-negative bacteria
those cases that fail to resolve D. Fungal infections
with nonoperative management E. Protozoan infection
C. Most lung abscesses will require
surgery 382. Which diseases can cause
D. Surgical therapy is reserved for empyema of the pleura: 1.
those patients who develop Parapneumonic effusion; 2. Congestive
severe hemoptysis, heart failure; 3. Lung abscess cavity
bronchopleural fistula or erodes into a pleural space; 4.
empyema Tuberculosis; 5. Collagen-vascular
E. Pulmonary resection is disease.
warranted if the abscess cavity is A. 1, 3, 4
larger than 6 cm in diameter for B. 1, 2, 4
more than 8 weeks of aggressive C. 1, 4, 5
antibiotic therapy D. 2, 3, 4
E. 3, 4, 5
379. After suffering a severe bout of
pneumonia, a 46-year-old renal 383. Choose correct principles in
transplantation patient develops a lung treatment of empyema: 1. Complete
abscess. She has been receiving evacuation of the content of
immunosuppression therapy since her infected space; 2. Elimination of
last kidney transplantation 3 years ago. cavity; 3. Control of causative
What is the most appropriate organisms/sterilisation; 4. Nutritional
treatment? support; 5. Ventilation support:
A. Needle aspiration A. 1, 2, 3
B. Urgent thoracotomy B. 2, 3, 4
C. Antituberculous therapy C. 1, 4, 5
D. Antibiotics and vigorous D. 3, 4, 5
attempts to obtain bronchial E. All are correct
drainage
E. Insertion of an intercostal pleural 384. Which microbiological aspect is
drin not correct concerning empyema:
A. 5-10% of parapneumonic
380. What minimal volume of pleural effusions become empyema
effusion can be seen on chest film? B. A lack of detectable causative
A. 50 cc organisms reported not more
B. 70 cc then 10%
C. 175 cc C. Blood cultures positive in only
D. 600 cc 10% of patients
E. 1000 cc D. Community acquired empyema
most oftenly is caused by
381. Name the microorganism, which Streptococcus pneumoniae
causes empyema most oftenly: E. Hospital-acquired empyema
A. Anaerobic bacteria most oftenly is caused by
B. Gram-positive bacteria Staphylococcus aureus
59
E. Chronic obstructive pulmonary
385. Immediately following a bout of deseases
pneumonia, a young woman develops a
large pleural effusion. A chest tube is 388. What is the most common
inserted and 600 mL of thin pus is symptom associated with pleural
obtained. A CAT scan shows effusion?
incomplete drainage and multiple A. Dyspnea
intrapleural loculations. Management B. Caugh
of this empyema requires: C. Pain
A. Insertion of multiple chest D. Diffuse edema
tubes under CAT guidance to E. Anaemia
drain either most or all
loculations 389. Physical examination is NOT
B. Treat the patient with informative until pleural effusion
antibiotics and continue single exceeds:
chest tube drainage A. 100 mL
C. Treat patient with antibiotics B. 300 mL
and continue single chest tube C. 30 mL
drainage waiting for a thick D. 1000 mL
peel to develop and then E. 2500 mL
proceed with open total lung
decortication 390. In the case of pleural
D. Proceed with thoracoscopy and effusion physical findings include
intrapleural toilette. Break the all of the following, EXCEPT:
loculations and place drains A. Decreased breath sounds
E. A thorough open total lung B. Dullness to percussion
decortication immediately C. Decreased tactile fremitus
D. Bronchial breath sounds
386. Which statements are NOT true? E. Pleural friction rub
A. The lungs occupy most of the
volume of each hemithorax 391. Transudates are caused by
B. Each lung is divided into lobes a small, defined group of etiologies,
C. The right lung has 3 lobes including the following: 1).
D. The left lung has 2 lobes Congestive heart failure, 2). Spread
E. Each segment is further divided TB infection, 3). Atelectasis due to
into lobes. malignancy or pulmonary
embolism, 4). Hypoalbuminemia,
387. The most often pleural effusion is 5). Congenital lung deseases.
caused by all of the following, A. 1,2,3
EXCEPT: B. 1,3,4
A. Congestive heart failure C. 1,4,5
B. Malignancy D. 2,3,5
C. Infections E. 2,4,5
D. Pulmonary embolism
60
392. The most common causes of 4). Fistula formation; 5). Spleen
exudates include the following: 1). or liver puncture
Congestive heart failure, 2). A. 1,3,5
Pulmonary embolism, 3). B. 1,2,4
Esophageal perforation, 4). C. 1,3,4
Myxedema, 5). Chylothorax D. 2,3,5
A. 1,2,3 E. 2,4,5
B. 1,3,4
C. 1,4,5 396. The recommended limit
D. 2,3,5 in a single thoracentesis procedure
E. 2,4,5 is:
A. 1000-1500 mL
393. Choose indications for diagnostic B. 150-200 mL
thoracentesis from the following: 1). C. 300-450 mL
Etiology of the effusion is unclear; D. 600-7500 mL
2). Effusion does not respond to E. 50-100 mL
therapy as expected; 3). Small
amoung of effusion in clinically 397. Pleurodesis is most
stable patients; 4). Underlying oftenly used for:
congestive heart failure; 5). A. Recurrent malignant effusions
Suspected malignancy B. Spreading TB infection
A. 1,2,5 C. COPD associated with pleural
B. 1,2,3 effusion
C. 1,4,5 D. If diagnosis of pyopneumothorax
D. 2,3,4 is confirmed
E. 3,4,5 E. Empiema associated with lung
abscess
394. Name contraindications for
diagnostic thoracentesis: 1). 398. Surgical intervention is
Pulmonary hypertension; 2). Small most oftenly required for:
volume of fluid; 3). A. Parapneumonic effusions that
Hypocoagulation; 4). Suspected cannot be drained adequately by
malignancy; 5). Mechanical needle
ventilation B. Lung abscess development after
A. 1,2,5 pneumonia
B. 1,2,4 C. Total hemithoracis effusion
C. 1,3,4 without clear diagnosis
D. 2,3,5 D. Multiple lung abscesses
E. 3,4,5 E. Lung abscess with purulent
sputum more than 100 mL/day
395. Complications of diagnostic
thoracentesis include: 1). Internal 399. In the management of
bleeding; 2). Systemic chylous effusions following action
hypotension; 3). Pneumothorax; is efficient:
A. Restrictions of fat intake
61
B. Restrictions of water intake 403. What condition DO
C. Meal rich for proteins NOT cause secondary abscess of the
D. Meal rich for carbohydrate lungs:
E. All are correct A. Preexisting bronchial obstruction
B. Infection spreading from an
400. Which statement extrapulmonary location
concerning effusion, obtained by C. Bronchiectasis
diagnostic thoracocentesis, is NOT D. Immunocompromised state of
TRUE? the patient
A. A putrid odor suggests an E. Heart failure
anaerobic empyema.
B. A milky, opalescent fluid 404. Most frequently, the lung
suggests a chylothorax abscess is:
C. All are correct A. complication of aspiration
D. Frankly purulent fluid indicates pneumonia caused by mouth
an empyema. anaerobes
E. Grossly bloody fluid may result B. caused by Peptostreptococcus
from trauma, malignancy species
C. unrecognizable on chest X-ray
401. The fluid, obtained by film
diagnostic thoracocentesis, is D. unexpected finding in healthy
considered an exudate if (choose patients
one correct sign): E. complication after acute
A. Ratio of pleural fluid to serum bronchitis
protein greater than 0.5
B. Low pleural fluid glucose 405. Which mechanism in
concentration (< 50 mg/dL) pathogenesis of lung abscess
C. Ratio of pleural fluid to serum formation is WRONG?
lactate dehydrogenase (LDH) lesser A. Bacteremia
than 0.1 B. Tricuspid valve endocarditis
D. Any of them C. Lemierre syndrome (acute
E. Pleural fluid pH less than 5.1 oropharyngeal infection
followed by septic
402. Definition of lung abscess thrombophlebitis of the internal
includes all, except: jugular vein)
A. necrosis of the pulmonary tissue D. Pulmonary embolism
B. formation of cavities containing E. Severe enterocolitis
necrotic debris
C. formation of cavities containing 406. What factors ARE NOT
necrotic fluid associated with a poor prognosis in
D. caused by microbial infection treatment of lung abscesses:
E. obstructive changes in the small A. advanced age
bronchi B. malnutrition
C. immunodeficiency
D. malignancy
62
E. mitral regurgitation E. 2,4,5

407. Which statements are 411. Name the most often


NOT true? reason for poor response to
A. Male sex predominates for lung treatment with antibiotics, in
abscess patients with lung abscesses:
B. Lung abscesses likely occur A. Bronchial obstruction with a
more commonly in elderly patients foreign body
C. Incidence of periodontal disease B. Tumor
increase likelyhood of lung abscess C. Reccurent aspiration
formation D. Fungial infection
D. Alcoholism is preexisting factor E. Poor lung perfusion
for abscess occurring
E. Cigarette smoking is strong 412. The usual indications for
underlying factor for lung abscess surgery, in patients with lung
abscess, are : 1). coexsisting severe
408. The unusual symptom of obstructive lung desease, 2).
acute lung abscess is: suspected neoplasm, 3). congenital
A. Fever lung malformation, 4). failure to
B. Cough with sputum discharge respond to medical management, 5).
C. Sweating amount of purulent sputum more
D. Chest pain than 100ml/day
E. Weight loss A. 1,2,3
B. 1,3,5
409. Objective findings in C. 1,4,5
patient with lung abscess are all, D. 2,3,4
EXCEPT: E. 2,4,5
A. Decreased breath sounds
B. Dullness on percussion 413. Name the complications
C. Bronchial breath sounds of pulmonary abscess: 1). Rupture
D. Inspiratory crackles into pleural space causing
E. Tachicardia associated with empyema, 2). Pulmonary
bradypnoe hypertension, 3). Spreading
bronchial sclerosis, 4).
410. Patients at the highest risk Bronchopleural fistula, 5).
for developing lung abscess have Respiratory failure.
the following risk factors: 1). A. 1,2,3
Periodontal disease, 2). Mild to B. 1,3,5
severe heart failure, 3). Alcohol C. 1,4,5
abuse, 4). Dysphagia, 5). Antibiotic D. 2,3,4
use in past. E. 2,4,5
A. 1,2,3
B. 1,3,4 414. Which of the following is
C. 1,4,5 FALSE?
D. 2,3,5
63
A. Postaspiration lung abscesses are indistinct outlines on the right
associated with aerobic bacteria side at the level of the 4-th rib. In
B. Prevention of aspiration is the centre of this opacity there is a
important to minimize the risk of horizontal level and clearing of lung
lung abscess tissue above it. What disease does
C. Early intubation in disable this X-ray pattern correspond with?
patients protects the airway from A. Right-sided hemothorax
massive aspiration B. Abscess of the right lung
D. The most common is aspiration C. Peripheral cancer
of oropharyngeal contents. D. Right-sided pneumothorax
E. Lung abscesses as a result of E. Tuberculoma of the right lung
aspiration most frequently occur
in the posterior segments of the 417. A 35 y.o. patient was admitted to
upper lobes or the superior the local hospital a week after a
segments of the lower lobes road accident with clinical picture
of clotted hemothorax. What is the
415. Which statements, most appropriate treatment tactic for
concerning investigations of prevention of acute pleural
patients with lung abscesses, are empyema?
TRUE? 1). A complete white blood A. Surgical removal of clotted
cell count with differential may hemothorax
reveal leukocytosis and a left shift; B. Treatment by pleural punctions
2). CT scan is the initiate step in C. Complex conservative therapy
evaluation of the patient with lung D. Passive drainage of pleural
abscess; 3). On chest X-ray lung cavity
abscess is an irregularly shaped E. Active drainage of pleural cavity
cavity with an air-fluid level inside;
4). Bronchoscopy has a very high 418. A 35 y.o. woman was admitted to
sensitivity and specificity in thoracic surgery department with
reaviling of lung abscesses; 5). fever up to 400C, onset of pain in
Peripheral lung abscesses with the side caused by deep breathing,
pleural contact or included inside a cough with considerable quantity of
lung consolidation are detectable purulent sputum and blood with bad
using lung ultrasonography at the smell. What is the most likely
bedside. Choose the correct diagnosis?
combination: A. Complication of liver
A. 1,2,3 echinococcosis
B. 1,3,5 B. Bronchiectatic disease
C. 1,4,5 C. Pulmonary tuberculosis
D. 2,3,4 D. Actinomycosis of lungs
E. 2,4,5 E. Abscess of the lung

416. X-ray pattern of thorax organs 419. A 52 year old patient complains
revealed a large intensive about pain in the right part of her
inhomogeneous opacity with chest, dyspnea, cough with a lot of
64
foul-smelling albuminoid sputum in
form of "meat slops". Objectively:
the patient's condition is grave,
cyanosis is present, breathing rate is
31/min, percussion sound above the
right lung is shortened, auscultation
revealed different rales. What is the
most probable diagnosis?
A. Chronic pneumonia
B. Pleura empyema
C. Lung abscess
D. Lung gangrene
E. Multiple bronchiectasis
65
1. Overal mortality rate in case of acute D. Appendicular abscess;
appendicitis is: E. Peritonitis.
A. 10-20%;
B. 5-10%; 7. A 34-year-old female patient suffered from
C. 0,2-0,8%; abdominal pain week ago; no other
D. 1-5%; gastrointestinal problems were noted. On
E. 25%. clinical examination, a mass of about 6 cm
was palpable in the right lower quadrant,
2. Name the destructive form of appendicitis. appeared hard, not reducible and fixed to
A. Appendicular colic; the parietal muscle. CBC: leucocyts –
B. Superficial; 7,5*109/l, ESR – 24 mm/hr. Temperature
C. Appendix hydrops; 37,4˚C. Triple antibiotic therapy with
D. Phlegmonous; cefotaxime, amikacin and tinidazole was
E. Catarrhal appendicitis. very effective. After 10 days no mass in
abdominal cavity was palpated. What time
3. Koher sign is: term is optimal to perform appendectomy?
A. Migration of the pain from the A. 1 week;
epigastrium to the right lower B. 2 weeks;
quadrant; C. 3 month;
B. Pain in the right lower quadrant; D. 1 year;
C. One time vomiting; E. 2 years.
D. Pain in the right upper quadrant;
E. Pain in the epigastrium. 8. What instrumental method of examination
is the most efficient in case of portal
4. In cases of appendicular infiltration is pyelophlebitis?
indicated: A. Plain abdominal film;
A. Laparoscopic appendectomy; B. Barium meal;
B. Concervative treatment; C. US;
C. Open appendectomy; D. Termography;
D. Draining; E. Doppler ultrasound.
E. Laparotomy.
9. The complications of acute appendicitis
5. In cases of appendicular abscess is are all, except:
indicated: A. Appendicular infiltration;
A. Laparoscopic appendectomy; B. Appendicular abscess;
B. Concervative treatment; C. Enzyme peritonitis;
C. Open appendectomy; D. Pyelophlebitis;
D. Draining, if possible - appendectomy; E. Sepsis.
E. Ileo-cecal resection.
10.A 63-year-old male patient in reasonably
6. A 34-year-old female patient suffered from good health suddenly suffered from fever
abdominal pain week ago; no other (>38°C) and a painful right iliac fossa
gastrointestinal problems were noted. On tumefaction; no other gastrointestinal
clinical examination, a mass of about 6 cm problems were noted. On clinical
was palpable in the right lower quadrant, examination, a mass of about 5 cm was
appeared hard, not reducible and fixed to the palpable in correspondence of right iliac
parietal muscle. CBC: leucocyts – 7,5*109/l, fossa and appeared hard, not reducible and
ESR – 24 mm/hr. Temperature 37,4˚C. Make fixed to the parietal muscle. Laboratory
the diagnosis? data showed leucocytosis, shift to the left
A. Appendicular colic; and elevated erythrocyte sedimentation
B. Appendicular hydrops; rate (74 mm/hr) as pathological findings.
C. Appendicular infiltration; Abdominal ultrasound examination
66
evidenced a fluid in the right lower and a gangrenous appendix is
quadrant, with heterogenic echotexture and removed. The stump is inverted. Why
a thickening of the ileocecal tract. does acute appendicitis in elderly
Abdominal CT, confirmed the presence of patients and in children have a worse
a complex, predominantly cystic, mass of prognosis?
large size (6×8 cm) with heterogeneous, A. The appendix is retrocecal.
mainly peripheral enhancement, the B. The appendix is in the preileal
adjacent cecum had its wall thickened and position.
it was not possible to differentiate the C. The appendix is in the pelvic position.
appendix separately from the mass, D. The omentum and peritoneal cavity
homolateral inguinal reactive appear to be less efficient in
lymphadenopathy was also present. The localizing the disease in these age
patient failed to respond to the initial groups.
conservative management, which consisted E. The appendix is longer in these age
of intravenous fluids and triple antibiotic groups.
therapy with cefotaxime, gentamicin and
metronidazole, without any improvement 13. A 12-year-old boy complains of pain in
of pain and fever. At a further ultrasound the lower abdomen (mainly on the right
examination, the mass appeared not side). Symptoms commenced 12 hours
modified. Make the diagnosis? before admission. He had noted anorexia
A. Appendicular infiltration; during this period. Examination revealed
B. Appendicular abscess; tenderness in the right iliac fossa, which
C. Peritonitis; was maximal 1 cm below McBurney’s
D. Appendicular colic; point. In appendicitis, where does the
E. Phlegmonous appendicitis; pain frequently commence?
A. In the umbilical region and then
11. A 17-year-old female model presents to moves to the right iliac fossa
the emergency room with a 1-day history B. In the back and moves to the right
of lower abdominal pain. On examination iliac fossa
she is most tender in the right lower C. In the rectal region and moves to the
quadrant (RLQ) and also has pelvic right iliac fossa
tenderness. White blood cell (WBC) D. In the right iliac fossa and remains
count is 13x109/l and temperature is there
38,2°C. A provisional diagnosis of E. In the right flank
uncomplicated appendicitis is made and
laparoscopic appendectomy is offered. 14. On examination, patients presenting with
Regarding laparoscopic appendectomy appendicitis typically show maximal
which of the following is TRUE? tenderness over which of the following?
A. It can be performed safely with A. Inguinal region
minimal morbidity compared to B. Immediately above the umbilicus
open technique. C. At a point between the outer one-third and
B. Length of hospital stay is longer than inner two-thirds of a line between the
with open technique. umbilicus and the anterior superior iliac
C. Posthospital recovery can be shorter spine
in uncomplicated appendicitis. D. At a point between the outer two-thirds
D. Return to full feeding is less than and inner one-third of a line between the
with open technique. umbilicus and the anterior superior iliac
E. Wound complication rate is greater spine
with open technique. E. At the midpoint of a line between the
umbilicus and the anterior superior iliac
12. A79-year-old man has had abdominal pain spine
for 4 days. An operation is performed,
67
15. A 29-year-old woman presents to her made. What morphological form acute
physician’s office with pain in the right appendicitis we must suspect?
iliac fossa. Examination reveals A. Gangrenous;
tenderness in this region. Her last B. Superficial;
menstrual cycle was 2 weeks previously. C. Appendix hydrops;
CBC: leukocyte count – 7,2x109/l, RBC D. Phlegmonous;
– 2,9x1012/l, Hb – 105 g/l. Make the E. Appendicular colic.
diagnosis.
A. Acute superficial appendicitis 19. A 20-year-old man has undergone
B. Ovarian apoplexy appendectomy for perforated
C. Renal colic appendicitis with generalized peritonitis.
D. Acute destructive appendicitis Seven days postoperatively, his
E. Acute pancreatitis temperature continues to spike to 39,5°C
despite antibiotic therapy with
16. A 28-year-old man is admitted to the ampicillin, gentamicin, and
emergency department complaining of metronidazole. Abdominal CT scan
pain in the umbilical region that moves reveals a large pelvic abscess. Soon
to the right iliac fossa. Which is a afterward, he has bleeding from the
corroborative sign of acute appendicitis? mouth and nose with increasing oozing
A. Referred pain in the right side with from the surgical wound and all
pressure on the left (Rovsing)sign intravenous puncture sites. What is the
B. Increase of pain with testiculalr most likely diagnosis?
elevation A. Anaphylactoid reaction to intravenous
C. Relief of pain in lower abdomen with dye;
extension of thigh B. Antibiotic-induced coagulopathy
D. Relief of pain in lower abdomen with C. Disseminated intravascular coagulation
internal rotation of right thigh D. Liver failure ;
E. Hyperesthesia in the right lower E. Congenital bleeding disorder.
abdomen
20. A 20-year-old man has undergone
17. At open operation a normal appendix is appendectomy for perforated
found, no other pathology in abdominal appendicitis with generalized peritonitis.
cavity. What is the most common Seven days postoperatively, his
procedure a surgeon should do if he temperature continues to spike to 39,5°C
finds a normal appendix? despite antibiotic therapy with
A. Evaluate the pelvis for tuboovarian ampicillin, gentamicin, and
abscess pelvic inflammatory disease, metronidazole. Abdominal CT scan
malignancy or etopic pregnancy reveals a large pelvic abscess. Soon
B. Removal of appendix afterward, he has bleeding from the
C. Evaluate the terminal ileum and mouth and nose with increasing oozing
cecum for signs of regional or from the surgical wound and all
bacterial enteritis intravenous puncture sites. What was a
D. Evaluate the upper abdomen for trigger for coagulopathy?
cholecystitis or perforated duodenal A. Sepsis;
ulcer B. Antibioticotherapy;
E. Evaluate for Meckel’s diverticulum C. Congenital disoder;
D. Proteolysis;
E.Operation.
18. M., 68-year old man, 14 hours ago
appeared continuous pain in the RLQ, 2 21. All listed below locations of appendix
hours ago pain decreased significantly. concerning cecum are correct, EXCEPT:
The diagnosis of acute appendicitis was A. medial
68
B. retrocecal
C. subhepatic 26. Typically, in the case of acute
D. pelvic appendicitis, symptoms of the disease
E. intermediate include: 1) rebound tenderness
(Blumberg’s sign) in the RLQ, 2)
22. Appendicular artery is the branch of: hematuria, 3) pain in the RLQ, 4)
A. a. ileocolica involuntary guarding, 5) normal body
B. a. mesenterica inferior temperature. Choose the RIGHT
C. a. hepatica communis combination.
D. celiac trunck A. 1, 3, 4
E. a. iliaca interna B. 1, 2, 4
C. 1, 2, 5
23. What is the Clado ligament? D. 2, 3, 5
A. lig. appendico-ovaricum E. 3, 4, 5
B. lig. appendico-cecalis
C. lig. appendico-transversum 27. Which sign reflects pelvic location of the
D. lig. circularis appendicis appendix?
E. lig. longitudinalis appendicis A. Rovsing’s sign
B. Blumberg’s sign
24. Classification proposed by V.I. Kolesov C. psoas sign
in 1972, is based on clinical course of D. obturator sign
disease and includes FOUR items. They E. Koher’s sign
are all, EXCEPT:
A. Appendicular colic 28. You will perform all listed laboratory
B. Simple appendicitis: superficial, tests to make the diagnosis and
catarrhal differential diagnosis of acute
C. Destructive appendicitis: appendicitis, EXCEPT?
phlegmonous, gangrenous, A. white blood cell (WBC) count
perforating B. urinanalysis
D. Complications of acute C. serum liver enzymes and amylase
appendicitis: appendicular levels
infiltrate, appendicular abscess, D. serum HCG (human chorionic
diffuse peritonitis gonadotropin) level in women of
E. Purulent complications: abscesses, childbearing age
sepsis E. serum protein

25. Which statement, concerning acute 29. Which statement, concerning acute
appendicitis, is NOT TRUE? appendicitis, is WRONG?
A. The tip of the appendix can be located A. Perforation of the appendix is
anywhere in the right lower quadrant accompanied with a sharp
of the abdomen or pelvis abdominal pain
B. Appendicitis occurs rarely in very B. Acute appendicitis with pelvic
young children and elderly persons. location of the appendix is always
C. Luminal obstruction leads to secretion characterized by clear clinical
of mucus and fluid with the rise in picture and typical course
luminal pressure causing ischemia of C. Often the patients, with acute
the wall of appendix appendicitis with pelvic location of
D. In general, patients with appendicitis the appendix, have diarrhea and/or
report nausea and loss of appetite dysuria
E. Migration of the pain from epigastric or D. White blood cells (WBC) are
paraumbilical region to right lower usually elevated in patients with
quadrant is a sign of patient’s recovery acute appendicitis
69
E. Infiltrate is being formed on 3-5 B. Oral cholecystogram
days of the disease C. Intravenous cholangiogram
D. Abdominal x-ray
30. On the 10th day after the admission to the E. ERCP
surgical department patient develops
suppuration of appendicular infiltrate. In 34. A 48-year-old woman is admitted to the
this case, clinical picture will include all hospital with severe abdominal pain,
symptoms, EXCEPT: tenderness in the right hypochondrium,
A. pain in the region of abdominal and a WBC count of 12x109/l. Acute
mass cholecystitis is established. After
B. fatigue diagnosis, cholecystectomy should be
C. headache performed within which of the following?
D. hectic fever A. 20–60 minutes
E. breathlessness B. The first 1-2 days following hospital
31. In attempting to minimize complications admission
during cholecystectomy, the surgeon C. 8 days
defines the triangle of Calot during the D. 3 weeks
operation. The boundaries of the triangle E. 3 months
of Calot (modified) are the common
hepatic duct medially, the cystic duct 35. A 60-year-old diabetic man is admitted to
inferiorly, and the liver superiorly. Which the hospital with a diagnosis of acute
structure courses through this triangle ? cholecystitis. The WBC count is 28
A. Left hepatic artery x109/l, and a plain film of the abdomen
B. Right renal vein and CT scan show evidence of intramural
C. Right hepatic artery gas in the gallbladder. What is the most
D. Cystic artery likely diagnosis?
E. Superior mesenteric vein A. Gallstone ileus
B. Acalculous cholecystitis
32. A 65-year-old woman is admitted with C. Cholangiohepatitis
RUQ pain radiating to the right shoulder, D. Sclerosing cholangitis
accompanied by nausea and vomiting. E. Emphysematous gallbladder
Examination reveals tenderness in the
RUQ and a positive Murphy’s sign. A 36. A 60-year-old woman is recovering from
diagnosis of acute (uncomplicated) a major pelvic cancer operation and
cholecystitis is made. What is the most develops severe abdominal pain and
likely finding? sepsis. Acute cholecystitis is established,
A. Serum bilirubin levels may be elevated laparotomy is performed. The gallbladder
B. Cholelithiasis is present in 90% of all is severely inflamed and removed. There
cases is no evidence of gallbladder stones
C. Bacteria are rarely found at operation (acalculous cholecystitis).
D. An elevated amylase level excludes this Cholecystectomy is performed. Which is
diagnosis true of acalculous cholecystitis?
E. A contracted gallbladder is noted on A. It is usually associated with stones in
ultrasound the CBD.
B. It occurs in 80–90% of cases of
33. A 38-year-old male lawyer develops cholecystitis.
abdominal pain after having a fatty meal. C. It has a more favorable prognosis than
Examination reveals tenderness in the right calculous cholecystitis.
hypochondrium and a positive Murphy’s D. It is increased in frequency after
sign. Which test is most likely to reveal trauma or operation.
acute cholecystitis? E. It is characterized on US scan by
A. US of the abdomen inlarged cystic duct.
70
C. Cholecystohepatic duct of Linka
37. Following recovery in the hospital from a D. Cholecystohepatic duct of Calot
fracture of the femur, a 45-year-old E. Cholecystohepatic duct of Koher
female patient develops RUQ abdominal
pain and fever. She has tenderness in the 41. Different hormones regulate function of
right subcostal region. There is evidence the gallbladder. Choose all of them: 1).
of progressive sepsis and hemodynamic Vasoactive intestinal polypeptide; 2).
instability. The WBC count is 24 x109/l. Cholecystokinin; 3). Thyroxine; 4).
A bedside sonogram confirms the Lipase; 5). Motilin.
presence of acalculous cholecystitis. A. 1, 2, 3
What should treatment involve? B. 1, 2, 5
A. Intravenous antibiotics alone C. 1, 3, 4
B. ERCP D. 3, 4, 5
C. Percutaneous drainage of the E. 2, 3, 4
gallbladder
D. Urgent cholecystectomy 42. What is the most common symptom
E. Elective cholecystectomy after 3 related to acute cholecystitis?
months A. RUQ abdominal pain
B. Jaundice
38. Following recovery in the hospital from a C. Fever
fracture of the femur, a 75-year-old D. Nausea and vomiting
nursing home female patient develops E. Weight loss
RUQ abdominal pain and fever. She has
tenderness in the right subcostal region. 43. Which condition does NOT predispose to
There is evidence of progressive sepsis gallstone formation?
and hemodynamic instability. The WBC A. Concentrated bile
count is 24 x109/l. A bedside sonogram B. Increased gallbladder motility
confirms the presence of acalculous C. Female sex
cholecystitis. What should treatment D. Obesity
involve? E. Previous truncal vagotomy
A. Intravenous antibiotics alone
B. Percutaneous drainage of the 44. Patient, who suffers from acute
gallbladder cholecystitis, can have all listed below
C. ERCP symptoms, EXCEPT:
D. Urgent cholecystectomy A. Nausea and vomiting
E. Elective cholecystectomy after 3 B. Anorexia
months C. Fever
D. Arterial hypotention
39. The gallbladder consists of following E. RUQ pain
parts, EXCEPT:
A. Fundus 45. Physical examination in patient with acute
B. Body cholecystitis, can reveale all signs, EXCEPT:
C. Infundibulum A. Guarding and rebound tenderness
D. Neck in the RUQ
E. Cardia B. Positive Murphy’s sign
C. Painful mass in the RUQ
40. Sometimes small accessory bile duct may D. Positive Blumberg’s in the RUQ
drain directly into the gallbladder. The E. Absence of liver dullness
name of this duct is?
A. Cholecystohepatic duct of Luschka 46. Which statement concerning imaging
B. Cholecystohepatic duct of studies in patients with acute cholecystitis is
Laschtuvka WRONG?
71
A. The most important value of plain C. Increased gallbladder motility
abdominal film is exclusion of other D. Decreased liver function
abdominal pathology (perforated E. Hyperglycemia
peptic ulcer)
B. Ultrasound of the abdomen is an 52. What type of bile stones didn’t exist?
effective method for identifying A. Cholesterol
gallstones in case of acute B. Pigment
cholecystitis C. Mixed
C. Gallstones and bile appear nearly D. Calcium
isodense on CT E. Protein
D. Cholangiography is the most accurate
method the extrahepatic biliary tree 53. At abdominal examination: patient is
visualization asked to breathe out and then physician gently
E. All patients with acute cholecystitis place the hand below the costal margin on the
should undergo MRI right side at the mid-clavicular line, the
patient is then instructed to breathe in. If the
47. The most efficient noninvasive patient stops breathing in the test is
instrumental method of examination in case considered positive. Name this test?
of acute cholecystitis is: A. Murphy’s sign
A. Oral cholecystography B. Kehr’s sign
B. Ultrasonography C. Blumberg’s sign
C. Termography D. Rovsing’s sign
D. GIT X-ray examination E. Psoas sign
E. IV cholangiography
54. Patient: 44 years female. Three weeks ago
48. Which signs are positive in patients with felt acute pain in right upper quadrant
acute cholecystitis? after fatty food, spasmolitics were
A. Ortner, Kehr, Murphy efficient. Complaints on admission: dull
B. Kocher, Sitkovski, Rovzing pain in right upper quadrant. Examination:
C. Pasternatski temperature 36,8ºC, soft mass palpated in
D. Mayo-Robson Kehr point. CBC: WBC – 6,8x109/l. US:
E. Cullen, Grey Turner very large gallbladder with thin wall,
stone 15 mm in diameter obstructing it,
49. Normal diameter of CBD is: common bile duct 7 mm in diameter.
A. up to 25 mm Make the diagnosis?
B. up to 15 mm F. Acute cholecystitis;
C. up to 10 mm G. Biliary pancreatitis;
D. up to 20 mm H. Gallbladder hydropsia;
E. up to 5 mm I. Cholangitis;
J. Mechanical jaundice.
50. Normal diameter of cystic duct is:
A. 2 mm 55. The management in case of acute
B. 3-4 mm cholecystitis complicated with
C. 10-12 mm choledocholithiasis should be?
D. 20 mm A. Conventional cholecystectomy;
E. 8-9 mm B. Laparoscopic cholecystectomy;
C. Cholecystostomy;
51. Choose the reasons which are responsible D. ECRP, sphincterotomy,
for gallstones formation during the cholecystectomy;
pregnancy: E. Concervative treatment.
A. Hyperestrogenemia, cholestasis
B. Decraesed duodenal motility
72
56. To what complication can lead acute obstructive B. Complete the laparoscopic
cholecystitis? cholecystectomy and repeat an
A. Cancer of pancreatic gland; ultrasound postoperatively. Observe
B. Pleural empiema; the patient if no CBD stone is
C. Gallbladder empiema; visualized
D. Perforative duodenal ulcer; C. Perform a CBD exploration either
E. Acute bowle obstruction. laparoscopically or open along with a
cholecystectomy
57. Which complication of acute cholecystitis D. Complete the laparoscopic
develops, when the patient have jaundice, cholecystectomy, no further treatment
high temperature (>38ºC), pain in upper is necessary
right quadrant, swetting: E. Complete the laparoscopic
A. Acute pancreatitis; cholecystectomy and plan for a
B. Bile duct fistula; postoperative hydroxy iminodiacetic
C. Gallbladder empiema; acid (HIDA) scan
D. Duodenal ulcer; 60. A 42-year-old accountant presents with
E. Cholangitis. recurrent RUQ pain of 3-year duration. He
had undergone a laparoscopic
58. An 85-year-old man is brought to the cholecystectomy 2-years ago for
hospital with a 2-day history of nausea presumed symptomatic cholelithiasis, but
and vomiting. He has not passed gas or the pain persisted. An upper GI endoscopy
moved his bowels for the last 5 days. is normal. A sonogram and CT scan of the
Abdominal films show dilated small abdomen are normal. An ERCP is
bowel, no air in the rectum and air in the performed, and the pressure in the CBD is
biliary tree. Which of the following 45-cm saline (normal bile duct pressure is
statements is TRUE? 10–18- cm saline). What is the most likely
A. Air in the biliary tree associated with diagnosis?
small-bowel obstruction suggests a A. Acalculous cholecystitis
diagnosis of gallstone ileus; B. Emphysematous cholecystitis
B. An enterotomy should be distal to the C. Biliary dyskinesia
site of obstruction and the stone D. Cancer of the gallbladder
should be removed; E. Myasthenia gravis
C Gallstone ileus is more common in the
young adults; 61. A 42-year-old accountant presents with
D. Cholecystectomy is contraindicated; recurrent RUQ pain of 3-year duration. He
E. Small-bowel obstruction usually occurs had undergone a laparoscopic
in the distal jejunum. cholecystectomy 2-years ago for presumed
symptomatic cholelithiasis, but the pain
59. An intraoperative cholangiogram is persisted. An upper GI endoscopy is normal.
performed during an elective laparoscopic A sonogram and CT scan of the abdomen are
cholecystectomy on a 30-year-old woman. normal. An ERCP is performed, and the
She has no previous surgical history. pressure in the CBD is 45-cm saline (normal
There is a 0.8-cm filling defect in the bile duct pressure is 10–18- cm saline).
distal common bile duct (CBD). The Choose the best treatment option:
surgeon should: A. Calcium channel blockers, if not
A. Complete the laparoscopic effective an endoscopic
cholecystectomy and check liver sphincterotomy
function tests (LFTs) postoperatively. B. Antibiotics, if not effective an
If they are normal, no further endoscopic sphincterotomy
treatment is needed C. Analgesics, if not effective an
endoscopic sphincterotomy
73
D. β-blockers, if not effective an E. Urobilin in urine
endoscopic sphincterotomy
E. No treatment 65. An 70-year-old male presents with a
clinical diagnosis of acute cholangitis.
62. A 43-year-old woman undergoes open Which organism is most likely involved
cholecystectomy. Intraoperative in the pathogenesis of ascending
cholangiogram revealed multiple stones in cholangitis?
the CBD. Exploration of the CBD was A. Clonorchis sinensis
performed to extract gallstones. The CBD B. Escherichia coli
was drained with a #18 T-tube. After 10 C. Salmonella
days, a T-tube cholangiogram reveals a D. Staphylococcus aureus
retained CBD stone. This should be E. Clostridia
treated by which of the following?
A. Laparotomy and CBD exploration 66. Following admission to the hospital for
B. Subcutaneous heparinization intestinal obstruction, a 48-year-old
C. Antibiotic therapy for 6 months and woman states that she previously had
then reevaluation undergone cholecystectomy and
D. ERCP, endoscopic sphincterotomy, choledochoduodenostomy. The most
CBD exploration likely indication for the performance of
E. Ultrasound crushing of the CBD stone the choledochoduodenostomy was:
A. Hepatic metastasis were present
63. A 62-year-old woman who underwent B. Multiple stones were present in the
cholecystectomy and gallbladder at the previous operation
choledochoduodenostomy (CBD C. Multiple stones were present in the
duodenal anastomosis) 5 years CBD at the previous operation
previously is admitted to the hospital D. The common hepatic duct had a
with a 3-day history of upper abdominal stricture
pain, chills, fever, and dark urine. Make E. The small intestine was occluded
the diagnosis:
A. Ascending cholangitis 67. In attempting to minimize complications
B. Cancer of gallblader during cholecystectomy, the surgeon
C. Acute gastritis defines the triangle of Calot during the
D. Acute biliary pancreatitis operation. The boundaries of the triangle
E. Liver abscess of Calot (modified) are the common
hepatic duct medially, the cystic duct
64. A 62-year-old woman who underwent inferiorly, and the liver superiorly.
cholecystectomy and Which structure courses through this
choledochoduodenostomy (CBD duodenal triangle?
anastomosis) 5 years previously is admitted to A. Left hepatic artery
the hospital with a 3-day history of upper B. Right renal vein
abdominal pain, chills, fever, and dark urine. C. Right hepatic artery
What laboratory finding that supports D. Cystic artery
diagnosis ascending cholangitis? E. Superior mesenteric vein
A. Amylase elevation with normal
findings on liver studies 68. A 55-year-old white female undergoes a
B. Alkaline phosphatase elevation with laparoscopic cholecystectomy for
normal or elevated normal bilirubin symptomatic cholelithiasis. The operation
levels went well, and the patient was discharged
C. Elevated serum glutamic home. One week later, she comes to your
oxaloacetictransaminase (SGOT) office for a routine postoperative follow-
levels up. The final pathology report shows an
D. Altered urea/creatinine ratio incidental finding of a gallbladder
74
carcinoma confined to the mucosa. In Stricture of the CBD. Choose the correct
further advising the patient, you should combination.
inform her that A. 1, 2, 3
A. She should undergo radiation therapy B. 1, 3, 4
B She should undergo right hepatectomy C. 2, 4, 5
to remove locally infiltrating disease D. 1, 3, 5
C. She should undergo regional E. 2, 4, 5
lymphadenectomy
D. She requires systemic chemotherapy 73. Which factors play important role in the
E. She does not require any further pathogenesis of acute cholangitis? 1. Biliary
therapy tract obstruction; 2. Elevated intraluminal
pressure; 3. Decreased bile production by
69. Acute cholangitis is characterized by hepatocytes 4. Decreased serotonin serum
three symptoms known as Charcot’s level; 5. Infection of the bile. Choose the
triad. They are: 1). Weight loss; 2). correct combination.
RUQ abdominal pain; 3). Jaundice; 4). A. 1, 2, 3
Nausea and vomiting; 5). Fever. Choose B. 2, 3, 4
the correct combination. C. 3, 4, 5
A. 1, 2, 3 D. 1, 2, 5
B. 1, 3, 4 E. 1, 4, 5
C. 1, 4, 5
D. 2, 3, 5 74. Which condition is NOT associated with
E. 2, 4, 5 higher morbidity and mortality rate in patients
with acute cholangitis:
70. Reynolds pentad known as symptoms in A. Hypotension
patients with severe acute cholangitis. B. Acute renal failure
Which symptom do NOT belong to C. Liver abscess
Reynold’s pentad? D. Liver cirrhosis
A. Altered mental status E. Age less than 50 years
B. Arterial hypotension
C. Jaundice 75. Which symptom together with Charcot's
D. Nausea and vomiting triad is associated with acute cholangitis:
E. Fever A. Coffee round vomitus
B. Pruritus
71. Cholangiography is the most accurate and C. Hypertention
sensitive method of biliary tree D. Diarrhea
visualization. Diagnostic cholangiogram E. Constipation
can be performed in different ways. Name
them? 1). Percutaneously; 2). 76. Which of the following DOESN’T
Endoscopically; 3). Intraoperatively; 5). increase the risk of cholangitis:
Intravenous. Choose the most complete A. CBD stones confirmed on US
answer. B. Recent cholecystectomy
A. 1, 2, 3 C. Recent ERCP
B. 1, 3, 4 D. History of cholangitis in the past
C. 1, 4, 5 medical history
D. All are correct E. Recent appendectomy
E. 2, 4, 5
77. In patients with cholangitis at physical
72. Acute cholangitis is a bacterial infection examination you can reveal the following: 1.
and most commonly is associated with: 1. RUQ tenderness; 2. Fever; 3. Mental status
Gallstones; 2. Acute hepatitis; 3. Malignant changes; 4. Hypotension; 5. Tachycardia.
tumor of CBD; 4. Truncal vagotomy; 5. Choose the correct combination.
75
A. 1, 2, 3 C. Cholangitis
B. 2, 3, 4 D. Cholecystic-enteric fistula
C. 3, 4, 5 E. Gallbladder hydropsia
D. All are correct
E. 1, 4, 5 83. Which disorder can cause gallbladder
mucocele? 1. Impacted stone in the
78. Which statement, regarding acute gallbladder neck or cystic duct; 2.
cholansitis, is NOT true? Spontaneously resolved acute cholecystitis; 3.
A. Partial obstruction is associated Tumors of the CBD; 4. Extrinsic compression
with a higher rate of infection than of cystic duct by lymph nodes; 5. Ascending
complete obstruction cholangitis. Choose the correct combination.
B. The bile is normally sterile A. 1, 2, 3
C. In case of cholangitis obstruction B. 2, 3, 5
of the common bile duct occurs C. 3, 4, 5
secondary to ascending infection D. 1, 2, 4
D. The male-to-female ratio is equal E. 1, 4, 5
in cholangitis
E. The median age at presentation is 84. Which statement concerning gallbladder
between 50 and 60 years. hydrops is WRONG?
A. Acute inflammation is absent
79. Which tumor does NOT cause B. A large, painful gallbladder is found
cholangitis? on physical examination
A. Pancreatic cancer C. Laboratory test are normal
B. Cholangiocarcinoma D. Ultrasonography of the RUQ shows an
C. Ampullary cancer impacted stone in the neck of an
D. Porta hepatis tumors enlarged gallbladder
E. Hepatocellular carcinoma E. Intraoperatively, the aspirate from the
gallbladder is clear fluid (white bile)
80. Which disease is NOT responsible for the
development of cholangitis? 85. The gallbladder hydrops may develop all
A. Strictures or stenosis of ampulla Vateri complication, listed below EXCEPT:
B. Endoscopic manipulation of the CBD A. Empyema of the gallbladder
C. AIDS cholangiopathy B. Perforation of the gallbladder
D. Ascaris lumbricoides infections C. Gastric outlet obstruction
E. Later dumping syndrome D. Cholecystenteric fistula
E. GIT bleeding
81. What changes in laboratory tests can be
find in patients with acute cholangitis? 1. 86. Calculous cholecystitis may develop
Elevated bilirubin; 2. Elevated alkaline biliodigestive fistula, which can cause
phosphatase; 3. Elevated C-reactive protein; gallstone ileus. In this case fistula is a
4. Elevated ESR; 5. Leukocytosis connection between gallbladder and..? 1.
A. All are present Duodenum; 2. Sigmoid colon; 3. Transversus
B. 1, 2, 3 colon; 4. Stomach; 5. Bladder. Choose the
C. 1, 3, 5 correct combination.
D. 1, 2, 4 A. 1, 2, 3
E. 2, 4, 5 B. 1, 3, 4
C. 3, 4, 5
82. On X-ray film air in the biliary tree was D. 1, 2, 5
detected, which condition does NOT cause E. 2, 4, 5
this?
A. Recent papillotomy 87. What hormone produces β-cells of
B. Emphysematous cholecystitis pancreatic gland:
76
A. Somatostatin; E. CT.
B. Somatotropin;
C. Insulin; 93. A 60-year-old alcoholic is admitted to the
D. Glucagon; hospital with a diagnosis of acute
E. Pancreozymin. pancreatitis. Upon admission, his white
blood cell (WBC) count is 21x109/l. His
88. Which ethiological factors causes acute pancreatitis lipase, lactate dehydrogenase and aspartate
most oftenly? aminotransferase are elevated, blood
A. Abdominal trauma, alimentary factor; glucose is 10 mmol/L. Which of the
B. Gallstones, alcohol; following is TRUE?
C. Hypercalcemia, hyperlipidemia; A. This patient is expected to have a
D. Drugs, toxins; mortality rate of less than 5%
E. ERCP, sphincter of Oddi disfunction. B. The patient’s lipase level is not
important indication of prognosis
89. Which thesis is correct according to C. This patient requires immediate surgery
pathogenesis of acute pancreatitis? D. A venous blood gas would be helpful in
A. Intrapancreatic enzyme’s activation; assessing the severity of illness in this
B. Pancreatic gland autolisys; patient
C. Neutrophils chemoactivation, E. A serum calcium level of 1.7 mmol/L
infiltration and inflamation; on the second hospital day is a bad
D. SIRS; prognostic sign
E. All are correct.
94. A 40-year-old alcoholic male is admitted
90. A 52-year-old male, without alcohol with severe epigastric pain radiating to
history, admitted to hospital with acute the back. Serum amylase level is
pain in epigastrium, radiating to the reported as normal (fluoroscopic
back. Amylase level elevated. Which method), but serum lipase is elevated.
radiological findings will help The serum is noted to be milky in
diagnosing acute pancreatitis? appearance. A diagnosis of pancreatitis
A. Changes in liver on CT scans; is made. The serum amylase is normal
B. Choledocholithiasis on US; because:
C. Stomach shifted anteriorly on contrast A. The patient has chronic renal failure
examination of GIT; B. The patient has hyperlipidemia
D. Cloiber cups on plain abdominal film; C. The patient has alcoholic cirrhosis
E. Air below diaphragm on plain D. The patient has alcoholic hepatitis
abdominal film. E. The diagnosis of pancreatitis is
incorrect
91. What is the volume of pancreatic juice
produced in 24 hours? 95. A 57-year-old woman is admitted to the
A. 200-500 ml; hospital with abdominal pain. She
B. 100-200 ml; reports that she drinks alcohol only at
C. 500-1000 ml; social occasions. The amylase is
D. 700 ml; elevated to 120 mmol/l. Which
E. 2500 ml following x-ray finding would support a
diagnosis of acute pancreatitis?
92. What instrumental examination should A. Hepatic lesion on CT scan
be performed to diagnose necrotizing B. Gas in abdominal cavity
pancreatitis? C. Dilated colon transversum
A. US; D. Large loop of colon in the RUQ
B. ECRP; E Irregular cutoff of the CBD on
C. Plain abdominal film; cholangiogram
D. Blood gases;
77
96. Following a motor vehicle accident a epigastrium with irradiation to the back,
truck driver complains of severe which develops after alcohol consumption.
abdominal pain. Serum amylase level is Acute pancreatitis was diagnosed. The patient
markedly increased to 400 mmol/l. Grey was performed CT scan: pancreatic
Turner’s sign is seen in the flanks. enlargement. What is the stage of acute
Pancreatic trauma is suspected. Which pancreatitis according to CT picture?
statement is true of pancreatic trauma? A. Grade A
A. It is oftenly caused by blunt injuries B. Grade B
B. It is usually an isolated single-organ C. Grade C
injury D. Grade D
C. It often requires a total E. Grade E
pancreatectomy
D. It may easily diagnosed at operation 101. Acute pancreatitis is most rearly seen in:
E. It is proved by the mildly elevated A. > 60 years old women
amylase level B. 21-45 years old men
C. 45-60 years old women
97. The severity of pancreatitis can be D. 45-60 years old men
estimated with: E. Children
A. Ranson Criteria
B. Alvarado scale 102. How peritoneum covers pancreatic
C. US examination results gland?
D. SOFA scale A. From all sides
E. CXR B. From anterior and posterior sides
C. From anterior and inferior sides
98. Clinical staging (Atlanta consensus) of D. Covers only anterior surface of the gland
acute pancreatitis consists of the E. Covers anterior and posterior sides only of
following forms of the disease: 1. the head of the gland
Pseudotumorous pancreatitis. 2. Mild
acute pancreatitis. 3. Sterile 103. For clinical picture of acute pancreatitis
pancreonecrosis. 4. Infected are typical all signs except:
pancronecrosis. 5. Pseudocyst of the A. Continuous severe pain located in the
pancreas. Select the correct combination epigastrium and/or left upper quadrant
of answers: B. The pain irradiates to the back
A. 2,3,4 C. Multiple vomiting
B. 1,2,3,5 D. Paralytic ileus
C. 3,4 E. Absence of liver dullness
D. 2,3,4,5
E. All correct 104. In differential diagnosis of acute
pancreatitis and acute bowel obstruction will
99. Enzyme toxemia in case of pancronecrosis be usefull: 1. CBC; 2. Liver enzymes serum
is caused by: 1. Trypsin. 2. levels; 3. Serum amylase and lipase levels; 4.
Phospholipase A2. 3. Chymotrypsin. 4. Plain abdominal film; 5. Serum electrolytes.
Elastase. 5. Enterokinase. Select the Choose best combination.
correct combination of answers: A. 1, 2
A. 1,4 B. 2, 3
B. 2,3,5 C. 3, 4
C. 1,2,3,4 D. 1, 5
D. 1,3,4,5 E. 2, 5
E. All correct
105. Choose changes in biochemical test
100. A 45-year-old male addmitted to the typical for patients with acute pancreatitis: 1.
surgical department with severe pain in Hypertriglyceridemia; 2. Hyperglycemia; 3.
78
Hypercalcemia; 4. Hypotriglyceridemia; 5. D. Pancreatic abscess;
Hypocalcemia. E. Cancer of the head of the pancreas.
A. 1, 2, 5
B. 1, 3, 4 110. The early complications of acute
C. 2, 3, 4 pancreatitis are all, except:
D. 3, 4, 5 A. Pancreatic shock;
E. 1, 2, 4 B. Acute respiratory distress syndrome;
C. Enzyme peritonitis;
106. Most common cause of death in early D. Renal failure;
acute pancreatitis is? E. Pancreatic fistula.
A. Renal failure
B. Cardiac failure 111. When infection complications of acute
C. Respiratory failure pancreatitis develops most oftenly?
D. Uncontrolled coagulopathy A. On 7-th day of disease;
E. Sepsis B. After 2 weeks;
C. On 4-th day of disease;
107. Severity of acute pancreatitis correlate D. After 6 months;
with levels of all of the following except E. On 2-nd day of disease.
А. Glucose
B. Amylase 112. Treatment of acute pancreatitis includes
C. Transaminase all EXCEPT:
D. Calcium A. Diurhetics;
E. WBC level B. Antibiotics;
C. Intravenous hydration;
108. A 60-year-old female treated in D. Analgesics;
surgical department for acute pancreatitis. E. Nothing per os.
On 30 day of treatment in epigastrium
appeared mass 10 cm in diameter. CBC: 113. A 24-year-old college student recovers
RBC – 3,7*1012/l, WBC – 10*109/l. US from a bout of severe pancreatitis. He
examination: fluid collected near the head has mild epigastric discomfort, sensation
of the pancreatic gland. What of bloating, and loss of appetite.
complication of acute pancreatitis Examination reveals an epigastric
developed in this case? fullness that on ultrasound is confirmed
A. Postnecrotic pseudocyst; to be a pseudocyst. The swelling
B. Retroperitoneal phlegmone; increases in size over a 3-week period of
C. Enzyme peritonitis; observation to 15 cm in diameter. What
D. Pancreatic abscess; should be the next step in management?
E. Cancer of the head of the pancreas. A. Percutaneous drainage of the cyst
B. Laparotomy and internal drainage of
109. A 45-year-old male treated in surgical the cyst
department for acute pancreatitis during C. Excision of pseudocyst
28 days. He started to complain for the D. Total pancreatectomy
temperature elevation to 39°C, E. Administration of pancreatic enzymes
weakness, fatigue. CBC: RBC –
3,5*1012/l, WBC – 21*109/l. CT: 11 cm 114. A 42-year-old woman with a history of
in diameter fluid collected near the head chronic alcoholism is admitted to the
of the pancreatic gland. What hospital because of acute pancreatitis.
complication of acute pancreatitis The bilirubin and amylase levels are in
developed in this case? the normal range. An ultrasound reveals
A. Postnecrotic pseudocyst; cholelithiasis (stones in the gallbladder,
B. Retroperitoneal phlegmone; CBD 6 mm in diameter). The symptoms
C. Enzyme peritonitis;
79
decreased on the fifth day after abdominal pain, nausea, vomiting.
admission. What should she be advised? Body temperature 36,8°C. At palpation:
A. To start on a low-fat diet at home defined tumor in the epigastrium.
B To increase the fat content of her diet Laboratory tests: pancreatic amylase
C. To undergo immediate normal. On the second day in the hospital
cholecystectomy CT was held: 9 cm pseudocyst of
D. To undergo cholecystectomy during pancreas was found. Which statement
the same hospital stay as well as an regarding this patient is correct?
assessment of her bile ducts А. Pseudocyst can cause compression of
E That she will be discharged and should stomach and biliary ducts
undergo elective cholecystectomy B. Spontaneous resorption never met
after 3 months C. Pseudocyst can be seen only in case of
acute pancreatitis
115. A 26-year-old woman with a known D. In this case pseudocyst doesn’t need
history of chronic alcoholism is admitted any treatment
to the hospital with severe abdominal E. Malignant degeneration occurs in 25%
pain due to acute pancreatitis. The serum of cases, if untreated
and urinary amylase levels are elevated.
On the day following admission to the 119. Which drug is used to suppress the
hospital, there is no improvement, and secretion of the pancreas?
she has a mild cough and slight dyspnea. A. Aprotinin
What is the most likely complication? B. Octreotide
A. Pulmonary atelectasis C. Papaverine
B. Bronchitis D. Imipenem/cilastatin
C. Pulmonary embolus E. Acetaminophen
D. Afferent loop syndrome
E. Pneumonia
120. Which drug is used to inhibite proteases?
116. The most often cause of death in case of A. Aprotinin
destructive pancreatitis is: B. Octreotide
A. Pulmonary embolism C. Papaverine
B. Mechanical jaundice D. Imipenem/cilastatin
C. Peritonitis E. Acetaminophen
D. Septic complications
E. Renal failure 121. Which statement concerning medical
treatment of patient with acute
117. 30-year-old man hospitalized with a pancreatitis on early phase (first 3-5 days)
severe epigastric pain. During is WRONG?
examination: hypoxemia, dehydration. A. Patients are kept “nothing per os”
Laboratory tests: increased levels of B. Aggressive fluid resuscitation is
amylase and lipase in the blood. CT critically important
confirmed severe acute pancreatitis. C. Antibiotics should be used in any case
Which antibiotic will reduce the risk of of acute pancreatitis
infection? D. Imipenem/cilastatin is indicated to
A. Ampicillin prevent infication in patients with
B. Erythromicin pancreatic necrosis
C. Gentamicin E. Early initiation of enteral nutrition is
D. Ceftriaxon important in treatment and infection
E. Imipenem/cilastatin prevention
118. 45-year-old man who abused alcohol,
admitted to the hospital complaining of
80
122. Name the source of infection in patients
with necrotizing pancreatitis? 126. Which sign is not associated with peptic
A. Pulmonary infection ulcer perforation:
B. Bacterial translocation from gut A. Sudden onset;
C. Skin infection B. Positive Blumberg sign;
D. Urinary infection C. Free air below diaphragm on plain
E. Nosocomial infection abdominal film;
D. Cloiberg caps;
123. A 44-year-old woman is admitted to the E. Intolerable abdominal pain.
hospital because of acute pancreatitis.
Biochemical test: bilirubin – 45 mcmol/l. 127. Which ethiological factors causes peptic ulcer
An ultrasound reveals stones in disease most oftenly?
gallbladder and CBD 14 mm in A. Abdominal trauma, alimentary factor;
diameter. MRCP: 8 mm stone in the B. H. pylori, NSAID's;
distal part of CBD, which is dilated to 14 C. H. pylori, hyperlipidemia;
mm. What should she be the first step in D. Drugs, toxins;
the treatment of patient? E. NSAID's, gastrinoma.
A. Whipple procedure
B Distal resection of the pancreas 128. A 30-year-old male is operated on for
C. Sphincterotomy and stone extraction peptic ulcer perforation, in 2,5 hours
D. Only medical treatment after the beginning of the disease.
E Urgent cholecystectomy Which operation will be most efficient
(radical operation for peptic ulcer)?
124. A 47-year-old man with a known history A. Simple closure and highly selective
of chronic alcoholism is admitted to the vagotomy;
hospital with severe abdominal pain due B. Simple closure with a Graham patch
to acute pancreatitis. His general using omentum;
condition worsens in first 48 hours. CBC: C. Simple closure;
RBC – 4,4*1012/l, WBC – 11,2*109/l. On D. Antrumectomy;
second series of CT scans rapid increase E. Stomach resection.
in retroperitoneal fluid accumulation was
found. Which complication of necrotizing 129. A 42-year-old male, with previous ulcer
pancreatitis develops in this case? history and typical clinical picture of
A. Enzyme peritonitis peptic ulcer perforation, on examination,
B. Retroperitoneal bleeding in 4 hours after the beginning of the
C. Pancreatic duct disruption disease, discomfort in right upper
D. Retroperitoneal phlegmon quadrant, heart rate – 74/min., mild
E. Pancreatic abscess abdominal wall muscles rigidity,
negative Blumberg sign. Free air below
125. Choose the WRONG statement diaphragm on X-ray abdominal film.
concerning pseudocyst definition: What is you diagnosis?
A. Peripancreatic fluid collection A. Peptic ulcer recurrence;
persisting for more than 4 weeks B. Acute cholecystitis;
B. Pseudocyst lack an epithelial layer C. Covered peptic ulcer perforation;
C. Most of them are filled with necrotic D. Chronic cholecystitis;
debris E. Acute appendicitis (subhepatic
D. Intervention (minimally invasive or location).
conventional) should be performed in
case of pseudocyst complications 130. Most oftenly perforated peptic ulcers are
development located at:
E. In all cases pseudocysts should be A. Posterior wall of antrum;
treated with conventional surgery B. Cardiac part of stomach;
81
C. Fundus of stomach; A. It results in a lower incidence of
D. Posterior wall of duodenal bulb; ulcer recurrence
E. Anterior wall of duodenal bulb. B The complication rate is lower
C. It reduces acid secretion to a greater
131. Penetrated ulcer can cause such extent
complications: 1). Abdominal abscess; D. It benefits patients with antral ulcers the
2). Portal pyelophlebitis; 3). Stomach- most
organ fistula; 4). Acute pancreatitis; 5). E. It includes removal of the ulcer
Bleeding.
A. 1, 2, 3; 135. A63-year-old woman is admitted to the
B. 2, 3, 5; hospital with severe abdominal pain of 3-
C. 3, 4, 5; hour duration. Abdominal examination
D. 1, 3, 5; reveals board-like rigidity, guarding, and
E. All. rebound tenderness. Her blood pressure is
90/50 mm Hg, pluse 110 bpm (beats per
132. A 45-year-old man complains of burning minute), and respiratory rate is 30 breaths
epigastric pain that wakes him up at per minute. After a thorough history and
night. The pain is relieved by eating or physical, and initiation of fluid
using over-thecounter antacids and H2 resuscitation, what diagnostic study
blockers. Diagnosis is best confirmed by should be performed?
which of the following? A. Supine abdominal x-rays
A. Urea breath test B. Upright chest x-ray
B. Serum gastrin levels C. Gastrograffin swallow
C. Barium meal examination D. Computerized axial tomography (CAT)
D. Upper endoscopy scan of the abdomen
E. Upper endoscopy and biopsy E. Abdominal sonogram

133. A 44-year-old dentist complains of 136. A frail elderly patient is found to have an
burning epigastric pain that wakes him up anterior perforation of a duodenal ulcer.
at night caused by a recurrent duodenal He has a recent history of nonsteroidal
ulcer. He has shown considerable anti-inflammatory drug (NSAID) use and
improvement following operative no previous history of peptic ulcer
treatment by a truncal vagotomy and disease. A large amount of bilious fluid is
pyloroplasty, 10 years prior to this found in the abdomen. What should be
incident. Which is TRUE of truncal the next step?
vagotomy? A. Lavage and omental patch closure of
A. It is performed exclusively via the the ulcer
thorax B. Lavage of the peritoneal cavity alone
B. It can be performed in the neck C. Total gastrectomy
C. If complete, it will result in increased D. Lavage, vagotomy, and
acid secretion gastroenterostomy
D. It requires a gastric drainage procedure E. Laser of the ulcer
E. It has been abandoned as a method to
treat ulcer disease. 137. Name 3 main signs of perforated peptic
ulcer?
134. A 42-year-old executive has refractory A. Abdominal pain, multiple vomiting,
chronic duodenal ulcer disease. His distention of abdominal cavity
physican has suggested several surgical B. Increasing abdominal pain, meteorism,
options. The patient has chosen a parietal peritonitis
(highly selective) vagotomy instead of a C. Ulcer in past medical history, sudden
truncal vagotomy and antrectomy abdominal pain, board-like rigidity
because?
82
D. Ulcer in past medical history, pain in B. Gastric outlet obstruction;
the epigastrium, which gradually increases, C. Penetration;
peritonitis D. Ulcer malignization;
E. Abdominal pain, pale and cold skin, low E. In all cases.
BP
142. During endoscopy on the proximal 1/3 of
138. A 63-year-old male patient admitted to lesser curvature of stomach the large
hospital with complaints of general ulcer (3x3 cm) with profuse arterial
weakness, dizziness, temporary loss of bleeding was visualized. Name the
consciousness. For last three years patient bleeding vessel.
was complaining of pain in epigastric A. Right gastric artery;
area, specially at night, heartburn. Medical B. Left gastric artery;
examination was not performed. 2 weeks C. Left gastroepiploic artery;
before hospitalization, patient complaint D. Right gastroepiploic artery;
of intense pain in epigastrium. On the day E. Short gastric arteries.
of hospitalization encountered severe
weakness, nausea, dizziness, twice black 143. 53-year-old patient, alcoholic,
stool with a rotten odor and twice lost of complaining for hematemesis that
consciousness. CBC: hemoglobin – 91 follows episodes of intense vomiting.
g/l, RBC – 2,3x109/l. Make the diagnosis? During endoscopy: liner tears below the
A. Bleeding from duodenal ulcer; gastroesophageal junction. Make the
B. Stomach cancer complicated with diagnosis?
bleeding; A. Peptic ulcer complicated with
C. Acute pancreatitis; bleeding;
D. Myocardial infarction; B. Mallory-Weiss syndrome;
E. Esophageal varices. C. Esophageal varices;
D. Dieulafoy’s lesion;
139. 63-year-old patient admitted with massive E. Gastritis.
vomiting with blood, which started
suddenly. Previous history of viral 144. Name most often cause of GIT
hepatitis. On examination: caput medusae, bleeding?
enlarged liver and spleen. What is the A. Peptic ulcer;
cause of bleeding? B. Liver cirrhosis;
A. Erosive gastroduodenitis; C. Portal hypertension;
B. Peptic ulcer disease complicated with bleeding; D. Colon diseases;
C. Esophageal varices; E. Esophagus diseases.
D. Mallory-Weiss syndrome;
E. Gastric cancer. 145. The main complication of GIT bleeding
is?
140. Choose the most efficient diagnostic A. Renal insufficiency;
method in case of peptic ulcer disease B. Liver failure;
complicated with bleeding: C. CNS hypoxia;
A. Abdominal plain film; D. Hypovolemic shock;
B. Ultrasound; E. Centralization of circulation.
C. CT scanning;
D. Endoscopy; 146. What is the most efficient and safe
E. Laparoscopy. method of bleeding stress ulcers
treatment?
141. In patient suffering from peptic ulcer A. Stomach resection;
disease the risk of bleeding is highest in B. Sengstaken-Blackmore tube;
case of: C. Combination of hemostatic and anti-
A. Ulcer perforation; acid treatment;
83
D. Endoscopic coagulation; that stopped spontaneously. She did
E. Gastrotomy and suturing of the not require transfusion. She had ingested
vessels. large amounts of aspirin in the past 4
months to relieve the pain caused by
147. Peptic ulcer complicated with bleeding severe rheumatoid arthritis. Endoscopy
is associated with such clinical signs: 1). confirms the presence of a duodenal
Increase of the pain; 2). Coffee ground ulcer. A biopsy is done. What is the next
vomitus; 3). Decrease of the pain; 4). step in the management of a duodenal
Bradycardia; 5). Melena ulcer associated with a positive biopsy
A. 1, 2, 3; for H. pylori?
B. 2, 3, 5; A. H2 blockers
C. 3, 4, 5; B. Bipolar electrocautery of the ulcer
D. 1, 3, 4; C. Triple therapy
E. All. D. Photocoagulation
E. Elective surgery
148. A 68-year-old woman has been
diagnosed with a benign ulcer on the 151. An 80-year-old grandfather gets
greater curvature of her stomach, 5 cm admitted to the hospital for a UGI bleed.
proximal to the antrum. After 3 months of He undergoes upper endoscopy and
standard medical therapy, she continues to bleeding ulcer is visualized. Attempts at
have guaiac positive stool, anemia, and endoscopic cauterization and
abdominal pain with failure of the ulcer to epinephrine injection are unsuccessful at
heal. Biopsies of the gastric ulcer have not stopping the bleeding. A previous
identified a malignancy. The next step in attempt at angioembolization was also
management is which of the following? unsuccessful. What is the next definitive
A. Treatment of the anemia and repeat all step in therapy?
studies in 6 weeks A. Elective surgery
B. Endoscopy and bipolar electrocautery B. PPI IV
or laser photocoagulation of the gastric C. Blood transfusion
ulcer D. Repeated attempts at bipolar
C. Admission of the patient for total electocanter
parenteral nutrition (TPN), treatment of E. Emergency surgery
anemia, and endoscopic therapy
D. Surgical intervention, including partial 152. A 54-year-old man presents with a
gastric resection massive UGI bleed. After resuscitation,
E. Surgical intervention, including total endoscopy is performed. No esophageal
gastrectomy varices, gastritis, or gastric ulcers are
seen. After irrigation, a pinpoint lesion is
149. Ahealthy 75-year-old man bleeds from a seen near the gastro-esophageal junction.
duodenal ucler. Medical management Make the diagnosis?
and endoscopic measures fail to stop the A. Mallory-Weiss syndrome
bleeding. What is the next step in B. Dieulafoy’s lesion
management? C. Carcinoma
A. Continued transfusion of 8 U of blood D. Gastro-intestinal stromal tumor
B. Oversewing of the bleeding point, E. Telangiectasia
vagotomy. and pyloroplasty
C. Oversewing of the bleeding point 153. A 54-year-old man presents with a
D. Administration of norepinephrine massive UGI bleed. After resuscitation,
E. Hepatic artery ligation endoscopy is performed. No esophageal
varices, gastritis, or gastric ulcers are
150. A73-year-old woman is admitted to the seen. After irrigation, a pinpoint lesion is
hospital with a mild UGI hemorrhage seen near the gastro-esophageal
84
junction.What can be said about this A. Intussusception;
disease? B. Paralytic ileus;
A. It is a carcinoid C. Volvulus;
B. It is related to alcohol use D. Hernia incarceration;
C. It is exclusively a mucosal lesion E. Spastic bowel obstruction.
D. Surgery if first-line therapy
E. Bleeding is from a submucosal vessel 159. For low large bowel obstruction are
characteristic all signs except:
154. The most efficient noninvasive A. Slow beginning;
diagnostic method for small bowel B. Abdominal distension;
obstruction is: C. Cloiberg caps on plain abdominal
A. Plain abdominal film; film;
B. Laparoscopy; D. Absence of stool;
C. Irrigography; E. Acute dehydration.
D. Colonoscopy;
E. Ultrasound examination. 160. Surgical treatment in case of acute bowel
obstruction is indicated in cases: 1.
155. Medical therapy will be effective in cases Electrolyte disbalance. 2. Abdominal cramps.
of bowel obstruction caused by: 3. Strangulation. 4. Obstruction has not
1.Volvulus; 2.Cancer of colon; 3.Paralytic resolved within 24-48 hours of concervative
ileus; 4.Spastic bowel obstruction; treatment 5. Multiple air-fluid level on plain
5.Coprostasis. abdominal film.
A. 1, 2, 3; A. 1, 2;
B. 2, 3, 4; B. 3, 4;
C. 3, 4, 5; C. 3, 5;
D. 1, 2, 5; D. 4, 5;
E. 1, 2, 4. E. 2, 3.

156. The signs of strangulation are: 1.Acute 161. A 42-year-old woman is admitted to the
onset of disease; 2.Severe abdominal pain; emergency department with severe colicky
3.Slow beginning; 4.Dull pain; 5. Dilated pain, vomiting, and abdominal distention.
small intestine loops with air/fluid levels on She has not passed stools or flatus for 48
plain abdominal film. hours. X-rays of the abdomen confirm the
A. 4, 5; presence of smallbowel obstruction. What
B. 1, 3, 4; is the most likely cause of small-bowel
C. 3, 4, 5; obstruction in the patients?
D. 1, 2, 5; A. Adhesions, inflammatory diseases;
E. 2, 3, 5. B. Helminths, tumors;
C. Groin hernia incarceration, adhesions;
157. The signs of obturation are: 1.Acute D. Gallstones, tumors;
onset of disease; 2.Severe abdominal pain; E. Tumors, groin hernia.
3.Slow beginning; 4.Dull pain; 5. Dilated air-
filled colon on plain abdominal film. 162. Most oftenly small bowel obstruction is
A. 1, 2, 3; caused by:
B. 3, 4, 5; A. Adhesions, inflammatory diseases;
C. 1, 4, 5; B. Helmints, tumors;
D. 2, 3, 5; C. Groin hernia incarceration, adhesions
E. 1, 2. D. Gallstones, tumors;
E. Tumors, groin hernia.
158. Patient with bowel obstruction is
complaining for blood in the stool. What is 163. An elderly nursing home patient is
you diagnosis? brought to the hospital with recent onset of
85
colicky abdominal pain, distension and 167. Name the scoring system which helps
obstipation on examination, the abdomen surgeon to make the prognosis in case of
is markedly distended and tympanitic. peritonitis?
There is no marked tenderness. Plain A. Manheim index;
abdominal x-ray shows a markedly B. APACHE II;
distended loop located mainly in the right C. ONTARIO Score;
upper quadrant. The likely diagnosis is: D. EuroSCORE;
A. Small-bowel obstruction; E. SOFA.
B. Chron’s disease;
C. Gallstone ileus; 168. A 68-year-old woman is admitted with
D. Mesenteric vascular occlusion; an acute surgical abdomen. After
E. Sigmoid volvulus. resuscitation with IV crystalloids
fluids and administration of
164. 56-year-old patient admitted to the antibiotics, she is taken for an
surgical department with complaints of immediate laparotomy. Perforated
abdominal pain, repeated vomiting, which diverticulitis of the sigmoid colon is
does not bring relief. The pain starts 2 found. The sigmoid colon is inflamed
hours before admission, after consumption but mobile and the mesentery contains
of large amout of food. Patient anxious, a perforated abscess. The best
pale skin, acrocyanosis, pulse 120 bpm, operation for this patient would be:
BP 90/60 mmHg. Abdomen moderately A. Insertion of a drain in the abscess;
distended in the epigastric region, in the B. Sigmoid resection including the
lower parts – sink in. On palpation: abscess and primary colon-to-colon
tenderness in the epigastrium. On anastomosis;
percussion: tympanic sound in the C. Sigmoid loop colostomy;
epigastic region, increased peristalsis. On D. Left hemicolectomy;
plain abdominal film dillated small E. Sigmoid resection and end sigmoid
intestinal loops. Make diagnosis? colostomy and oversew the rectum
A. Peptic ulcer perforation (Hartmann procedure).
B. Groin hernia incarceration
C. Acute pancreatitis 169. Which statement is wrong concerning
D. Small intestine volvulus primary microbial peritonitis?
E. Bowel obstruction caused by the tumor A. Occurs without perforation of a hollow
of large intestine viscus;
B. Occurs with perforation of a hollow
165. Which sign is characterized by rebound viscus;
tenderness over the site of abnormality in C. Caused by direct seeding of
patients with peritonitis? microorganisms;
A. Kocher’s sign; D. Seeding of microorganisms via
B. Blumberg's sign; bacterial translocation from the gut;
C. Murphy’s sign; E. Seeding of microorganisms via
D. Pasternatski’s sign; hematogenous dissemination.
E. Cullen’s sign.
170. Which statement is wrong concerning
166. What is the leading etiological factor secondary microbial peritonitis?
causing spontaneous bacterial peritonitis? A. Occurs subsequent to perforation of a
A. Candida albicans; hollow viscus;
B. Streptococcus pyogenes; B. Endogenous microbes spill out into the
C. Staphylococcus aureus; peritoneal cavity;
D. Escherichia Coli; C. Even after timely surgical intervention
E. Pseudomonas aeruginosa. and preemptive antibiotic therapy,
about 15-30% of patients demonstrate
86
ongoing infection consisting of domination;
recurrent secondary microbial D. Staphylococcus;
peritonitis, intraabdominal abscess, or E. Streptococcus
tertiary microbial peritonitis.
D. Perforation of the colon is associated 176. Choose a reason for the use of
with lower infection rates; metronidazole as a component of
E. Patients who develop secondary antibacterial therapy of patients with
microbial peritonitis should undergo diffuse peritonitis?
surgery to alleviate the source of A. Elimination of anaerobic bacteria;
ongoing peritoneal soilage. B. Elimination of gram-positive flora;
C. Elimination of gram-negative flora;
171. Which examination would be most D. Elimination of fungal infections;
efficient in finding the cause for E. Antiprotozoal antibiotoc
secondary bacterial peritonitis?
A. Laparoscopy; 177. For the clinical course of acute
B. Plain abdominal film; peritonitis three stages are typical.
C. Ultrasound; Choose the corect combination: 1.
D. CT scan; Subclinical; 2. Reactive; 3. Toxic; 4.
E. Level of CRP. Septic; 5. Terminal:
A. 1, 2, 3;
172. One of the listed below diseases didn’t B. 2, 3, 4;
cause secondary peritonitis C. 1, 2, 5;
A. Acute cholecystitis; D. 3, 4, 5;
B. Destructive appendicitis; E. 2, 3, 5
C. Acute cholangitis;
D. Bowel obstruction; 178. For clinical picture of subdiaphragmatic
E. Mesenteric infarction abscess is typical everything except:
A. Reduction of pulmonary respiratory
173. Choose the clinical sign, not typical for excursions;
acute peritonitis: B. Elevation of diaphragm dome;
A. Tachycardia; C. Reactive pleural effusion;
B. Positive Blumberg sign;
C. Muscles rigidity;
D. Leukocytosis;
D. Paralytic ileus; E. Hematemesis
E. Positive Ortner sign
179. Name the walls of inguinal canal: 1.
174. Select the method of instrumental Inguinal ligament; 2. External oblique
examination, which is not suitable for abdominal m. aponeurosis; 3.
localization of intra-abdominal abscesses: Transversalis fascia; 4. Round ligament of
A. US; the uterus; 5. M. rectus abdominis; 6.
B. Colonoscopy; Spermatic cord; 7. M. transversus
C. CT; abdominis, internal oblique abdominal m..
D. MRI; A. 1, 2, 4, 5;
E. Laparoscopy B. 1, 2, 3, 7;
C. 2, 3, 4, 5;
175. Specify the main microorganisms, which D. 3, 4, 5, 6;
are identified in the abdominal cavity of E. 4, 5, 6, 7.
patients with purulent peritonitis:
A. Monomicrobial; 180. A 60-year-old male presents with an
B. Gram-positive microorganisms inguinal hernia of recent onset. Which of the
domination; following statements are TRUE?
C. Gram-negative microorganisms
87
A. The hernia is more likely to be direct A. Incarcerated;
than indirect; B. Irreducible;
B. Presents through the posterior wall of C. Sliding;
the inguinal canal, lateral to the deep D. Richter’s;
inguinal ring; E. Interstitial.
C. Is located lateral to epigastric vessels;
D. Is more likely than a femoral hernia to 185. Which nerves can be injured during
strangulate; surgery for repair of inguinal hernia: 1).
E. The sac is congenital. Ilioinguinal n., 2). Pudendal n., 3).
Iliohypogastric n., 4). Femoral n., 5).
181. A 70-year-old cigarette smoker presents Popliteal n., 6). Genitofemoral n..
with a right inguinal mass that has enlarged A. 1, 3, 6
and has caused discomfort in recent months. B. 2, 3, 5
The swelling, which does not extend to the C. 1, 2, 3
scrotum, reduces when resting. What is the D. 3, 5, 6
likely diagnosis? E. 1, 5, 6
A. Strangulated indirect inguinal
hernia; 186. Which of the following structures would
B. Direct inguinal hernia be encountered during repair of an inguinal
C. Hydrocele hernia in a male?
D. Aneurysm of the femoral artery; A. Spermatic cord;
E. Cyst of the cord. B. Round ligament;
C. Obturator nerve;
182. Name the best tension-free method for D. Symphysis pubis
inguinal hernia repair. E. Nerve to the adductor muscles of the
A. Bassini repair; thigh.
B. Shouldice repair;
C. Stoppa repair; 187. In repair of a femoral hernia, the
D. Lichtenstein repair; structure most vulnerable to major
E. McVay (Cooper’s ligament) repair. injury lies:
A. Medially;
183. A 62-year-old male presents with an B. Laterally;
irreducible swelling and severe pain in the left C. Anteriorly;
groin. He had a known reducible hernia for 15 D. Posteriorly;
years prior to this. He had a bowel movement E. Superficially.
while in the emergency room. At surgery, a
Richter’s hernia was found. Which of the 188. A 28-year-old professional football
following statements is TRUE? player has sudden pain and swelling in
A. It presents lateral to the rectus sheath; the right groin when attempting to
B. It presents through the lumbar intercept a pass. He is admitted to the
triangle; local emergency department. On
C. It presents through the obturator examination, there is a tender swelling
foramen; in the right groin. The scrotum and
D. It contains a Meckel’s diverticulum; penis show no abnormality. What is
E. It may allow normal passage of stool. the next step in management?
A. Needle aspiration to exclude hematoma;
184. At surgery for a right inguinal hernia, a B. Forceful manual reduction;
72-year-old man is found to have a hernia sac C. Laparotomy within 20 minutes;
that is not independent of the bowel wall. The D. Preoperative preparation and
cecum forms part of the wall of the sac. Such exploration of the groin with hernia
a hernia is properly referred to as which of the repair;
following? E. Morphine and reevaluation within 12
88
hours B. It is protective against gastric
carcinoma.
189. A 45-year-old man complains of burning C. It is associated with chronic gastritis.
epigastric pain that wakes him up at night. D. It causes gastric ulcer but not duodenal
The pain is relieved by eating or using over- ulcer.
the-counter antacids and H2 blockers. E. It cannot be detected by the urea breath
Diagnosis is best confirmed by which of the test.
following?
A. Urea breath test 193. A 68-year-old woman has been
B. Serum gastrin levels diagnosed with a benign ulcer on the greater
C. Barium meal examination curvature of her stomach, 5 cm proximal to
D. Ultrasound the antrum. After 3 months of standard
E. Upper endoscopy and biopsy medical therapy, she continues to have guaiac
positive stool, anemia, and abdominal pain
190. A 44-year-old patient has refractory with failure of the ulcer to heal. Biopsies of
chronic duodenal ulcer disease. His physican the gastric ulcer have not identified a
has suggested several surgical options. The malignancy. The next step in management is
patient has chosen a parietal (highly selective) which of the following?
vagotomy instead of a truncal vagotomy and A. Treatment of the anemia and repeat all
antrectomy because? studies in 6 weeks
A. It results in a lower incidence of ulcer B. Endoscopy and bipolar electrocautery or
recurrence. laser photocoagulation of the gastric
B. It benefits patients with antral ulcers the ulcer
most. C. Admission of the patient for total
C. It includes stomach resection. parenteral nutrition (TPN), treatment
D. It left the nerve supply to pylorus intact of anemia, and endoscopic therapy
(nerves of Latarjet). D. Surgical intervention, including partial
E. It includes removal of the ulcer. gastric resection
E. Surgical intervention, including total
191. A frail elderly patient is found to have an gastrectomy
anterior perforation of a duodenal ulcer. He
has a recent history of nonsteroidal anti- 194. Over the past 6 months, a 60-year-old
inflammatory drug (NSAID) use and no woman with long standing duodenal ulcer
previous history of peptic ulcer disease. A disease has been complaining of anorexia,
large amount of bilious fluid is found in the nausea, weight loss and repeated vomiting.
abdomen. What should be the next step? She recognizes undigested food in the
A. Lavage of the peritoneal cavity alone vomitus. Examination and workup reveal
B. Lavage and omental patch closure of the dehydration, hypokalemia, and
ulcer hypochloremic alkalosis. What is the most
C. Total gastrectomy likely diagnosis?
D. Lavage, vagotomy, and A. Carcinoma of the fundus
gastroenterostomy B. Penetrating ulcer
E. Laser of the ulcer C. Pyloric obstruction due to cicatricial
stenosis of the lumen of the duodenum
192. A 37-year-old man has had recurrent D. ZES (Zollinger Ellison Syndrome)
symptoms suggestive of peptic ulcer disease E. Anorexia nervosa
for 4 years. Endoscopy reveals an ulcer
located on duodenum buld. A mucosal biopsy 195. A 2-cm ulcer on the greater curvature of
reveals Helicobacter. pylori. What is TRUE the stomach is diagnosed in a 70-year-old
about H. pylori? woman by a barium study. Gastric analysis to
A. Active organisms can be discerned by maximal acid stimulation shows achlorhydria.
serology. What is the next step in management?
89
A. Antacids, H2 blockers, and repeat C. Superior mesenteric vein,
barium study in 6 to 8 weeks inferior mesenteric vein
B. Proton pump inhibitor (e.g., omeprazole D. Splenic vein + inferior
C. Prostoglandin E (misoprostol) and mesenteric vein, superior
repeat barium study in 6 to 8 weeks mesenteric vein
D. Immediate elective surgery E. Splenic vein, superior
E. Upper endoscopy with multiple biopsies mesenteric vein, inferior
(at least 8 or 9) for the ulcer mesenteric artery

196. A 9-year-old girl had multiple episodes 200. Budd-Chiari syndrome develops due to:
of upper GI bleeding. Contrast enhanced CT A. Thrombosis of hemorrhoidal
scan showed multiple cavernous veins
malformation surrounding the portal vein. She B. Thrombosis of hepatic veins
is admitted with severe hematemesis and C. Thrombosis of superior
melena. At birth, she had developed an mesenteric vein
infection around the umbilicus. What is the D. Thrombosis of inferior
most likely site of bleeding? mesenteric vein
A. Mallory-Weiss tear of the E. Thrombosis of splenic vein
lower end of the esophagus
B. Duodenal varices 201. The portal vein drains blood from the
C. Peptic ulcer following organs, except:
D. Esophageal varices A. liver
E. Meckel's diverticulum B. spleen
C. pancreas
197. A 43-year-old man with chronic hepatitis D. small intestines
and liver cirrhosis is admitted with upper GI E. stomach
bleeding. He has marked ascites and shows
multiple telangiectasias, liver palmar 202. What is the most common complication
erythema, and clubbing. A diagnosis of of portal hypertension:
bleeding esophageal varices secondary to A. chronic pancreatitis
portal hypertension is made. Portal pressure is B. obstructive jaundice
considered elevated when it is above which of C. hematochezia
the following? D. liver cirrhosis
A. 0.15 mm Hg E. variceal hemorrhage
B. 1.5 mm Hg
C. 10 mm Hg 203. Determining the cause of portal
D. 40 mm Hg hypertension involves all except (choose one
E. 105 mm Hg wrong answer):
A. chronic fever
198. For how many segments the liver is B. history of alcohol abuse
divided? C. umbilical infection
A. 11 D. history of blood transfusions,
B. 9 intravenous drug use (hepatitis
C. 8 B and C)
D. 7 E. history of jaundice
E. 5
204. Signs of portosystemic collateral
199. Name the main vessels which forms formation include the following: 1.
portal vein Dilated veins in the anterior abdominal
A. Azygos veins wall (umbilical epigastric vein shunts) 2.
B. Hepatic veins Venous pattern on the flanks (portal-
parietal peritoneal shunting) 3.
90
Paraumbilical hernia 4. Rectal C. US investigation with
hemorrhoids 5. Ascites - Shifting dullness Duplex-Doppler
and fluid wave 6. Caput medusa (tortuous D. upper GI endoscopy
collaterals around the umbilicus).Choose E. hepatic venous pressure
the correct combination of answers: gradient (HVPG) measurement
A. 2, 3, 5, 6.
B. 1, 2, 3, 5, 6. 209. Patient with 10-year history of hepatitis
C. 1, 2, 4, 6. C is admitted to emergency department with
D. 2, 4, 5, 6. massive GI bleeding. Within admission there
E. 1, 2, 4, 5, 6. were signs of rebleeding, but patient
complains on severe weakness and dizziness.
205. US features suggestive of hepatic What complications can develop in this case :
cirrhosis with portal hypertension include the 1. hepatic encephalopathy, 2. bronchial
following: 1. nodular liver surface or nodular aspiration, 3. renal failure, 4. portal vein
regenerative hyperplasia. 2. enlarged IVC. 3. thrombosis, 5. acute liver necrosis.
splenomegaly. 4. "portal" gastropathy. 5. A. 1, 3, 4.
ascitis. 6. portal vein cavernous formation. B. 1, 3, 5.
Exclude false answers: C. 1, 2, 3.
A. 3, 4, 6. D. 2, 3, 5.
B. 2, 4. E. 2, 4, 5.
C. 2, 4, 6.
D. 1, 2, 4. 210. 62-year old man with long history of
E. 1, 3, 5. liver cirrhosis developed massive variceal
bleeding. Physician performed upper
206. Name scoring system used to evaluate endoscopy and variceal bleeding was
severity of liver cirrhosis: confirmed. First attempt of endoscopic
A. APACHE II variceal ligation failed. Balloon-tube
B. Glasgow score tamponade was considered to stop the
C. Child-Pugh scoring system bleeding. For how long Balloon-tube
D. Ranson Criteria tamponade is allowed?
E. Alvorado score A. 6-9 h
B. up to 24 h
207. To evaluate liver function abnormalities C. only within resuscitation
all listed below laboratory test should be D. 1-2 h
performed, except: E. 2-3 days
A. Viral hepatitis serologies
B. Platelet count 211. Choose wrong statement concerning
C. Prothrombin time blood supply of liver:
D. Albumin A. The liver receives a dual blood supply
E. Liver function tests from both the portal vein and the
hepatic artery
208. All patients with cirrhosis should be B. 75% of flow from the portal vein and
considered for the presence of varices at the 25% from the hepatic artery
time of the initial diagnosis of cirrhosis. C. Venous drainage is via the hepatic
Gastroesophageal varices confirm the veins, which drain directly into the
diagnosis of portal hypertension. What inferior vena cava
examination is the most reliable in revealing
the varices:
D. Portal flow is increased by food intake
A. antegrade cholangiogram E. The liver receives blood supply only
B. liver-spleen scan (with from the proper hepatic artery
technetium sulfur colloid)
91
212. Under the normal conditions per 24 are used to PREVENT initial bleeding by
hours the liver produces bile in the volume: lowering the portal preassure?
A. 50-100 ml A. α-blockers
B. 150-200 ml (phenoxybenzamine,
C. 1500-2000 ml phentolamine)
D. 600-1000 ml B. β-blockers (propranolol,
E. 2000-2500 ml nadolol)
C. antibiotics (cefazolin,
213. Choose the reasons which cause ceftriaxone)
presinusoidal (prehepatic) portal D. Lactulose
hypertension: 1. Budd–Chiari syndrome; 2. E. Diuretics (furosemide)
Viral hepatitis C; 3. Thrombosis of portal vein
or one of it’s major branches; 4. Thrombosis 217. A 49-year-old man with a history of
of inferior vena cava 5. Alcohol liver cirrhosis is admitted with significant
cirrhosis; 6. Umbilical infection in infants. hematemesis. There is jaundice and clubbing
A. 1, 3 of the fingers. His extremities are cold and
B. 3, 6 clammy, and the systolic blood pressure drops
C. 1, 4 to 84 mm Hg. After BP stabilization, which
D. 3, 5 drugs should be used to STOP bleeding?
E. 4, 6 A. phenoxybenzamine or
phentolamine
214. Choose the reasons which cause B. propranolol or nadolol
sinusoidal (hepatic) portal hypertension: 1. C. furosemide
Budd–Chiari syndrome; 2. Viral hepatitis C; D. vasopressin or octreotide
3. Thrombosis of portal vein or one of it’s E. prednisolon or
major branches; 4. Thrombosis of inferior hydrocortisone
vena cava 5. Alcohol liver cirrhosis; 6.
Umbilical infection in infants. 218. A 49-year-old man with a history of
A. 1, 3 cirrhosis is admitted with significant
B. 3, 6 hematemesis. There is jaundice and clubbing
C. 1, 4 of the fingers. His extremities are cold and
D. 3, 5 clammy, and the systolic blood pressure drops
E. 2, 5 to 84 mm Hg. After BP stabilization, which
intervention should be performed primarily to
215. Choose the reasons which cause STOP bleeding?
postsinusoidal (posthepatic) portal A. Urgent endoscopy and endoscopic
hypertension: 1. Budd–Chiari syndrome; 2. bleeding management (band ligation
Viral hepatitis C; 3. Thrombosis of portal vein or sclerotherapy)
or one of it’s major branches; 4. Thrombosis B. Open surgery to stop the bleeding
of inferior vena cava 5. Alcohol liver C. Sengstaken-Blakemore tube
cirrhosis; 6. Umbilical infection in infants.
A. 1, 5
D. Emergency lienorenal shunt
B. 3, 6 E. Vagotomy
C. 1, 4
D. 3, 5 219. A 42-year-old woman with a known
E. 4, 6 history of esophageal varices secondary to
hepatitis and cirrhosis is admitted with
216. The high mortality associated with first severe hematemesis from esophageal
variceal bleeding episodes has led to the varices. Bleeding persists after
investigation of a variety of methods of vasopressin therapy, after endoscopic
prevention of initial bleeding. Which drugs bleeding management (band ligation and
sclerotherapy). What should be the next
92
step in management? E. Laboratory tests are performed
A. Emergency portocaval shunt to establish the etiology of liver
B. Emergency lienorenal shunt cirrhosis
C. Splenectomy
D. Vagotomy 222. To determine the etiology of liver
E. Transjugular intrahepatic cirrhosis laboratory test should be performed.
portasystemic shunt (TIPS) Which tests will be positive or increased in
case of cholestatic liver cirrhosis? Choose
220. A 25-year-old man from rural district on correct combination: 1. Alcohol screening; 2.
US was diagnosed a cyst 60 mm in diameter, Gamma glutamyl transpeptidase; 3. ALT is
serological test for antigens specific for greater than AST; 4. AST is greater than
Echinococcus granulosus was positive. Which ALT; 5. Viral serology (HBV, HCV); 6.
statement is wrong concerning treatment of Alkaline phosphatase.
this patient? A. 1, 2
A. Systemic antihelminthic agents are B. 2, 3
generally very effective against C. 2, 4
human Echinococcus D. 2, 5
B. Surgery remains the standard E. 2, 6
approach for hydatid disease of the
liver 223. To determine the etiology of liver
C. Surgery is indicated in all patients cirrhosis laboratory test should be performed.
with symptomatic disease Which tests will be positive or increased in
D. Surgical treatment should be case of alcohol liver cirrhosis? Choose correct
considered in patients with combination: 1. Alcohol screening; 2. Gamma
asymptomatic disease discovered glutamyl transpeptidase; 3. ALT is greater
accidentally, when the cyst is large than AST; 4. AST is greater than ALT; 5.
(>5 cm) Viral serology (HBV, HCV); 6. Alkaline
phosphatase.
E. To be curative, surgical therapy A. 1, 2
must remove all the living parasite
B. 1, 3
and leave no viable daughter cysts
C. 1, 4
or protoscolices
D. 1, 5
E. 1, 6
221. Which statement is wrong concerning
liver cirrhosis?
224. To determine the etiology of liver
A. Cirrhosis is a histologic diagnosis, cirrhosis laboratory test should be performed.
based on three essential criteria: Which tests will be positive or increased in
diffuse disease, presence of case of viral liver cirrhosis? Choose correct
fibrosis, and replacement of combination: 1. Alcohol screening; 2. Gamma
normal architecture by abnormal glutamyl transpeptidase; 3. ALT is greater
nodules than AST; 4. AST is greater than ALT; 5.
B. Cirrhosis is a reversible process Viral serology (HBV, HCV); 6. Alkaline
C. Morphologicaly liver cirrhosis can phosphatase.
be devided into two groups: A. 1, 5
macronodular (>3 mm) and B. 2, 5
micronodular (<3 mm) C. 3, 5
D. Clinicaly cirrhosis is classified to D. 4, 5
two stages: compensated and E. 5, 6
decompensated
225. *In 50-year-old patient., suffering from
Budd-Chiari`s syndrome, appeared
progressive pain in right subcostal area,
93
jaundice, varicose veins of the esophagus, B. US
rectum, abdominal wall, ascites, C. Radioisotope scanning
splenomegaly. Liver cirrhosis was D. Needle biopsy with histological
diagnosed. What was the mechanism of examination
liver cirrhosis development? E. Biochemical blood test
A. Cholestatic hepatitis
B. Portal hypertension 230. A patient delivered to the emergency
C. Autoimmune hepatitis department with obstructive jaundice. Choose
D. Toxic hepatitis clinical signs which are typical for obstructive
E. Viral hepatitis jaundice caused by choledocholisiasis: 1.
Upper right quadrant pain; 2. Itch; 3. Back
226. *In patient M. for a long time suffering pain; 4. Vomiting; 5. Discolouration of stool:
from liver cirrhosis, recently appeared A. 1, 2, 5
complaints for moderate pain in the B. 2, 3, 5
epigastric area, constant bloating, which C. 1, 4, 5
aggravated after eating. Objective: caput D. 3, 4, 5
medusae on abdominal wall, signs of free E. 1, 2, 3
fluid in the abdominal cavity, enlarged
liver and spleen. US: expansion of portal 231. 67-year-old man was made diagnosis cancer of
vein, increased liver and spleen. Which pancreas, tumor is located in the head of the gland
complication of liver cirrhosis we can and causes obstructive jaundice. Which
think about? clinical signs are typical for obstructive
A. Portal vein thrombosis jaundice caused by cancer of the pancreas:
B. Hepatocellular failure 1. Upper right quadrant pain; 2. Nausea; 3.
C. Portal hypertension Palpable enlarged gallblader; 4. Weight
D. Peritonitis loss; 5. Discolouration of stool:
E. Intestinal dysbacteriosis A. 1, 3, 4
B. 2, 3, 4
227. Choose the symptom of portal C. 1, 2, 3
hypertension: D. 3, 4, 5
A. Tongue papilla atrophy E. 2, 3, 5
B. Jaundice
C. Ascites 232. A 58-year-old man with a 20-year
D. Erythema of palms history of alcoholism and pancreatitis is
E. Increased levels of liver admitted to the hospital with an elevated
enzymes in serum bilirubin level to 65 µmol/L, acholic stools,
and an amylase level of 120 U*H/L.
228. Select the causes of ascites in patients Obstructive jaundice in case of chronic
with liver cirrhosis: 1. Portal hypertension pancreatitis usually results from which of the
2. AST and ALT high levels 3. following?
Secondary hyperaldosteronism 4. A. Sclerosing cholangitis
Hypoalbuminemia 5. High level of B. Common bile duct compression
bilirubin in the blood serum. caused by inflammation
A. 1, 3, 4 C. Alcoholic hepatitis
B. 1, 2, 3 D. Biliary dyskinesia
C. 2, 3, 5 E. Splenic vein thrombosis
D. 1, 3
E. All are correct 233. A 62-year-old man is admitted with
abdominal pain and weight loss of 5 kg over
229. Which method is the most efficient in the past month. He has continued to consume
diagnosing liver cirrhosis? large amounts of rum. Examination reveals
A. CT icteric sclera. The indirect bilirubin level is 50
94
µmol/L with a total bilirubin of 65 µmol/L. should be undertaken as the next step in
An ultrasound shows a 4-cm fluid collection therapy?
near the head of the pancreas. What is the A. Should be discharged home under
most likely cause of jaundice in a patient with observation
alcoholic pancreatitis? B. Should be observed in the hospital
A. Alcoholic hepatitis C. Undergo surgical exploration of the
B. Carcinoma of pancreas CBD
C. Intrahepatic cyst D. ERCP with sphincterotomy and
D. Pancreatic pseudocyst stone removal
E. Hemolytic anemia E. Anticoagulants

234. A 66-year-old man with obstructive 238. A 49-year-old African American woman
jaundice is found on ERCP to have born in New York is admitted with RUQ pain,
periampullary carcinoma. He is otherwise in fever, and jaundice (Charcot’s triad.) A
excellent physical shape and there is no diagnosis of ascending cholangitis is made.
evidence of metastasis. What is the most With regard to the etiology of ascending
appropriate treatment? cholangitis, which of the following is TRUE?
A. External radiotherapy A. It usually occurs in the absence of
B. Local excision and radiotherapy jaundice
C. Radical excision (Whipple B. It usually occurs secondary to CBD
procedure) when possible stones
D. Internal radiation seeds via catheter C. It occurs frequently after
E. Stent and chemotherapy choledochoduodenostomy
D. It does not occur in patients with
235. Which instrumental examination is cholangiocarcinoma
considered to the most efficient and safe in E. It is mainly caused by the liver
finding the cause of obstructive jaundice? insufficiency
A. CT
B. US 239. A surgeon is removing the gallbladder of
C. Plain abdominal film a 35-year-old obese man. One week
D. MRCP previously the patient had recovered from
E. ERCP obstructive jaundice and at operation,
numerous small stones are present in the
236. A 67-year-old woman is evaluated for gallbladder. In addition to cholecystectomy,
obstructive jaundice. Which biochemical test, the surgeon should also perform which of the
except bilirubin level, helps in diagnosing following?
obstructive jaundice? A. No further treatment
A. Alkaline phosphatase B. Liver biopsy
B. AST C. Intraoperative cholangiogram
C. CRP D. Removal of the head of the pancreas
D. Protein E. CBD exploration
E. Amilase
240. Choose the diseases for which Courvoisier
237. A 40-year-old man underwent sign is typical: 1. Chronic calculous
laparoscopic cholecystectomy 2 years earlier. cholecystitis; 2. Pancreatic cancer; 3.
He remains asymptomatic until 1 week before Acute pancreatitis; 4. Cancer of papilla
admission, when he complains of RUQ pain Vatteri; 5. Liver cirrhosis.
and jaundice. He develops a fever and has А. 1, 2, 5;
several rigor attacks on the day of admission. В. 3, 5;
An ultrasound confirms the presence of С. 2, 5;
gallstones in the distal CBD. The patient is D. 2, 3, 5;
given antibiotics. Which of the following Е. 2, 4.
95
A. Gallstones
241. Name the indications for Intraoperative B. Cholangitis
cholangiography: 1. Detection of stones C. Chronic pancreatitis
in CBD during palpation; 2. Suspicion D. CBD malignancy
for scar narrow of papilla Vatteri; 3. E. Acute hepatitis
Jaundice before surgical intervention; 4.
Diameter of CBD more than 10 mm; 5. 247. Extrahepatic biliary tree consists of all
Jaundice during surgical intervention. except:
А. 1, 2, 3, 4; A. Wirsung’s duct
В. 1, 3, 4; B. Common hepatic duct
С. 3, 4; C. Cystic duct
D. 1, 3, 4, 5; D. Gallbladder
Е. All are correct. E. Common bile duct

242. Choose the most serious complications of 248. What is the most common benign causes
mechanical jaundice: of obstructive jaundice?
А. violation of absorption in small A. Gallstones
intestine; B. Strictures
В. reduction of protein-synthesizing C. Chronic alcohol abuse
function of liver; D. Primary sclerosing cholangitis
С. hepatic-renal failure; E. Liver cirrhosis
D. gallbladder empyema;
Е. gallbladder hydrops. 249. What is NOT the additional source of
bilirubin production?
243. Choose the indications for choledochotomy: 1. A. Ineffective erythropoiesis
Biliary dyskinesia; 2. Acute cholangitis; B. Myoglobinemia
3. Gallbladder perforation and peritonitis; C. Cytochromes metabolism
4. Obstructive jaundice; 5. Detection of D. Large hematoma lysis
stones in CBD during palpation. E. Gallstones dissolving
А. 1, 3, 4;
В. 2, 3, 4; 250. All diseases can cause hemolysis,
С. 2, 3, 4, 5; except:
D. 2, 4, 5; A. Autoimmune disorders
Е. All are correct. B. Hypersplenism
C. Defects in hemoglobin structure
244. Which disease did not cause obstructive D. Sickle cell disease
jaundice? E. Iron deficiency anemia
A. 1,6 cm stone in gallbladder;
B. Cancer of papilla Vatteri; 251. What are the most common causes of
C. Residual stone in CBD; hepatitis? 1. Portal hypertention. 2. Alcohol.
D. Pancreatic cancer; 3. Autoimmune disorders. 4. Cholelithiasis. 5.
E. Choledocholithiasis. Viruses.
A. 2,3,5
245. Prehepatic jaundice is caused by: B. 1,2,4
A. Hemolysis C. 3,4,5
B. Gallstones D. 1,3,4
C. Pancreatic cancer E. 1,2,5
D. Renal fealure
E. Pancreatic preudocyst 252. Chronic alcohol abuse may result in: 1.
Fatty liver (steatosis). 2. Biliary tract
246. Posthepatic jaundice can be caused by all infection. 3. Gallstone formation. 4. Cirrhosis.
except: 5. Hepatitis.
96
A. 2,4,5 D. Pancreatic cancer is associated
B. 1,4,5 with positive Courvoisier's sign
C. 2,3,4 E. Common bile duct is divided into two
D. 1,2,3 parts: supraduodenal and
E. 1,3,5 retroduodenal

253. Classicaly cholangitis is diagnosed 258. All diseases listed below can lead to
clinically as syndrome known as Charcot’s extrahepatic biliary obstruction,
triad. EXCEPT:
A. Fever, hypotension and jaundice A. Choledocholithiasis
B. Fever, RUQ pain and hypotension B. Acute virual hepatitis
C. Pruritus, RUQ pain and jaundice C. Ampullary cancer
D. Pruritus, hypotention, and jaundice D. Pancreatic cancer
E. Fever, RUQ pain, and jaundice E. Biliary strictures

254. Which enzymes are markers of biliary 259. Decompression of extrahepatic biliary
hypertension? 1. Alkaline phosphatase. 2. obstruction can be achieved by three
gamma glutamyl transpeptidase. 3. methods: 1. Surgical bypass. 2.
Enterocinase, 4. Acid phosphatase. 5. AST. Percutaneous insertion of stents. 3.
A. 2,4 Endoscopic insertion of stents. 4. TIPS
B. 1,5 procedure. 5. Nasogastric decompression.
C. 2,3 A. 2,4,5
D. 1,2 B. 1,4,5
E. 3,5 C. 2,3,4
D. 1,2,3
255. Bile is produced by the liver and E. 1,3,5
contains all components, EXCEPT:
A. Bile salts. 260. Which statements, concerning jaundice,
B. Water and electrolytes. is NOT true?
C. Cholesterol. A. High levels of circulating bile salts are
D. Bilirubin. associated with pruritus
E. Vitamins A, K, D B. Jaundice, dark urine, discoloration of
stool and pruritus are the hallmark of
256. ERCP and percutaneous transhepatic obstructive jaundice
cholangiography are invasive and can be C. History of fever, biliary colic and
associated with following complications, intermittent jaundice may be
EXCEPT: suggestive of cholangitis or
A. Cholangitis choledocholithiasis
B. Biliary leakage – biliary peritonitis D. Weight loss, abdominal mass, pain
C. Pancreatitis radiating to the back and progressively
D. Bleeding deepening jaundice may be suggestive
E. Obstructive jaundice of pancreatic cancer
E. A palpably enlarged gall bladder in a
257. Which statement is NOT true? jaundiced patient is also suggestive of
A. Intrahepatic disorders can lead to chronic alcohol abuse
unconjugated-conjugated
hyperbilirubinemia 261. Complications of obstructive jaundice
B. Posthepatic disorders can cause are all of the following, EXCEPT:
conjugated hyperbilirubinemia A. Sepsis caused by cholangitis
C. Bilirubin is a breakdown product of B. Biliary cirrhosis
heme C. Pancreatitis
D. Renal and liver failure
97
E. Respiratory failure adjacent hollow viscera; 4. Exocrine
insufficiency; 5. Diabetus mellitus.
262. Chronic pancreatitis (CP) is A. 1, 2, 5
characterized by (choose correct clues of CP B. 2, 3, 4
pathogenesis): 1. Parenchymal fibrosis; 2. C. 3, 4, 5
Ductal strictures; 3. Atrophy of acinar and D. 1, 2, 3
islet tissue; 4. Pseudocyst formation; 5. CBD E. 1, 2, 4
compression.
A. 1, 2, 3 268. Longitudinal pancreaticojejunostomy is
B. All correct also known as:
C. 2, 3, 4 A. Puestow procedure
D. 3, 4, 5 B. Whiple procedure
E. 1, 2, 4 C. Billroth reconstruction
D. Bassini’s operation
263. Name main forms of CP: 1. Proliferative; E. Hartman’s procedure
2. Calcific; 3. Degenerative; 4.
Obstructive; 5. Inflamatory. 269. The most common complication of CP
A. 1, 2, 3 is:
B. 2, 3, 4 A. Mechanical jaundice
C. 2, 4, 5 B. Compression of duodenum
D. 3, 4, 5 C. Pseudocyst
E. 1, 4, 5 D. Diabetus mellitus
E. Exocrine isufficiency
264. The most often reason causing CP is:
A. Alcohol ingestion 270. A 40-year-old woman with severe
B. Gallstones chronic pancreatitis is scheduled to
C. Pancreatic cancer undergo an operation, because other forms
D. Autoimmune disorders of treatment have failed. The ultrasound
E. Trauma of the pancreas shows no evidence of pseudocyst
formation or cholelithiasis and endoscopic
265. Name main clinical signs of CP: 1. retrograde cholangiopancreatogram
Epigastric pain; 2. Vomiting; 3. Poor appetite; (ERCP) demonstrates dilated main
4. Weight loss; 5. Positive Blumberg sign in pancreatic duct (12 mm) with multiple
RUQ stricture formation. Which operation is
A. 1, 2, 3 suitable to treat this condition?
B. 2, 3, 4 A. Gastrojejunostomy
C. 3, 4, 5 B. Pancreaticojejunostomy
D. 2, 4, 5 C. Cholecystectomy
E. 1, 3, 4 D. Splenectomy
E. Subtotal pancreatectomy
266. Which instrumental examination is a
reference standard diagnostic method for 271. A 45-year-old patient with chronic
chronic pancreatitis? pancreatitis is suffering from malnutrition
A. US and weight loss secondary to inadequate
B. CT pancreatic exocrine secretions. Which is
C. MRCP TRUE regarding pancreatic secretions?
D. Plain abdominal film A. Pancreas releases fluid poor in
E. ERCP enzymes
B. Pancreas releases fluid rich in
267. Choose indications for surgical treatment enzymes and bicarbonate
of CP: 1. Pseudocyst formation; 2. Disabling C. Cholecystokinin doesn’t have
abdominal pain; 3. Disabling obstruction of influence into pancreatic secretion
98
D. All pancreatic enzymes are secreted B. 1, 3
in an active form C. 1, 4
E. The pancreas produces proteolytic D. 1, 5
enzymes only E. 2, 5

272. Which statement is WRONG concerning 276. All problems that compromise blood
anatomy of the pancreas? flow listed below can cause acute visceral
A. Pancreas is located mesoperitoneally ischemia, except?
B. Pancreas consists of head, neck, A. Acute embolic occlusion
body and tail B. Acute thrombotic occlusion
C. The head of the gland lies nestled in C. Nonocclusive mesenteric ischemia
the C-loop of the second part of the D. Splanchnic artery aneurysm
duodenum E. Mesenteric veins thrombosis
D. The tail of the gland extends
obliquely into the hilum of the 277. Which clinical sings are typical for early
spleen stages of acute visceral ischemia?
E. The neck of the pancreas overlies the A. Severe abdominal pain, vomiting,
spine, where it is susceptible to diarrhea, leukocytosis
injury in blunt abdominal trauma B. Mild abdominal pain, constipation
C. Pulsating abdominal mass
273. The islets of Langerhans are small D. No typical clinical signs
islands of endocrine cells within a sea of E. Bloody stool, signs of peritonitis
exocrine tissue, they does NOT consists
of: 278. Name later manifestations of acute
A. A-cells visceral ischemia?
B. B-cells A. Severe abdominal pain, vomiting,
C. C-cells diarrhea, leukocytosis
D. PP-cells B. Mild abdominal pain, constipation
E. D-cells C. Pulsating abdominal mass
D. No typical clinical signs
274. The pancreas has a major exocrine E. Bloody stool, signs of peritonitis
function in the production of digestive
enzymes. These include: 279. Which examination is considered to be a
A. Amylase, which functions in the gold standard for diagnosis of acute visceral
breakdown of starches ischemia?
B. Lipase, which functions to hydrolyze A. Ultrasound
fatty acids B. CT
C. Trypsin and chymotrypsin, which C. Selective mesenteric angiography
function to degrade proteins D. Duplex ultrasonography
D. Enterokinase, which functions to E. MRI
hydrolaze proteins
E. Nucleases such as deoxyribonuclease 280. Which examination is considered to be
and ribonuclease, which function to the best for screening of acute visceral
break down DNA and RNA, ischemia caused by thrombotic ischemia or
respectively venous thrombosis?
A. Ultrasound
275. Choose TWO main etiological factors B. CT
causing chronic pancreatitis: 1. C. Selective mesenteric angiography
Alcohol; 2. Autoimmune disorders; 3. D. Duplex ultrasonography
Hyperlipidemia; 4. Pancreatic duct E. MRI
obstruction; 5. Idiopathic.
A. 1, 2
99
281. A 60-year-old man with a history of 286. The intestine is viable in over 90% of
atrial fibrillation is found to severe abdominal patients if the duration of mesenteric ischemia
pain, vomiting, diarrhea, WBC=23*109/l. The symptoms lasts:
embolus is most probably originating from A. 12 hours or less
which of the following? B. 24 hours or less
A. An atherosclerotic plaque C. 36 hours or less
B. An abdominal aortic aneurysm D. 48 hours or less
C. Heart E. 72 hours or less
D. Lungs
E. Paradoxical embolus 287. Superior mesenteric artery embolism can
be caused by: 1. Aneurysm of the left
282. A 60-year-old man with a history of ventricle after myocardial infarction; 2.
atrial fibrillation is found to severe abdominal Atrial fibrillation; 3. Bacterial
pain, vomiting, diarrhea, WBC=23*109/l. endocarditis; 4. Right ventricle
What is the most appropriate surgical hypertrophy; 5. Pulmonary artery
treatment for this patient? stenosis. Choose correct combination:
A. Embolectomy А. 1, 3, 4;
B. Lumbar sympathectomy B. 1, 2, 3;
C. Bypass surgery C. 2, 3, 4;
D. Intestine resection D. 2, 3, 5;
E. Heparinization E. All are correct.

283. Name the most often cause for 288. What can cause infarction of the
mesenteric thrombosis? intestine: 1. Superior mesenteric artery
A. Blunt abdominal trauma embolism; 2. Superior and inferior mesenteric
B. Arteriosclerotic plaque artery embolism; 3. Superior mesenteric
C. Mesenteric artery aneurysm artery thrombosis; 4. Superior mesenteric vein
D. Embolus thrombosis; 5. Prolonged spasm of small
E. Vasospasm intestine arteries. Choose the BEST
combination.
284. A 66-year-old woman is admitted for Z. 1, 2, 3
hyperalimentation due to malnutrition AA. 2, 3, 4
consequent to massive small-bowel BB. 3, 4, 5
resection. What is the most likely CC. All diseases can cause intestine
condition that leads to the need to infarction
perform a massive resection? DD. None of these diseases can
A. Autoimmune disease cause intestine infarction
B. Mesenteric ischemia
C. Mesenteric adenitis 289. In a patient with superior mesenteric
D. Cancer artery embolism in stage of bowel infarction
E. Pseudomyxoma peritonei (part of small intestine necrotised) should be
performed the following operation:
285. Name the form of acute mesenteric ischemia A. Thrombectomy
which has the highest mortality rate? B. Isolated embolectomy
A. Acute embolic occlusion C. Embolectomy and resection of
B. Acute thrombotic occlusion necrotised intestine
C. Nonocclusive mesenteric ischemia D. Total colectomy
D. Splanchnic artery aneurysm E. Periarterial sympathectomy
E. Mesenteric vein thrombosis
290. A 42-years-old patient, who suffers from
mitral stenosis and atrial fibrillation, 6 hours
ago appeared severe abdominal pain,
100
vomiting, diarrhea. On examination: Septic shock; 4. Hemorrhagic shock; 5.
tenderness in mesogastrium, negative Cardiac decompensation.
Blumberg sign. CBC: Leukocytes – 21*109/l. A. 1, 2, 3
What causes acute mesenteric ischemia in this B. 2, 3, 5
case? C. 2, 3, 4
A. Acute embolic occlusion of D. None of these diseases cause
superior mesenteric artery NOMI
B. Acute thrombotic occlusion of E. All these diseases cause NOMI
superior mesenteric artery
C. Nonocclusive mesenteric ischemia 295. In patients with acute mesenteric
D. Portal vein thrombosis ischemia due to mesenteric embolism,
E. Mesenteric vein thrombosis which of the following statements is
correct?
291. Which parts of the GI tract will be A. Most oftenly embolization to
ischemic in case of thrombosis of the orifice inferior mesenteric artery is
of superior mesenteric artery? observed
A. Stomach and duodenum B. Embolus most oftenly origins
B. Stomach, duodenum and ileum from right heart
C. Small intestine, cecum, colon C. Thrombolytic therapy may be
ascendance attempted in patients without
D. Colon and rectum signs of bowel infarction or
E. All parts of small and large gastrointestinal bleeding
intestine D. Arteriography usually reveals
the embolus lodged at the
292. Which operations we can perform in case orifice of the superior
of embolic occlusion of superior mesenteric mesenteric artery
artery (choose the best combination): 1. E. At the time of exploration in
Embolectomy; 2. Embolectomy and resection case of superior mesenteric
of part of small intestine; 3. Embolectomy artery embolism, ischemia is
and left hemicolectomy; 4. Embolectomy and most severe in the left colon
right hemicolectomy; 5. Total excision of
ileum, jejunum and right hemicolectomy. 296. A 68-year-old man is admitted to the
A. 1, 2, 3 coronary care unit with an acute
B. 2, 3, 4 myocardial infarction. His postinfarction
C. 1, 4, 5 course is marked by congestive heart
D. 1, 2, 4 failure and intermittent hypotension. On
E. All operations can be the fourth hospital day, he develops
performed severe midabdominal pain. On physical
examination, blood pressure is 90/60
293. Most often cause of acute mesenteric mm Hg and pulse is 110 beats/min and
ischemia is: regular; the abdomen is soft with mild
A. Embolisation to the superior generalized tenderness and distention.
mesenteric artery Bowel sounds are hypoactive; stool
B. Thrombosis of superior mesenteric hematest is positive. The next step in this
artery patient’s management should be which
C. Nonocclusive mesenteric ischemia of the following?
D. Portal vein thrombosis A. Barium enema
E. Mesenteric vein thrombosis B. Upper gastrointestinal
endoscopy
294. What can cause nonocclusive mesenteric C. Angiography
ischemia (NOMI)? 1. Severe mesenteric D. Ultrasonography
vasoconstriction; 2. Myocardial infarction; 3. E. Celiotomy
101
D. 2, 4, 5
297. The earliest symptom of CP is E. All can be used
abdominal pain, choose WRONG
statement concerning the pain in case of 300. In patients with chronic pancreatitis
chronic pancreatitis: surgical treatment is indicated in: 1.
A. In CP, two pain patterns have been Disabling pain, which interferes with the
described: continuous and patient’s ability to work, is refractory to
intermittent pancreatic enzyme therapy, requires high
B. When pain is intermittent, episodes doses of oral narcotics; 2. Diabetes
may be separated by pain-free mellitus; 3. Malabsorption; 4. ERCP
intervals of months or years evidence of a dilated pancreatic duct (>
C. Episodes of continuous pain last 6 mm); 5. Obstruction of CBD.
from daily to 2- to 3-days per week A. 1, 2, 3
for at least 2 months B. 1, 3, 4
D. Pain appeares in the epigastric C. 1, 4, 5
region at the beginning of disease D. 2, 3, 4
with a subsequent shift to the right E. All are indications for surgical
iliac region treatment
E. Pancreatic pain is felt in the
epigastrium or upper abdomen, 301. In diagnosing CP in its early stages,
with penetration to the back or clinicians should recognize that:
radiation to the left intercostal A. Pain is present, to some degree, in
region all patients
B. Pain may be absent in 10% and more
298. With advancing disease, patients develop patients
exocrine and endocrine insufficiency and C. Direct pancreatic secretory
thus lose the ability to digest protein and assessment remains the gold
fat. Which statement is WRONG? standard for diagnosing pancreatic
A. The patien manifests diabetes insufficiency and is performed
mellitus and malabsorption on early frequently
(revesible) stage D. Radiographic imaging confirms
B. Protein malabsorption results in pancreatitis in early stages of the
creatorrhea disease
C. Since malnutrition impairs E. Endoscopic US is gold standard
immunity, incidence of infection is instrumental examination for
likely to rise among affected making the diagnosis chronic
patients pancreatitis
D. With the destruction of insulin-
producing pancreatic cells, the
302. Name the most often
patient may develop diabetes
mellitus complication of chronic
E. Fat malabsorption results in
pancreatitis:
steatorrhea
A. Left sided pleural effusion
299. Choose the combination of instrumental
B. Disseminated intravascular
examinations, which can be used in
patients with chronic pancreatitis? 1. coagulation
Enoscopic US; 2. CT; 3. MRCP; 4. Plain
C. Splenic vein thrombosis
abdominal film 5. ERCP.
A. 1, 2, 5 D. Pancreatic pseudocyst
B. 1, 3, 5
E. Pancreatic ascities
C. 1, 4, 5
102
C. All layers of the bowel wall were
303. All statements concerning Chron’s involved
disease are true, except? D. Skip lesions were noted
A. Affects all parts of GI (from the E. The preoperative GI series showed a
oropharynx to the anus) narrowing string like stricture in the
B. Nonspecific inflamation ileum (string sign)
C. Superficial (mucosal) inflamation
D. Ileocolic region affected most 308. A 35-year-old man has known ulcerative
oftenly colitis. Which of the following is an
E. Etiology is not known indication for total proctocolectomy?
A. Occasional bouts of colic and
304. All statements concerning Ulcerative diarrhea
colitis (UC) are true, except? B. Sclerosing cholangitis
A. Affects colon and rectum C. Toxic megacolon
B. Transmural inflamation D. Arthritides
C. 20-30% of patients with UC have E. Iron deficiency anemia
another family member with the disease
D. Etiology is not known 309. A 54-year-old man with diarrhea is
E. Smokers are in the risk group for UC found to have ulcerative colitis. Colectomy
should be advised in patients with
305. The typical complications of Chron’s ulcerative colitis who have symptoms that
disease are all, except: persist for more than which of the
A. Abdominal abscesses following?
B. Internal fistula A. 1 month
C. Intestinal obstruction caused by B. 6 months
strictures C. 1 year
D. Sclerosing cholangitis D. 10–20 years
E. Hemorrhage E. More than 25 years

306. A 26-year-old man is present with mild 310. A 48-year-old woman develops colon
clinical signs of Chron’s colitis. What primary cancer. She is known to have a long history
treatment this patient should be of ulcerative colitis. In ulcerative colitis,
recommended: which of the following is a characteristic of
A. Right hemicolectomy colon cancer?
B. Left hemicolectomy A. Occurs more frequently than in the
C. Treatment of anemia rest of the population.
D. Total coloproctectomy B. Is more likely to occur when the
E. Medical treatment with ulcerative disease is confined to the
aminosalicylates left colon.
C. Occurs equally in the right and left
307. A 43-year-old woman undergoes side.
investigation for colitis. In her history, it is D. Has a synchronous carcinoma in 4–
noted that 20 years earlier she underwent a 5% of cases.
surgical procedure on the large intestine. Is E. Has an excellent prognosis because
the diagnosis more likely to be ulcerative of physician awareness.
colitis rather than Crohn’s disease because
at the previous operation? 311. A 40-year-old man with a long history of
A. The serosa appeared normal on bloody diarrhea presents with increased
inspection, but the colon mucosa was abdominal pain, vomiting, and fever. On
extensively involved examination, he is found to be dehydrated
B. There was evidence of fistula and shows tachycardia and hypotension.
formation The abdomen is markedly tender with
103
guarding and rigidity. What is the most 316. What imaging study is necessary to
likely cause? make diagnosis uncomplicated perirectal
A. Toxic megacolon in ulcerative colitis abscess fistula disease?
B. Small-bowel perforation from A. Sinogram
regional enteritis B. Transrectal US
C. Perforated carcinoma of the sigmoid C. CT
colon D. No imagine study
D. Volvulus of the sigmoid colon E. MRI
E. Acute perforated diverticulitis
317. The most effective treatment, which is
312. A 25-year-old man has recurrent, successful in healing 90% of anal fisures,
indolent fistula in ano. He also complains includes: 1. Stul softeners; 2. Lexatives; 3.
of weight loss, recurrent attacks of diarrhea Antibiotics; 4. NSAIDs; 5. Sitz bath.
with blood mixed in the stool, and A. 1, 2, 3
tenesmus. Proctoscopy revealed a healthy, B. 1, 2, 4
normal-appearing rectum. What is the most C. 2, 3, 4
likely diagnosis? D. 3, 4, 5
A. Colitis associated with acquired E. 1, 2, 5.
immunodeficiency syndrome (AIDS)
B. Ulcerative colitis 318. Uncomplicated interanal hemorrhoids
C. Amoebic colitis typicaly are acossiated with:
D. Ischemic colitis A. Anorectal pain
E. Crohn’s colitis B. Pain after defecation
C. Thrombosis
313. Perirectal abscess fistulous disease is D. Perirectal abscesses
most oftenly: E. Bright-red bleeding per rectum
A. Associated with a specific systemic
disease 319. Interanal and external hemorrhoids can
B. Associated with specific infection develop all complications except:
disease A. Incarceration
C. Cryptoglandular in origin B. Necrosis
D. Associated with hemorrhoids C. Perianal condylomas
E. Etiology is not known D. Thrombosis
E. Bleeding
314. According to their location perirectal
abscesses are classified to (one answer is 320. Protoscopy reveals nonbleeding grade I
not correct): hemorrhoids. Indications for surgical
A. Superficial treatment are all except:
B. Supralevator A. III-IV grade hemorrhoids
C. Ischiorectal B. Severe bleeding
D. Intersphincteric C. Thrombosis
E. Perianal D. I-II grade hemorrhoids
E. Necrosis
315. The typical complications of perirectal
abcess are all, except: 321. Why during hemorrhoidal bleeding the
A. Internal fistula blood is bright-red?
B. Hemorrhoids A. Hemorrhoidal vein have lots of
C. Sphincter injury shunts with rectal arteries
D. Perineal sepsis B. Hemorrhoids never bleed
E. Chronic fistula C. Hemorrhoids develops from arteries
D. Bleeding is associsted with
coagulopathy
104
E. Most oftenly upper parts of colon are D. Streptococcus pyogenes
bleeding, which are richy vascularized E. Escherichia coli

328. Which of the following is clearly


322. A 44-year-old man has recurrent demarcated?
hemorrhoids. What treatment modality is A. Furuncle
not indicated in case of recurency? B. Carbuncle
A. Conventional surgery C. Cellulitis
B. Minimaly invasive treatment D. Necrotizing fasciitis
C. Increasing dietary fiber E. Erysipelas
D. decreasing constipating foods,
329. 6-years-old boy was brought to the out-
E. decreasing time spent on the toilet patient surgical department by parents
with complaints for a rash on the face. On
323. List the layers of skin from the most examination: thin-walled vesicles and
superficial to the deepest layer: 1. dermis, yellow crusts on the chin, enlarged
2. epidermis, 3. deep fascia, 4. regional lymph nodes, temperature –
subcutaneous tissue, 5. superficial fascia. 37,8̊C. What is the most probable
A. 1 2 3 4 5 diagnosis?
B. 2 3 4 5 1 A. Furuncle
C. 2 1 5 4 3 B. Carbuncle
D. 4 5 3 1 2 C. Impetigo
E. 1 2 3 5 4 D. Necrotizing fasciitis
E. Erysipelas
324. Which is the most commonly identified
infectious agent causing skin and soft tissue 330. Which skin and soft tissue infection have
infection? highest mortality rate?
A. Staphylococcus aureus A. Furuncle
B. Streptococcus pyogenes B. Carbuncle
C. Pseudomonas aeruginosa C. Cellulitis
D. Clostridium perfringens D. Necrotizing fasciitis
E. Escherichia coli E. Erysipelas
325. Erysipelas most oftenly is caused by: 331. Choose WRONG thesis concerning
A. Staphylococcus aureus necrotizing fasciitis.
B. Streptococcus pyogenes A. Necrotizing fasciitis is more
C. Pseudomonas aeruginosa commonly polymicrobial disease
D. Clostridium perfringens B. The most commonly affected sites are
E. Escherichia coli the extremities
C. Necrotizing infections typically
326. A 23 year old patient fell ill 3 weeks ago progress more rapidly (within 24–48
when she noticed a very painful hr) than more superficial cellulitic
induration in her axillary area, 4-5 days processes
later it burst and discharged a lot of pus. D. Signs and symptoms are diffuse
After this some new infiltrations appeared swelling of the affected area without
around the affected area. The patient has well-demarcated borders and pain out
never suffered from skin diseases before. of proportion to physical findings
What is the most probable diagnosis? E. CT and MRI may show air in the
A. Hydradenitis tissues or enhancement with
B. Streptococcal impetigo intravenous contrast and these signs
C. Furuncle are specific to necrotizing fasciitis
D. Mycosis
E. Carbuncle 332. Choose CORRECT thesis for the
treatment of skin and soft tissue infection
327. Which of the following is a β-haemolytic (SSTI).
group of bacteria that commonly cause A. Most SSTIs can be managed on an
purulent skin and soft tissue infection? inpatient basis
A. Staphilococcus aureus B. Superficial infections typically do not
B. Pasteurella multocida require systemic treatment and usually
C. Haemophilus influenzae respond to topical agents
105
C. SSTI caused by healthcare-associated C. Congenital disoder;
MRSA have better prognosis than D. Proteolysis;
community-associates MRSA E. Operation.
D. In the case of furuncles and
carbuncles, incision and drainage of 335. The diagnosis of pseudomembranous
abscesses is required colitis is aided by:
A. Positive blood culture for Clostridium
E. Impetigo is typicaly managed with difficile
topical agents B. Raised antibody levels in blood to
Clostridium difficile toxin
333. A 63-year-old male patient in reasonably C. High C-reactive protein level
good health suddenly suffered from fever D. Detection of Clostridium difficile toxin
(>38°C) and a painful right iliac fossa in the stool
tumefaction; no other gastrointestinal E. Isolation of Clostridium perfringens
problems were noted. On clinical from the stool
examination, a mass of about 5 cm was
palpable in correspondence of right iliac 336. The following is cause of
fossa and appeared hard, not reducible and pseudomembranous colitis in man:
fixed to the parietal muscle. Laboratory A. Clostridium histolyticum
data showed leucocytosis, shift to the left B. Clostridium septicum
and elevated erythrocyte sedimentation C. Clostridium sporogenes
rate (74 mm/hr) as pathological findings. D. Clostridium perfringens
Abdominal ultrasound examination E. Clostridium difficile
evidenced a fluid in the right lower
quadrant, with heterogenic echotexture 337. 30-year-old man hospitalized with a
and a thickening of the ileocecal tract. severe epigastric pain. During
Abdominal CT, confirmed the presence of examination: hypoxemia, dehydration.
a complex, predominantly cystic, mass of Laboratory tests: increased levels of
large size (6×8 cm) with heterogeneous, amylase and lipase in the blood. CT
mainly peripheral enhancement, the confirmed severe acute pancreatitis.
adjacent cecum had its wall thickened and Which antibiotic will reduce the risk of
it was not possible to differentiate the infection?
appendix separately from the mass, A. Ampicillin
homolateral inguinal reactive B. Erythromicin
lymphadenopathy was also present. The C. Gentamicin
patient failed to respond to the initial D. Ceftriaxon
conservative management, which E. Imipenem/cilastatin
consisted of intravenous fluids and triple
antibiotic therapy with cefotaxime, 338. Which statement concerning medical
gentamicin and metronidazole, without treatment of patient with acute
any improvement of pain and fever. At a pancreatitis on early phase (first 3-5 days)
further ultrasound examination, the mass is WRONG?
appeared not modified. Make the A. Patients are kept “nothing per os”
diagnosis? B. Aggressive fluid resuscitation is
A. Appendicular infiltration; critically important
B. Appendicular abscess; C. Antibiotics should be used in any case
C. Diffuse peritonitis; of acute pancreatitis
D. Appendicular colic; D. Imipenem/cilastatin is indicated to
E. Phlegmonous appendicitis; prevent infication in patients with
pancreatic necrosis
334. A 20-year-old man has undergone E. Early initiation of enteral nutrition is
appendectomy for perforated appendicitis important in treatment and infection
with generalized peritonitis. Seven days prevention
postoperatively, his temperature
continues to spike to 39,5°C despite 339. Which of the following is not a В-lactam
antibiotic therapy with ampicillin, antibiotic?
gentamicin, and metronidazole. A. Meropenem
Abdominal CT scan reveals a large pelvic B. Ceftriaxone
abscess. Soon afterward, he has bleeding C. Benzylpenicillin
from the mouth and nose with increasing D. Gentamicin
oozing from the surgical wound and all E. Monobactam
intravenous puncture sites. What was a
trigger for coagulopathy? 340. What is the commonest cause of
A. Sepsis; Antibiotic-associated diarrhea (choose best
B. Antibiotic therapy; answer)?
106
A. Malabsortion Evisceration; 4). Grade IV
B. Allergic reactions
C. Reversible renal impairment on liver injury; 5). Grade V spleen
accumulation injury. Choose the best
D. Clostridium difficile
E. Bone marrow depression combination.
a. 1, 2, 3
341. What is the commonest side-effect of B-
lactams? B. 1, 3, 4
A. Malabsortion C. 1, 4, 5
B. Allergic reactions
C. Reversible renal impairment on D. 2, 3, 5
accumulation E. All conditions are indications
D. Clostridium difficile for laparotomy
E. Bone marrow depression
342. What therapy is based on predicted 346. Which diagnostic procedure
susceptibility of likely pathogens and local
antimicrobial policy? should be performed at first in a
A. Targeted Therapy hemodynamically stable patient
B. Etiological therapy
C. Suceptibility-guided therapy with penetrating abdominal
D. Empiric therapy trauma?
E. Symptomatic therapy
A. Ultrasound (Focused
343. Which diagnostic procedure abdominal sonography for
should be performed at first in a trauma)
hemodynamically unstable patient B. СТ
with blunt abdominal trauma? C. Laparoscopy
A. Ultrasound (Focused abdominal D. Local wound exploration
E. Diagnostic peritoneal lavage
sonography for trauma)
B. Triple contrast CT
C. Diagnostic peritoneal lavage 347. Which diagnostic procedure
D. Laparoscopy should be performed secondly in
E. Plain abdominal film a hemodynamically stable patient
with penetrating abdominal
344. Which diagnostic procedure trauma?
should be performed at secondly in A. Ultrasound (Focused
a hemodynamically unstable patient abdominal sonography for
with blunt abdominal trauma? trauma)
A. Ultrasound (Focused abdominal B. СТ
sonography for trauma) C. Laparoscopy
B. Triple contrast CT D. Local wound exploration
C. Diagnostic peritoneal lavage E. Diagnostic peritoneal lavage
D. Laparoscopy
E. Plain abdominal film 348. Which organs are injured most
oftenly in patients with blunt
345. The indications for laparotomy in abdominal trauma: 1). Stomach;
patient with penetrating 2). Duodenum; 3). Liver; 4).
abdominal trauma are: 1). Spleen; 5). Small intestine; 6).
Hemodynamic instability; 2). Large intestine.
Positive peritoneal signs; 3). A. 1, 2
107
B. 2, 5 350. Laparoscopy in abdominal
C. 3, 4 trauma may be indicated in which of
D. 5, 6 the following?
E. 1, 6 A. To exclude diaphragmatic
injury
349. A 7-year-old boy was involved in B. In patients with multiple
a motorcycle crash while seated in previous abdominal
the back of a minivan without operations
restraints. His vital signs in the C. If there is limited
emergency room are stable but he is cardiovascular reserve
complaining of left upper quadrant D. If severe diffuse peritonitis
abdominal pain. The FAST scan exists
shows scanty fluid around in the left E. In hemodynamically
colic gutter. An abdominal and unstable patients
pelvic CT scan with iv and po
contrast is performed and the 360. A 30-year-old restrained driver
radiologist suggests a “blush” was involved in a motor-vehicle
(arterial extravasation) in the splenic crash. He is hemodynamically
parenchyma. The spleen itself stable and has a large seat belt sign
sustained a deep parenchymal tear on the abdomen. His abdomen is
and is classified as a grade III tender to palpation. In this patient
injury. The child remains one should be most concerned
hemodynamically stable. What is about:
recommended next? A. Liver and spleen injury
A. Continuous hemodynamic B. Transection of the head of the
monitoring, celiac pancreas
angiogram, and angio C. Renal pedicle avulsion
embolisation of splenic D. Hollow-viscus injuries
artery. E. Pelvic fracture
B. Immediate exploration in
the operation room 361. A 20-year-old unrestrained driver
C. If hemodynamic instability was involved in a motor-vehicle
develops, aggressive fluid crash. A computed tomography
resuscitation including a (CT) of the abdomen revealed a
repeated bolus of 20 large hematoma in the second
mL/kg lactated Ringer’s portion of duodenum. The rest of
solution followed by a the abdomen is normal. The initial
liver spleen scan management of this duodenal
D. Monitoring only hematoma should be:
E. Pneumovax and elective A. Operative evacuation
splenectomy in 6 weeks B. Nasogastric
decompression,
intravenous fluids, and
108
gradual resumption of oral pulse is 120 bpm, and respiration
diet rate is 28 breaths per minute. Her
C. Endoscopic retrograde abdomen has a stab wound in the
cholangiopancreatogram anterior axillary line at the right
(ERCP) costal margin. Two large-bore
D. Laparotomy, pyloric intravenous lines, a nasogastric
exclusion, and tube, and a Foley catheter are
gastrojejunostomy inserted. The blood pressure rises to
E. Octreotide 85 mm Hg after 2 L of Ringer’s
lactate. The appropriate
362. In a patient who had a motor-cycle management is which of the
crash, a CT of the abdomen following?
revealed a peripancreatic hematoma A. Peritoneal lavage
and indistinct pancreatic border. B. Ultrasound of the abdomen
The most definitive test for a C. Laparoscopic assessment
pancreatic injury requiring operative of the peritoneal cavity
intervention is: D. Exploratory laparotomy
A. ERCP E. CT of the head
B. Ultrasonography
C. CT scanning 365. A 22-year-old woman presents to
D. Operative exploration the emergency department with a
E. Amylase test of lavage fluid chief complaint of severe left upper
quadrant (LUQ) pain after being
363. A 25-year-old man fell down from punched by her husband. Her blood
his bicycle and hit a concrete wall pressure is 110/70 mm Hg, pulse is
on his left side. An ultrasound 100 bpm, and respiration rate is 24
examination showed free fluid in breaths per minute. The best means
the abdomen. A CT scan confirmed to establish a diagnosis is which of
a grade III splenic injury. The most the following?
important contraindication for a A. FAST
nonoperative management of the B. Physical examination
splenic injury is: C. CT of the abdomen
A. Hemodynamic instability D. Peritoneal lavage
B. Active bleeding on CT scan E. Upper gastrointestinal (GI)
C. Adult patient series
D. Lack of availability of blood
for transfusion 366. A 60-year-old man is attacked
E. Extensive associated injuries with a baseball bat and sustains
multiple blows to the abdomen. He
364. A 17-year-old girl presents to the presents to the emergency
emergency department with a stab department in shock and is brought
wound to the abdomen and a blow to the operating room (OR), where a
to the head that left her groggy. Her laparotomy reveals massive
blood pressure is 80/0 mm Hg, hemoperitoneum and a stellate
109
fracture of the right and left lobes of C. Postoperatively, if the
the liver. Which of the following patient develops fever
techniques should be used D. Postoperatively, based on
immediately? culture and sensitivity of
A. Pringle’s maneuver fecal contamination found
B. Packing the liver at the time of surgery
C. Suture ligation E. Intraoperatively, if any
D. Ligation of the right hepatic hollow viscus is found to
artery be injured
E. Ligation of the proper hepatic
artery 369. A 70-year-old woman is hit by a
car and injures her midabdomen.
367. A 23-year-old man is shot with a The best way to rule out a rupture of
handgun and found to have a the second part of the duodenum is
through-and-through injury to the by which mode?
right transverse colon. There is little A. Repeated physical
fecal contamination and no bowel examinations
devascularization. At operation, B. Ultrasound
what does he require? C. Repeated amylase levels
A. Right hemicolectomy with D. CT with oral and
ileotransverse colon intravenous contrast
anastomosis E. Peritoneal lavage
B. Right hemicolectomy with
ileostomy and mucous fistula 370. A 15-year-old girl had an injury to
C. Debridement and closure of the right retroperitoneum with
wounds with exteriorization duodenal contusion. What is the
of colon test required to exclude a rupture
D. Debridement and closure of of the duodenum?
wounds A. Serum amylase
E. Segmental resection with B. Dimethyliminodiacetic acid
primary anastomosis (HIDA) scan
C. Gastrografin study
368. A 20-year-old woman presents to D. Upper GI with barium
the emergency department with a E. ERCP
stab wound to the abdomen. There
is minimal abdominal tenderness. 371. A 33-year-old man presents to the
Local wound exploration indicates emergency department with a
that the knife penetrated the gunshot injury to the abdomen. At
peritoneum. What is the ideal use of laparotomy, a deep laceration is
antibiotic administration? found in the pancreas just to the
A. Preoperatively left of the vertebral column with
B. Intraoperatively, if a colon severance of the pancreatic duct.
injury is found What is the next step in
management?
110
A. Intraoperative cholangiogram
B. Debridement and drainage of
defect
C. Distal pancreatectomy
D. Closure of abdomen with J-P
drains
E. Vagotomy

You might also like